User login
Vigorous physical activity may lower risk of NHL
Photo by Shannon E. Renfroe
People who regularly engage in vigorous physical activity throughout their lifetime may have a lower risk of developing non-Hodgkin lymphoma (NHL), according to research published in Cancer Epidemiology, Biomarkers & Prevention.
“We know that being physically active reduces the risk of colon cancer and breast cancer, and also leads to a range of other physical and mental health benefits,” said study author Terry Boyle, PhD, of the University of British Columbia in Vancouver, Canada.
“Our findings suggest that people who do vigorous physical activity may also have a lower risk for NHL.”
Dr Boyle and his colleagues used data from a case-control study conducted between 2000 and 2004 in British Columbia. The team analyzed 749 NHL patients and 818 control subjects matched for age, gender, and residential location.
Study subjects recorded information on demographics and various risk factors for NHL, including lifetime recreational physical activity, on a questionnaire. Participants were asked to record the average number of days per week and average number of hours per day they performed mild, moderate, or vigorous physical activity for each decade of life.
The researchers defined “mild” activities as those that increase heart and breathing rates above resting level, “moderate” activities as those that increase heart rate moderately, and “vigorous” activities as those that increase breathing and heart rates to a high level. Mild and moderate activity were ultimately combined into a single category.
The team assigned a metabolic-equivalent (MET) value to the different types of physical activity.
Then, to assess the association between lifetime physical activity and NHL risk, the researchers calculated the average MET-hours per week over a lifetime for total physical activity, moderate-intensity activity, and vigorous-intensity activity. Finally, they classified participants into quartiles.
Participants who engaged in the most vigorously intense physical activity throughout their lifetime were classified in the second, third, and fourth quartiles. These subjects had about a 25% to 30% lower risk for NHL when compared to participants in the lowest (first) quartile of vigorously intense physical activity.
The adjusted odds ratio was 0.69 for the second quartile, 0.68 for the third, and 0.75 for the fourth (PTrend=0.072).
There was an inverse association between lifetime vigorous-intensity physical activity and overall NHL risk in males and females, as well as for all NHL subtypes. Furthermore, vigorous physical activity did not confer a greater benefit for any specific age group.
The researchers found no association between total lifetime physical activity and NHL risk or lifetime moderate-intensity physical activity and NHL risk.
Despite these results, Dr Boyle said there isn’t enough research on this topic to confirm that being physically active reduces the risk of NHL.
“So we are planning to pool data from several studies to investigate this topic further,” he said. “We know that different types of NHL may have different risk factors, so we are also planning to investigate whether physical activity influences the risk for different types of NHL in different ways.”
Photo by Shannon E. Renfroe
People who regularly engage in vigorous physical activity throughout their lifetime may have a lower risk of developing non-Hodgkin lymphoma (NHL), according to research published in Cancer Epidemiology, Biomarkers & Prevention.
“We know that being physically active reduces the risk of colon cancer and breast cancer, and also leads to a range of other physical and mental health benefits,” said study author Terry Boyle, PhD, of the University of British Columbia in Vancouver, Canada.
“Our findings suggest that people who do vigorous physical activity may also have a lower risk for NHL.”
Dr Boyle and his colleagues used data from a case-control study conducted between 2000 and 2004 in British Columbia. The team analyzed 749 NHL patients and 818 control subjects matched for age, gender, and residential location.
Study subjects recorded information on demographics and various risk factors for NHL, including lifetime recreational physical activity, on a questionnaire. Participants were asked to record the average number of days per week and average number of hours per day they performed mild, moderate, or vigorous physical activity for each decade of life.
The researchers defined “mild” activities as those that increase heart and breathing rates above resting level, “moderate” activities as those that increase heart rate moderately, and “vigorous” activities as those that increase breathing and heart rates to a high level. Mild and moderate activity were ultimately combined into a single category.
The team assigned a metabolic-equivalent (MET) value to the different types of physical activity.
Then, to assess the association between lifetime physical activity and NHL risk, the researchers calculated the average MET-hours per week over a lifetime for total physical activity, moderate-intensity activity, and vigorous-intensity activity. Finally, they classified participants into quartiles.
Participants who engaged in the most vigorously intense physical activity throughout their lifetime were classified in the second, third, and fourth quartiles. These subjects had about a 25% to 30% lower risk for NHL when compared to participants in the lowest (first) quartile of vigorously intense physical activity.
The adjusted odds ratio was 0.69 for the second quartile, 0.68 for the third, and 0.75 for the fourth (PTrend=0.072).
There was an inverse association between lifetime vigorous-intensity physical activity and overall NHL risk in males and females, as well as for all NHL subtypes. Furthermore, vigorous physical activity did not confer a greater benefit for any specific age group.
The researchers found no association between total lifetime physical activity and NHL risk or lifetime moderate-intensity physical activity and NHL risk.
Despite these results, Dr Boyle said there isn’t enough research on this topic to confirm that being physically active reduces the risk of NHL.
“So we are planning to pool data from several studies to investigate this topic further,” he said. “We know that different types of NHL may have different risk factors, so we are also planning to investigate whether physical activity influences the risk for different types of NHL in different ways.”
Photo by Shannon E. Renfroe
People who regularly engage in vigorous physical activity throughout their lifetime may have a lower risk of developing non-Hodgkin lymphoma (NHL), according to research published in Cancer Epidemiology, Biomarkers & Prevention.
“We know that being physically active reduces the risk of colon cancer and breast cancer, and also leads to a range of other physical and mental health benefits,” said study author Terry Boyle, PhD, of the University of British Columbia in Vancouver, Canada.
“Our findings suggest that people who do vigorous physical activity may also have a lower risk for NHL.”
Dr Boyle and his colleagues used data from a case-control study conducted between 2000 and 2004 in British Columbia. The team analyzed 749 NHL patients and 818 control subjects matched for age, gender, and residential location.
Study subjects recorded information on demographics and various risk factors for NHL, including lifetime recreational physical activity, on a questionnaire. Participants were asked to record the average number of days per week and average number of hours per day they performed mild, moderate, or vigorous physical activity for each decade of life.
The researchers defined “mild” activities as those that increase heart and breathing rates above resting level, “moderate” activities as those that increase heart rate moderately, and “vigorous” activities as those that increase breathing and heart rates to a high level. Mild and moderate activity were ultimately combined into a single category.
The team assigned a metabolic-equivalent (MET) value to the different types of physical activity.
Then, to assess the association between lifetime physical activity and NHL risk, the researchers calculated the average MET-hours per week over a lifetime for total physical activity, moderate-intensity activity, and vigorous-intensity activity. Finally, they classified participants into quartiles.
Participants who engaged in the most vigorously intense physical activity throughout their lifetime were classified in the second, third, and fourth quartiles. These subjects had about a 25% to 30% lower risk for NHL when compared to participants in the lowest (first) quartile of vigorously intense physical activity.
The adjusted odds ratio was 0.69 for the second quartile, 0.68 for the third, and 0.75 for the fourth (PTrend=0.072).
There was an inverse association between lifetime vigorous-intensity physical activity and overall NHL risk in males and females, as well as for all NHL subtypes. Furthermore, vigorous physical activity did not confer a greater benefit for any specific age group.
The researchers found no association between total lifetime physical activity and NHL risk or lifetime moderate-intensity physical activity and NHL risk.
Despite these results, Dr Boyle said there isn’t enough research on this topic to confirm that being physically active reduces the risk of NHL.
“So we are planning to pool data from several studies to investigate this topic further,” he said. “We know that different types of NHL may have different risk factors, so we are also planning to investigate whether physical activity influences the risk for different types of NHL in different ways.”
Symptoms confer higher-than-expected risk of HL, NHL
Results from two new studies indicate that lymphadenopathy and head and neck masses are associated with a higher risk of lymphoma than we thought.
These two factors proved to be the strongest predictors of Hodgkin lymphoma (HL) and non-Hodgkin lymphoma (NHL).
So unless these symptoms can be explained, general practitioners should refer affected patients to specialists as quickly as possible, study investigators said.
Both studies were published in the British Journal of General Practice.
“Cancer guidelines are based on the most robust evidence, and, up to now, this has been missing,” said Willie Hamilton, MD, of the University of Exeter Medical School in the UK.
“Our research has revealed the importance of persistent, swollen lymph glands, particularly in the neck, as part of cancer. Of course, swollen glands are common with throat infections, but in cancer, they are usually larger and painless. It’s been known for a long time that this could represent cancer. This study shows that the risk is higher than previously thought.”
The first study was a large-scale assessment of symptoms that are markers of NHL. Researchers assessed 4362 NHL patients (≥ 40 years of age) and 19,468 controls.
The 5 symptoms associated with the highest risk of developing NHL were lymphadenopathy (odds ratio [OR]=263), head and neck mass not described as lymphadenopathy (OR=49), other mass (OR=12), weight loss (OR=3.2), and abdominal pain (OR=2.5).
In the second study, investigators assessed 283 HL patients (≥ 40 years of age) and 1237 control subjects.
The team found that 6 features were independently associated with HL—lymphadenopathy (OR=280), head and neck mass not described as lymphadenopathy (OR=260), other mass (OR=12), thrombocytosis (OR=6.0), raised inflammatory markers (OR=5.2), and low full blood count (OR=2.8).
Combining the results of both studies, the investigators found that, for subjects older than 60 years of age, lymphadenopathy had a positive-predictive value of 18.6% for either NHL or HL. The positive-predictive value was 4.6% for head and neck mass and 1.1% for a mass elsewhere.
Therefore, the team said patients in this age group who present with lymphadenopathy or a head and neck mass should be referred to a specialist, unless there is a clear alternative explanation.
Referral is particularly urgent if either symptom has been present for 6 weeks or more, according to the investigators. They said that no blood test or other symptoms change that.
“Early diagnosis is vital to reducing cancer deaths,” said Liz Shephard, PhD, of the University of Exeter Medical School. “We now hope that this research will feed into guidelines to help GPs refer earlier and potentially to save lives.”
Results from two new studies indicate that lymphadenopathy and head and neck masses are associated with a higher risk of lymphoma than we thought.
These two factors proved to be the strongest predictors of Hodgkin lymphoma (HL) and non-Hodgkin lymphoma (NHL).
So unless these symptoms can be explained, general practitioners should refer affected patients to specialists as quickly as possible, study investigators said.
Both studies were published in the British Journal of General Practice.
“Cancer guidelines are based on the most robust evidence, and, up to now, this has been missing,” said Willie Hamilton, MD, of the University of Exeter Medical School in the UK.
“Our research has revealed the importance of persistent, swollen lymph glands, particularly in the neck, as part of cancer. Of course, swollen glands are common with throat infections, but in cancer, they are usually larger and painless. It’s been known for a long time that this could represent cancer. This study shows that the risk is higher than previously thought.”
The first study was a large-scale assessment of symptoms that are markers of NHL. Researchers assessed 4362 NHL patients (≥ 40 years of age) and 19,468 controls.
The 5 symptoms associated with the highest risk of developing NHL were lymphadenopathy (odds ratio [OR]=263), head and neck mass not described as lymphadenopathy (OR=49), other mass (OR=12), weight loss (OR=3.2), and abdominal pain (OR=2.5).
In the second study, investigators assessed 283 HL patients (≥ 40 years of age) and 1237 control subjects.
The team found that 6 features were independently associated with HL—lymphadenopathy (OR=280), head and neck mass not described as lymphadenopathy (OR=260), other mass (OR=12), thrombocytosis (OR=6.0), raised inflammatory markers (OR=5.2), and low full blood count (OR=2.8).
Combining the results of both studies, the investigators found that, for subjects older than 60 years of age, lymphadenopathy had a positive-predictive value of 18.6% for either NHL or HL. The positive-predictive value was 4.6% for head and neck mass and 1.1% for a mass elsewhere.
Therefore, the team said patients in this age group who present with lymphadenopathy or a head and neck mass should be referred to a specialist, unless there is a clear alternative explanation.
Referral is particularly urgent if either symptom has been present for 6 weeks or more, according to the investigators. They said that no blood test or other symptoms change that.
“Early diagnosis is vital to reducing cancer deaths,” said Liz Shephard, PhD, of the University of Exeter Medical School. “We now hope that this research will feed into guidelines to help GPs refer earlier and potentially to save lives.”
Results from two new studies indicate that lymphadenopathy and head and neck masses are associated with a higher risk of lymphoma than we thought.
These two factors proved to be the strongest predictors of Hodgkin lymphoma (HL) and non-Hodgkin lymphoma (NHL).
So unless these symptoms can be explained, general practitioners should refer affected patients to specialists as quickly as possible, study investigators said.
Both studies were published in the British Journal of General Practice.
“Cancer guidelines are based on the most robust evidence, and, up to now, this has been missing,” said Willie Hamilton, MD, of the University of Exeter Medical School in the UK.
“Our research has revealed the importance of persistent, swollen lymph glands, particularly in the neck, as part of cancer. Of course, swollen glands are common with throat infections, but in cancer, they are usually larger and painless. It’s been known for a long time that this could represent cancer. This study shows that the risk is higher than previously thought.”
The first study was a large-scale assessment of symptoms that are markers of NHL. Researchers assessed 4362 NHL patients (≥ 40 years of age) and 19,468 controls.
The 5 symptoms associated with the highest risk of developing NHL were lymphadenopathy (odds ratio [OR]=263), head and neck mass not described as lymphadenopathy (OR=49), other mass (OR=12), weight loss (OR=3.2), and abdominal pain (OR=2.5).
In the second study, investigators assessed 283 HL patients (≥ 40 years of age) and 1237 control subjects.
The team found that 6 features were independently associated with HL—lymphadenopathy (OR=280), head and neck mass not described as lymphadenopathy (OR=260), other mass (OR=12), thrombocytosis (OR=6.0), raised inflammatory markers (OR=5.2), and low full blood count (OR=2.8).
Combining the results of both studies, the investigators found that, for subjects older than 60 years of age, lymphadenopathy had a positive-predictive value of 18.6% for either NHL or HL. The positive-predictive value was 4.6% for head and neck mass and 1.1% for a mass elsewhere.
Therefore, the team said patients in this age group who present with lymphadenopathy or a head and neck mass should be referred to a specialist, unless there is a clear alternative explanation.
Referral is particularly urgent if either symptom has been present for 6 weeks or more, according to the investigators. They said that no blood test or other symptoms change that.
“Early diagnosis is vital to reducing cancer deaths,” said Liz Shephard, PhD, of the University of Exeter Medical School. “We now hope that this research will feed into guidelines to help GPs refer earlier and potentially to save lives.”
Artificial blood vessels give way to the real thing
biocompatible polymer
Photo courtesy of Vienna
University of Technology
Scientists have created implantable artificial blood vessels using a newly developed polymer that is biodegradable.
In experiments with rats, these thin-walled vascular grafts were replaced by endogenous material, ultimately leaving natural, fully functional blood vessels in their place.
Furthermore, 6 months after the artificial vessels were implanted, none of the animals had experienced thromboses, inflammation, or aneurysms.
Helga Bergmeister, MD, DVM, PhD, of the Medical University of Vienna in Austria, and her colleagues conducted this research and described the results in Acta Biomaterialia.
To create artificial blood vessels that are compatible with body tissue, the researchers developed a new polymer—thermoplastic polyurethane.
“By selecting very specific molecular building blocks, we have succeeded in synthesizing a polymer with the desired properties,” said Robert Liska, PhD, of the Vienna University of Technology.
To produce the grafts, the researchers spun polymer solutions in an electrical field to form very fine threads and wound these threads onto a spool.
“The wall of these artificial blood vessels is very similar to that of natural ones,” said Heinrich Schima, PhD, of the Medical University of Vienna.
The polymer fabric is slightly porous. So, initially, it allows a small amount of blood to seep through, which enriches the wall with growth factors. And this encourages the migration of endogenous cells.
The researchers implanted these artificial blood vessels in rats and found them to be safe and functional long-term.
“The rats’ blood vessels were examined 6 months after insertion of the vascular prostheses,” Dr Bergmeister said.
“We did not find any aneurysms, thrombosis, or inflammation. Endogenous cells had colonized the vascular prostheses and turned the artificial constructs into natural body tissue.”
In fact, natural body tissue regrew much faster than expected.
The researchers said their thin-walled grafts “offer a new and desirable form of biodegradable vascular implant.”
biocompatible polymer
Photo courtesy of Vienna
University of Technology
Scientists have created implantable artificial blood vessels using a newly developed polymer that is biodegradable.
In experiments with rats, these thin-walled vascular grafts were replaced by endogenous material, ultimately leaving natural, fully functional blood vessels in their place.
Furthermore, 6 months after the artificial vessels were implanted, none of the animals had experienced thromboses, inflammation, or aneurysms.
Helga Bergmeister, MD, DVM, PhD, of the Medical University of Vienna in Austria, and her colleagues conducted this research and described the results in Acta Biomaterialia.
To create artificial blood vessels that are compatible with body tissue, the researchers developed a new polymer—thermoplastic polyurethane.
“By selecting very specific molecular building blocks, we have succeeded in synthesizing a polymer with the desired properties,” said Robert Liska, PhD, of the Vienna University of Technology.
To produce the grafts, the researchers spun polymer solutions in an electrical field to form very fine threads and wound these threads onto a spool.
“The wall of these artificial blood vessels is very similar to that of natural ones,” said Heinrich Schima, PhD, of the Medical University of Vienna.
The polymer fabric is slightly porous. So, initially, it allows a small amount of blood to seep through, which enriches the wall with growth factors. And this encourages the migration of endogenous cells.
The researchers implanted these artificial blood vessels in rats and found them to be safe and functional long-term.
“The rats’ blood vessels were examined 6 months after insertion of the vascular prostheses,” Dr Bergmeister said.
“We did not find any aneurysms, thrombosis, or inflammation. Endogenous cells had colonized the vascular prostheses and turned the artificial constructs into natural body tissue.”
In fact, natural body tissue regrew much faster than expected.
The researchers said their thin-walled grafts “offer a new and desirable form of biodegradable vascular implant.”
biocompatible polymer
Photo courtesy of Vienna
University of Technology
Scientists have created implantable artificial blood vessels using a newly developed polymer that is biodegradable.
In experiments with rats, these thin-walled vascular grafts were replaced by endogenous material, ultimately leaving natural, fully functional blood vessels in their place.
Furthermore, 6 months after the artificial vessels were implanted, none of the animals had experienced thromboses, inflammation, or aneurysms.
Helga Bergmeister, MD, DVM, PhD, of the Medical University of Vienna in Austria, and her colleagues conducted this research and described the results in Acta Biomaterialia.
To create artificial blood vessels that are compatible with body tissue, the researchers developed a new polymer—thermoplastic polyurethane.
“By selecting very specific molecular building blocks, we have succeeded in synthesizing a polymer with the desired properties,” said Robert Liska, PhD, of the Vienna University of Technology.
To produce the grafts, the researchers spun polymer solutions in an electrical field to form very fine threads and wound these threads onto a spool.
“The wall of these artificial blood vessels is very similar to that of natural ones,” said Heinrich Schima, PhD, of the Medical University of Vienna.
The polymer fabric is slightly porous. So, initially, it allows a small amount of blood to seep through, which enriches the wall with growth factors. And this encourages the migration of endogenous cells.
The researchers implanted these artificial blood vessels in rats and found them to be safe and functional long-term.
“The rats’ blood vessels were examined 6 months after insertion of the vascular prostheses,” Dr Bergmeister said.
“We did not find any aneurysms, thrombosis, or inflammation. Endogenous cells had colonized the vascular prostheses and turned the artificial constructs into natural body tissue.”
In fact, natural body tissue regrew much faster than expected.
The researchers said their thin-walled grafts “offer a new and desirable form of biodegradable vascular implant.”
Acid exposure time found most useful in pH-impedance testing
pH-impedance testing best predicted response to reflux treatment when patients were off proton pump inhibitors, and abnormal acid exposure time was the single most useful testing parameter, a prospective study has found.
“Impedance-based reflux parameters complement but do not replace acid-based parameters in predicting symptom outcome from both medical and surgical antireflux therapy,” wrote Dr. Amit Patel and his associates at the Washington School of Medicine, St. Louis. The study appears in the May issue of Clinical Gastroenterology and Hepatology (doi:10.1016/j.cgh.2014.08.02).
“Because abnormal acid reflux time and symptom-reflux correlation parameters are detected more often when testing is performed off therapy, pH-impedance testing off antisecretory therapy maximizes prediction of symptomatic outcome from GERD [gastroesophageal reflux disease] therapy,” the researchers wrote.
Clinicians continue to debate the role of pH-impedance testing in management of GERD, whether testing should be performed on or off antisecretory therapy, and which testing parameters are most useful for predicting treatment response, the investigators noted. Therefore, they followed 187 adults with persistent GERD symptoms who underwent pH-impedance testing at their center during a 5-year period. Average age of patients was 54 years, almost 71% were female, and none had histopathologic evidence of esophageal motor disorders. Almost half had been off proton pump inhibitors for 7 days when tested, and 68% were managed medically as opposed to surgically.
Global symptom assessment (GSS) and dominant symptom intensity (DSI) scores both improved significantly during an average of 40 months of follow-up, the researchers said. After the researchers controlled for demographics, symptoms at presentation, use of proton pump inhibitors (PPI), and parameters that predicted treatment response in the univariate analyses, only abnormal acid exposure time (AET) predicted linear improvement in DSI scores (P =.027), while abnormal AET (P =.002) and symptom association probability (P =.026) predicted significant improvements in linear and dichotomous GSS, they reported. Abnormal AET and being off PPIs during testing also more than doubled the odds of at least a 50% improvement in GSS.
In contrast, dichotomous reflux exposure time did not predict response in any of the analyses, said the investigators. “Established thresholds for the total number of reflux events did not predict linear or dichotomous global symptom severity improvement,” they added. “Our data therefore suggest that performing pH-impedance testing off PPI therapy increases the yield of abnormal AET and symptom-reflux association with reflux events, facilitating predicting value for symptom improvement with both medical and surgical antireflux therapy.”
The researchers could not corroborate patients’ compliance with antisecretory therapy, assess the reasons physicians chose medical or surgical management, or evaluate the impact of the placebo effect or other factors unrelated to reflux, they said.
The study was partly funded by the National Institute for Diabetes and Digestive and Kidney Diseases, the National Institutes of Health, and the Washington University Department of Medicine Mentors in Medicine and Clinical Science Training and Research programs. The authors declared no relevant conflicts of interest.
pH-impedance testing best predicted response to reflux treatment when patients were off proton pump inhibitors, and abnormal acid exposure time was the single most useful testing parameter, a prospective study has found.
“Impedance-based reflux parameters complement but do not replace acid-based parameters in predicting symptom outcome from both medical and surgical antireflux therapy,” wrote Dr. Amit Patel and his associates at the Washington School of Medicine, St. Louis. The study appears in the May issue of Clinical Gastroenterology and Hepatology (doi:10.1016/j.cgh.2014.08.02).
“Because abnormal acid reflux time and symptom-reflux correlation parameters are detected more often when testing is performed off therapy, pH-impedance testing off antisecretory therapy maximizes prediction of symptomatic outcome from GERD [gastroesophageal reflux disease] therapy,” the researchers wrote.
Clinicians continue to debate the role of pH-impedance testing in management of GERD, whether testing should be performed on or off antisecretory therapy, and which testing parameters are most useful for predicting treatment response, the investigators noted. Therefore, they followed 187 adults with persistent GERD symptoms who underwent pH-impedance testing at their center during a 5-year period. Average age of patients was 54 years, almost 71% were female, and none had histopathologic evidence of esophageal motor disorders. Almost half had been off proton pump inhibitors for 7 days when tested, and 68% were managed medically as opposed to surgically.
Global symptom assessment (GSS) and dominant symptom intensity (DSI) scores both improved significantly during an average of 40 months of follow-up, the researchers said. After the researchers controlled for demographics, symptoms at presentation, use of proton pump inhibitors (PPI), and parameters that predicted treatment response in the univariate analyses, only abnormal acid exposure time (AET) predicted linear improvement in DSI scores (P =.027), while abnormal AET (P =.002) and symptom association probability (P =.026) predicted significant improvements in linear and dichotomous GSS, they reported. Abnormal AET and being off PPIs during testing also more than doubled the odds of at least a 50% improvement in GSS.
In contrast, dichotomous reflux exposure time did not predict response in any of the analyses, said the investigators. “Established thresholds for the total number of reflux events did not predict linear or dichotomous global symptom severity improvement,” they added. “Our data therefore suggest that performing pH-impedance testing off PPI therapy increases the yield of abnormal AET and symptom-reflux association with reflux events, facilitating predicting value for symptom improvement with both medical and surgical antireflux therapy.”
The researchers could not corroborate patients’ compliance with antisecretory therapy, assess the reasons physicians chose medical or surgical management, or evaluate the impact of the placebo effect or other factors unrelated to reflux, they said.
The study was partly funded by the National Institute for Diabetes and Digestive and Kidney Diseases, the National Institutes of Health, and the Washington University Department of Medicine Mentors in Medicine and Clinical Science Training and Research programs. The authors declared no relevant conflicts of interest.
pH-impedance testing best predicted response to reflux treatment when patients were off proton pump inhibitors, and abnormal acid exposure time was the single most useful testing parameter, a prospective study has found.
“Impedance-based reflux parameters complement but do not replace acid-based parameters in predicting symptom outcome from both medical and surgical antireflux therapy,” wrote Dr. Amit Patel and his associates at the Washington School of Medicine, St. Louis. The study appears in the May issue of Clinical Gastroenterology and Hepatology (doi:10.1016/j.cgh.2014.08.02).
“Because abnormal acid reflux time and symptom-reflux correlation parameters are detected more often when testing is performed off therapy, pH-impedance testing off antisecretory therapy maximizes prediction of symptomatic outcome from GERD [gastroesophageal reflux disease] therapy,” the researchers wrote.
Clinicians continue to debate the role of pH-impedance testing in management of GERD, whether testing should be performed on or off antisecretory therapy, and which testing parameters are most useful for predicting treatment response, the investigators noted. Therefore, they followed 187 adults with persistent GERD symptoms who underwent pH-impedance testing at their center during a 5-year period. Average age of patients was 54 years, almost 71% were female, and none had histopathologic evidence of esophageal motor disorders. Almost half had been off proton pump inhibitors for 7 days when tested, and 68% were managed medically as opposed to surgically.
Global symptom assessment (GSS) and dominant symptom intensity (DSI) scores both improved significantly during an average of 40 months of follow-up, the researchers said. After the researchers controlled for demographics, symptoms at presentation, use of proton pump inhibitors (PPI), and parameters that predicted treatment response in the univariate analyses, only abnormal acid exposure time (AET) predicted linear improvement in DSI scores (P =.027), while abnormal AET (P =.002) and symptom association probability (P =.026) predicted significant improvements in linear and dichotomous GSS, they reported. Abnormal AET and being off PPIs during testing also more than doubled the odds of at least a 50% improvement in GSS.
In contrast, dichotomous reflux exposure time did not predict response in any of the analyses, said the investigators. “Established thresholds for the total number of reflux events did not predict linear or dichotomous global symptom severity improvement,” they added. “Our data therefore suggest that performing pH-impedance testing off PPI therapy increases the yield of abnormal AET and symptom-reflux association with reflux events, facilitating predicting value for symptom improvement with both medical and surgical antireflux therapy.”
The researchers could not corroborate patients’ compliance with antisecretory therapy, assess the reasons physicians chose medical or surgical management, or evaluate the impact of the placebo effect or other factors unrelated to reflux, they said.
The study was partly funded by the National Institute for Diabetes and Digestive and Kidney Diseases, the National Institutes of Health, and the Washington University Department of Medicine Mentors in Medicine and Clinical Science Training and Research programs. The authors declared no relevant conflicts of interest.
Key clinical point: pH-impedance testing best predicted response to reflux treatment when patients were off proton pump inhibitors, and abnormal acid exposure time was the single most useful parameter.
Major finding: Acid exposure time consistently predicted response to antireflux therapy in univariate and multivariable analyses, while reflux exposure time did not.
Data source: Prospective, single-center study of 187 patients who underwent pH-impedance testing.
Disclosures: This study was partly funded by the National Institute for Diabetes and Digestive and Kidney Diseases, the National Institutes of Health, and the Washington University Department of Medicine Mentors in Medicine and Clinical Science Training and Research programs. The authors declared no relevant conflicts of interest.
Dermatology update: The dawn of targeted treatment
New targeted therapies are changing the way patients with advanced dermatologic diseases are treated. Innovative molecular biology techniques developed as far back as the 1970s have engendered tremendous insight into the cellular and molecular pathogenesis of numerous diseases. Novel medications based on these insights are now bearing fruit, as directed biologic therapies that are revolutionizing clinical practice are increasingly becoming available.
This article reviews advances in targeted therapies for advanced basal cell carcinoma, psoriasis, and metastatic melanoma.
TARGETED THERAPY FOR BASAL CELL CARCINOMA
Case 1. A 56-year-old man presents with a progressively enlarging leg ulcer. Although it has been treated empirically for years as a venous stasis ulcer, biopsy reveals that it is basal cell carcinoma. Imaging shows muscle and tendon invasion, making surgical intervention short of amputation challenging (Figure 1). What are his options?
Basal cell carcinoma is the most common cancer in humans, accounting for 25% of all cancers and more than 2 million cases in the United States every year. In most cases, surgical excision is curative, but a subset of patients have inoperable, locally advanced, or metastatic disease that drastically limits treatment options. The median survival in metastatic basal cell carcinoma is 24 months, and conventional chemotherapy has not been shown to improve the prognosis.1,2
In addition to the burden of sporadic basal cell carcinoma, patients with the rare autosomal-dominant genetic disorder basal cell nevus syndrome (Gorlin syndrome) develop multiple basal cell lesions over their lifetime. The syndrome may also involve abnormalities of the skeletal system, genitourinary tract, and central nervous system, including development of medulloblastoma.
In Gorlin syndrome, basal cell carcinomas occur often and early; about half of white patients with the syndrome develop their first lesions by age 21, and 90% by age 35. The lesions occur in multiple numbers and can develop anywhere on the body, including on non–sun-exposed areas. Patients who have Gorlin syndrome need meticulous monitoring every 2 to 3 months so that basal cell lesions can be recognized early and treated before they become locally advanced. Keeping up with the numerous medical appointments and invasive treatments can be physically and mentally taxing for patients.
Specific pathway and mutations identified
In 1996, Gorlin syndrome was found to be caused by mutations of the human homolog of the PATCHED gene, which codes for a receptor in the “hedgehog” pathway.3 Two years later, the same mutations were found to be involved in many sporadic basal cell carcinomas, and we now believe that at least 85% of basal cell carcinomas involve abnormal activation of hedgehog pathway signaling.4,5
Vismodegib developed as targeted therapy
In 2009, Robarge et al6 described a potent inhibitor of the hedgehog pathway that was later optimized for potency and desirable pharmacologic traits, resulting in the drug vismodegib.7,8
Two phase 2 multicenter clinical trials9,10 of vismodegib were published in 2012. In the first, which was not randomized,9 33 patients with metastatic basal cell carcinoma and 63 patients with locally advanced disease were treated with vismodegib. Of those with metastatic disease, 30% achieved an objective response. Of those with locally advanced disease, 43% achieved an objective response and 21% achieved a complete response.
In the second trial,10 patients with Gorlin syndrome were randomized to either vismodegib (26 patients) or placebo (16 patients). After 8 months, the vismodegib group had developed significantly fewer new surgically eligible tumors (2 vs 29 per year), their tumors were smaller (change from baseline of the sum of the longest diameters –65% vs –11%), and they needed fewer surgeries (mean 0.31 vs 4.4 per patient). No tumors progressed in the treatment group. Results in some patients were dramatic, with complete healing of large ulcerative tumors. The trial was ended early in view of significant efficacy in the treatment group.
Based on these trials, the US Food and Drug Administration (FDA) approved vismodegib for treating metastatic and locally advanced basal cell carcinoma.
Resistance and adverse effects common
Unfortunately, vismodegib has significant drawbacks. About 20% of patients develop resistance, with tumors recurring after several months of therapy.11 Adverse effects most commonly reported were muscle spasms (68%), alopecia (63%), taste distortion (51%), weight loss (46%), and fatigue (36%). Although these effects were considered mild or moderate, they tended to persist, and almost every patient developed at least one. In the nonrandomized trial,9 more than 25% of patients discontinued treatment because of adverse effects, and more than half of patients did the same in the basal cell nevus syndrome trial.10
New uses may reduce shortcomings
Studies are under way to determine how best to use vismodegib.
One possibility is to use the drug for a few months to shrink tumors to the point that they become eligible for surgery. This is especially important for high-risk tumors, such as those near the eye or other vital structures. In 11 patients, Ally et al12 found that the surgical defect area was reduced by 27% from baseline after 4 months of treatment with vismodegib, allowing for curative surgery in some.
Another option is to combine vismodegib with other agents—either new ones on the horizon or existing nonspecific medications. For example, the antifungal itraconazole has been shown to inhibit hedgehog signaling and perhaps could be combined with vismodegib to increase response and reduce resistance.
Finally, a topical or intralesional form of vismodegib would be useful not only to reduce systemic toxicity, but also to increase efficacy when combined with other topical or systemic medications.
TARGETED THERAPY FOR PSORIASIS VULGARIS
Case 2. A 28-year-old woman presents with worsening psoriasis. About 35% of her body surface is involved, including the palms and soles, making it difficult for her to perform activities of daily living (Figure 2). What are her options?
Psoriasis is a chronic immune-mediated disease that affects up to 3% of people worldwide. In its moderate to severe forms, we recognize psoriasis as a systemic inflammatory disease that may adversely affect organ systems other than the skin. Commonly associated comorbid diseases include inflammatory (psoriatic) arthritis, cardiovascular disease, malignancies (eg, lymphoma), and inflammatory bowel disease. In addition, patients are well known to have significantly impaired quality of life because of low self-esteem, stigmatization affecting their social and work relationships, and, in up to 60%, clinical depression.13,14 The onset of psoriatic arthritis, particularly of erosive disease, is an important decision point for starting aggressive treatment, as joint destruction is irreversible.
Early targeted therapy aimed at TNF alpha, IL-12, and IL-23
Histologically, psoriasis involves thickening of the epidermis caused by hyperproliferation of keratinocytes. Based on this, prior to the 1980s, the dominant hypothesis concerning its pathogenesis was that it was caused by an inherent defect of keratinocytes. In the 1980s and 1990s, however, molecular research revealed that psoriasis was an immune-mediated disease caused by immunologic dysregulation predominantly involving T-helper 1 (Th-1) cells, with the inflammatory cytokines tumor necrosis factor (TNF) alpha, interferon gamma, interleukin (IL) 12, and IL-23 playing prominent roles.15 These findings led to the development and FDA approval of the first effective, targeted, psoriasis treatments, TNF-alpha inhibitors and the IL-12/23 inhibitor ustekinumab.
Etanercept, the first TNF-alpha inhibitor to become available, was approved in 2004 for moderate to severe psoriasis. In 2008, the IL-12/23 inhibitor ustekinumab was approved for the same indication. These drugs are efficacious, are generally safe, and have revolutionized the treatment of psoriasis and psoriatic arthritis, and they are now prescribed on a daily basis.16,17
In the clinical trials that led to approval of these drugs, the main outcome measure was the Psoriasis Area and Severity Index (PASI), a clinical scoring tool that assesses clinical aspects of psoriatic disease including body surface area involvement, degree of thickness, erythema, and scaling of psoriatic plaques. PASI scores range from 0 (no psoriasis) to 72 (most severe psoriasis). Achieving “PASI 75” indicates at least 75% improvement from the baseline score and represents the most common primary outcome measure in clinical trials assessing efficacy of new treatments. Up to 80% of patients who received currently available TNF-alpha inhibitors and ustekinumab in pivotal clinical trials achieved PASI 75 when assessed at 12 to 16 weeks after starting treatment. A moderate percentage of patients (19%–57%, depending on the trial) achieved 90% improvement (PASI 90), and a minority (up to 18%) achieved PASI 100, indicating complete clearing of their psoriasis.18–22
Newly developed therapies target IL-17A
In the mid-2000s, Th-17 cells were discovered, a new lineage of T cells distinct from Th-1 and Th-2 cells. Th-17 cells are characterized by their production of IL-17, a pro-inflammatory cytokine with six family members (IL-17A through IL-17F). Over the next few years, experiments revealed that Th-17 cells and IL-17A play key roles in psoriasis immunologic dysregulation.15 These findings led to a paradigm shift in hypotheses concerning psoriasis pathogenesis, with Th-17 cells and IL-17 replacing Th-1 cells and associated cytokines as dominant mediators of tissue damage.
Additionally, these findings led to new ideas for treatment. Three monoclonal antibodies that target IL-17 inhibition are currently under investigation. Secukinumab and ixekizumab bind to IL-17A and inhibit it from downstream signaling, whereas brodalumab binds to the IL-17A receptor, blocking all six IL-17 cytokines (IL-17A to IL-17F).23
Clinical trials of IL-17 inhibitors show excellent skin improvement
Secukinumab. In 2014, the results of two phase 3 trials of secukinumab were published.24
In the Efficacy and Safety of Subcutaneous Secukinumab for Moderate to Severe Chronic Plaque-type Psoriasis for up to 1 Year trial,24 patients were given either secukinumab 300 mg or 150 mg subcutaneously at defined time points; 82% and 72%, respectively, attained PASI 75 at 12 weeks.
Similar results were seen in the Safety and Efficacy of Secukinumab Compared to Etanercept in Subjects With Moderate to Severe, Chronic Plaque-Type Psoriasis study,24 in which PASI 75 was achieved by 77% of patients receiving secukinumab 300 mg, 67% of those receiving secukinumab 150 mg, and only 44% of those receiving etanercept 50 mg twice weekly at 12 weeks. Rates of infection with secukinumab and etanercept were similar.
The most striking results of these trials were that more than half of patients receiving the 300-mg dose achieved at least 90% improvement in their PASI score (PASI 90) by week 12, and in more than a quarter of patients the psoriasis completely cleared (PASI 100). These results were dramatically better than for etanercept (PASI 90 21%; PASI 100 4%).
Additionally, secukinumab worked fast. The median time to PASI 50 with secukinumab 300 mg was less than half that seen with etanercept (3 weeks vs 7 weeks).
Ixekizumab. In 2012, a phase 2 trial evaluated subcutaneous injections of ixekizumab in dosages ranging from 10 to 150 mg at defined intervals for 16 weeks.25 Of those receiving the highest dosage, 82% attained PASI 75 at 12 weeks, on par with what is noted in patients receiving TNF-alpha inhibitors and IL-12/23 inhibitors. Remarkably, however, almost three-quarters of patients (71%) achieved PASI 90, and 39% achieved PASI 100. Improvement in psoriasis was apparent as early as 1 week after injection.
Brodalumab. A 2012 phase 2 trial of various dosages of the IL-17 receptor inhibitor brodalumab26 also showed excellent PASI 75 achievement with the highest dosage (82%). Astonishingly, though, PASI 90 was achieved by 75% of patients, and PASI 100 by 62%.
Overall, although the percentages of patients achieving PASI 75 with the new IL-17 inhibitor drugs are comparable to those seen with TNF-alpha inhibitors and IL-12/23 inhibitors, the extraordinarily high percentages of patients who achieved PASI 90 and PASI 100 are unprecedented.18–22
Arthritis improvement not shown
Where the IL-17 inhibitors eventually settle within algorithms of psoriasis treatment largely depends on their efficacy in treating psoriatic arthritis compared with TNF-alpha inhibitors and IL-12/23 inhibitors. Joint inflammation is typically evaluated with the American College of Rheumatology (ACR) scoring tool, which in simple terms can be thought of as analogous to the PASI scoring tool for the skin. Although the ACR scoring tool was developed to assess joint inflammation in clinical trials for patients with rheumatoid arthritis, it is commonly used to assess improvement of psoriatic arthritis in clinical trials. The ACR tool involves assessing and scoring the number of swollen and tender joints, but also incorporates serologic assessment of acute-phase reactants (erythrocyte sedimentation rate or C-reactive protein level), patient and physician global assessment, pain, and function. ACR 20 implies roughly a 20% improvement in these criteria, whereas ACR 50 indicates 50% improvement, and so on.
Two phase 2 trials of IL-17 inhibitors for psoriatic arthritis have been published, one with secukinumab27 and one with brodalumab.28 Neither had impressive improvement in the ACR score vs TNF inhibitors—39% for ACR 20 at week 12 and less than 10% for ACR 70. Clinical trial design may have played a role, and phase 3 trials are under way for all three IL-17 inhibitors.
Adverse effects of IL-17 inhibitors
For the most part, adverse effects reported with the IL-17 inhibitors have been mild and similar to those reported with available biologic treatments for psoriasis. Adverse effects most commonly reported have been nasopharyngitis, upper respiratory infection, arthralgia, and mild injection-site reactions. In the future, attention will be paid to the rate of infections known to be associated with IL-17, mainly localized infections with Staphylococcus aureus and Candida species. Some patients have developed Candida esophagitis, but this appears to resolve with discontinuation of the drugs. Neutropenia has occurred, but very few patients have developed grade 3 (500–1,000 cells/mm3) or worse. All adverse effects were reversible with discontinuation of treatment.
Approval of secukinumab, and current studies of IL-17 inhibitors
On January 21, 2015, secukinumab was approved by the FDA for treatment of moderate to severe psoriasis vulgaris in adult patients and is now available by prescription.
More trials of IL-17 inhibitors for the treatment of psoriasis and psoriatic arthritis are under way and are at various phases at the time of this writing.23
TARGETED THERAPY FOR ADVANCED MELANOMA
Case 3. A 58-year-old man presents with an irregular pigmented lesion on his back. Biopsy shows malignant melanoma with an intense, chronic inflammatory infiltrate surrounding the tumor (Figure 3). The tumor was surgically excised with standard margins. Two years later, the patient developed multiple pigmented lesions on the face and complained of headache. Magnetic resonance imaging of the brain revealed multiple enhancing lesions consistent with metastatic melanoma (Figure 3). What are this patient’s options?
Melanoma is the fifth most common cancer in humans, with about 132,000 new cases diagnosed worldwide each year and 48,000 deaths from advanced disease. Its incidence has risen rapidly over the last few decades. Advanced disease has a poor prognosis, with the median overall survival less than 1 year and 5-year survival less than 10%.
Despite decades of research, a paucity of FDA-approved medications were available to treat advanced melanoma until recently. The alkylating agent dacarbazine was approved in 1975, interferon alpha in 1995, and high-dose IL-2 in 1998. Although some patients respond, studies have not shown significant improvement in survival with any of these medications.29–31
In 2002, Davies et al32 found that 50% to 65% of metastatic cutaneous melanomas have a mutation in the BRAF gene. Interestingly, 80% of these patients share a single specific mutation: substitution of glutamic acid for valine in codon 600 (BRAF V600E). The second most common mutation is a single substitution of a lysine for that same valine (BRAF V600K). Additionally, NRAS is mutated in about 20% of melanomas. These discoveries implicated a mitogen-activated protein kinase (MAPK) pathway (Figure 4) as playing a critical role in metastatic melanoma for a large percentage of patients.29
Based on this knowledge, several targeted therapies for melanoma have been developed, and some have been approved.
BRAF inhibitors—first success against melanoma
Vemurafenib. In 2010, Flaherty et al33 reported on a phase 1 and phase 2 clinical trial of vemurafenib (960 mg orally twice daily), a potent inhibitor of BRAF with the V600E mutation. They demonstrated a clinical benefit in 80% of patients with stage IV BRAF-mutant melanoma, an unprecedented response that opened the door to changes in the treatment of metastatic melanoma.
The phase 3 BRAF Inhibitor in Melanoma (BRIM)-3 clinical trial,34 published in 2011, randomized 675 previously untreated patients with advanced melanoma to either vemurafenib 960 mg orally twice daily or dacarbazine, the standard of care. The trial was terminated early when an interim analysis showed a significant overall advantage for vemurafenib (median progression-free survival 5.3 months vs 1.6 months for dacarbazine). Based on these results, vemurafenib was FDA-approved in August 2011 for use in patients with BRAF-mutant melanoma.
Dabrafenib. In a phase 3 clinical trial in 2012, Hauschild et al35 randomized 250 patients with BRAF (V600E)-mutated melanoma in a 3:1 ratio to receive either dabrafenib, a more potent second-generation BRAF inhibitor, or dacarbazine. Half of patients responded to dabrafenib, with a significantly improved progression-free survival rate (5.1 vs 2.7 months respectively), leading to FDA approval for its use in BRAF-mutant melanoma in May 2013.
Adverse effects common to vemurafenib and dabrafenib include rash, fatigue, fever, and joint pain. In addition, up to 25% of patients develop multiple secondary cutaneous squamous cell carcinomas and keratoacanthomas, usually within the first few months of therapy, which are believed to be caused by paradoxical activation of the MAPK pathway.
A more important problem with these medications is the development of resistance. Tumors typically progress again after a median progression-free survival of 6 to 7 months.
MEK inhibitors—another line of defense
Inhibitors of MEK—a serine-threonine kinase that is part of the same MAPK pathway involving BRAF—have been developed as well.
Trametinib. In 2012, trametinib, an allosteric MEK inhibitor, was used in an open-label phase 3 trial in 322 patients with advanced melanoma. Progression-free survival was 4.8 months for trametinib-treated patients compared with 1.5 months for the standard chemotherapy group (dacarbazine or paclitaxel).36 These results led to FDA approval of trametinib in May 2013 for treating BRAF-mutant melanoma.29
Cobimetinib is a second MEK inhibitor being evaluated alone and in combination with other targeted treatments for advanced melanoma.
Both MEK inhibitors have adverse effects similar to those seen with the BRAF inhibitors, including diarrhea, rash, fatigue, and edema. They also tend to cause asymptomatic elevated creatine kinase and transient retinopathy, reduced ejection fraction, and ventricular dysfunction. Unlike BRAF inhibitors, they are not associated with development of secondary cutaneous squamous cell carcinomas or keratoacanthomas. However, as with BRAF inhibitors, resistance is a problem with MEK inhibitors, with most patients relapsing less than a year after starting therapy.
Combination therapy improves outcomes
Possible mechanisms underlying resistance to these medications are being studied. A number of important factors appear to drive resistance, including expression of truncated BRAF proteins that do not bind the BRAF inhibitors and still activate downstream signaling, and amplification of BRAF to such a degree that it overwhelms the medications. This has led to the idea of combining BRAF inhibitors and MEK inhibitors to block the MAPK pathway at two points, potentially increasing the response and decreasing resistance.
Two trials have evaluated combinations of BRAF and MEK inhibitors in patients with advanced melanoma. Larkin et al37 conducted a phase 3 study evaluating combined vemurafenib (a BRAF inhibitor) and cobimetinib (a MEK inhibitor) vs combined vemurafenib and placebo. Survival with the combination therapy was 9.9 months vs 6.2 months with the single treatment.
The incidence of serious adverse effects was not significantly increased with the combination therapy, and keratoacanthomas, cutaneous squamous cell carcinomas, alopecia, and arthralgias were reduced compared with the vemurafenib and placebo group.
Another trial38 evaluating combined dabrafenib (a BRAF inhibitor) and trametinib (a MEK inhibitor) vs combined dabrafenib and placebo had similar findings: increased survival in the combined therapy group (9.3 months vs 8.8 months) and lower rates of squamous cell carcinoma (2% vs 9%).
In January 2014, the FDA approved the combination of BRAF and MEK inhibitors for the treatment of BRAF-mutant metastatic melanoma based on improved survival and generally reduced adverse effects.
IMMUNOTHERAPIES FOR NON-BRAF MELANOMA
Although BRAF and MEK inhibitors represent tremendous advances, their use is limited to the approximately 50% to 65% of patients with advanced melanoma who have BRAF V600 mutations. For others, only the traditional standard medications have been available until recently.
Two of those standard FDA-approved medications, interferon alpha-2b and IL-2, represent immunotherapies. Interferon alpha-2b up-regulates antigen presentation and increases antigen recognition by T cells. Overall, about 20% of patients in clinical trials have achieved responses.
IL-2 is a cytokine that increases T-cell proliferation and maturation into effector T cells. High-dose IL-2 has produced responses in 15% of patients, with a durable complete response in a small proportion.
Though success with these medications was modest, the fact that some patients responded to them indicates that immunotherapy could be a viable strategy for treating metastatic melanoma.30 This is underscored by the fact that some patients can mount an adaptive immune response specifically directed against antigenic proteins expressed in their tumors, resulting in expansion of cytotoxic T cells and control or even elimination of the malignancy.30
Tumors manipulate host immune checkpoints
Molecular biology has provided tremendous insight into tumor immunology over the past several decades, and we now recognize that a hallmark of cancer is escape from immune control.
Cancer cells contain a multitude of mutations that produce proteins that should be recognized by the immune system as foreign but in most individuals are not. This is because T-cell activity is down-regulated in cancer due to cancer cells’ ability to manipulate the host’s normal immunologic inhibitory pathways critical for maintaining self-tolerance.
In general, T-cell activation is initiated when an antigen-presenting cell presents an antigen to a T cell in a major histocompatibility complex-restricted manner. To prevent T cells from being activated by self-antigens and initiating autoimmunity, the interaction between antigen-presenting cells and T cells is regulated by checkpoints (Figure 5). First, for an antigen-presenting cell/T-cell interaction to result in T-cell activation, the T-cell receptor CD28 must bind CD80 on the antigen-presenting cell to drive a “positive” signal. Early in the interaction, the T-cell receptor CTLA-4 is up-regulated and competes with CD28 for binding of CD80. If CTLA-4, and not CD28, binds CD80, a “negative” signal is sent to the T cell and down-regulates it, making the interaction unproductive. Importantly, it is the CTLA-4:CD80 interaction that appears to be crucial for the ability of tumors to dampen T-cell responses to cancer cells.
Ipilimumab is a fully humanized monoclonal antibody that binds to CTLA-4, blocking its ability to bind to CD80 and thereby enhancing T-cell activation. In a phase 3 trial, Hodi et al39 evaluated its use in treating advanced melanoma, with some enrolled patients having failed IL-2 treatment. Patients receiving ipilimumab with or without a glycoprotein-100 peptide vaccine (gp100) had an overall survival benefit of 10.1 months compared with 6.4 months for patients treated with gp100 alone. At 24 months, the survival rate with ipilimumab alone was 23.5%, almost double that of patients receiving gp100 alone.
Ipilimumab received FDA approval for treatment of metastatic melanoma in March 2011. This, and the BRAF inhibitors, were the first drugs approved by the FDA for the treatment of advanced melanoma in more than a decade.
Common adverse effects of ipilimumab include fatigue, diarrhea, rash, and pruritus. As expected, given its mechanism of action, up to about 25% of patients experience severe autoimmune-related events that may variably manifest as colitis, rash, hepatitis, neuritis, hypothyroidism, hypopituitarism, and hypophysitis. Another problem with this medication is that a subset of patients do not respond.
Cancer cells disguised as normal cells
Cancer cells can also manipulate another immunologic checkpoint to evade attack by the host immune system (Figure 5). Cytotoxic T cells may recognize antigens on tumor cells and become activated and primed to directly destroy them. However, tumor cells, like normal cells express the programmed death ligands RTK-L1 and PD-L2. These ligands function to bind to the PD-1 receptor on activated T cells to indicate they are “self” and inhibit the cytotoxic T cells from destroying them.
Evasion of immune system attack by manipulating checkpoints involving CTLA-4 and PD-1 helps explain why malignancies can seemingly be associated with brisk inflammatory responses, such as the tumor in Case 3, yet progress and eventually metastasize (Figure 3).
Two medications—nivolumab and pembrolizumab—have been developed in an attempt to disrupt the ability of tumor cells to trick the immune system into accepting them as “self” by manipulating the PD-L1/PD-L2: PD-1 interaction. Both drugs are monoclonal antibodies that bind to PD-1 and, thus, effectively block the ability of PD-L1 or PD-L2 on tumor cells to bind these ligands and signal to activated T cells that they are “self.” This blocking allows T cells to then carry out their killing of tumor cells they initially recognize as foreign.
Nivolumab. In 2014, a phase 3 trial40 compared nivolumab and dacarbazine in patients with untreated advanced melanoma without a BRAF mutation. Objective response rates were 40.0% in the nivolumab group vs 13.9% in the dacarbazine group. This trial was stopped early because of significantly better survival rates in patients taking nivolumab compared with standard chemotherapy.
Interestingly, only 35% of patients who responded to nivolumab had evidence of PD-L1 expression on the surface of their tumor cells as assessed by immunohistochemical assay. Regardless of PD-L1 status, nivolumab-treated patients had improved overall survival compared with those treated with dacarbazine. The response rate with nivolumab was only slightly better in the subgroup of patients whose tumors expressed PD-L1 than in the subgroup without PD-L1.
On December 22, 2014, the FDA granted accelerated approval to nivolumab for the treatment of patients with unresectable or metastatic melanoma and disease progression following ipilimumab treatment and, if BRAF V600 mutation-positive, a BRAF inhibitor.
Pembrolizumab. Also in 2014, an open-label, randomized, phase 1b trial of pembrolizumab treatment at two different dosage schedules was conducted in patients with advanced melanoma that had become refractory either to ipilimumab or a BRAF inhibitor.41 Treatment with pembrolizumab had an objective response rate of 26% at both doses.
In September 2014, the FDA granted accelerated approval for the use of pembrolizumab to treat patients with unresectable or metastatic melanoma and disease progression following treatment with ipilimumab or a BRAF inhibitor.
Adverse effects of PD-1 inhibitors are similar to those seen with ipilimumab, the most common (occurring in at least 20%) being fatigue, cough, nausea, pruritus, rash, decreased appetite, constipation, muscle pain, and diarrhea. Serious effects from pembrolizumab (occurring in at least 2%) were kidney failure, dyspnea, pneumonia, and cellulitis. As seen with ipilimumab, clinically significant autoimmune adverse reactions occur with PD-1 inhibitors, including pneumonitis, colitis, hypophysitis, nephritis, and hepatitis.
Combination therapy under investigation
A phase 1 trial using combination therapy with both immune checkpoint inhibitors—nivolumab (anti-PD-1) and ipilimumab (anti-CTLA-4)—in patients with treatment-resistant metastatic melanoma was published in 2013.42 More than half of patients achieved objective responses, with tumor regression of at least 80% in those who had a response. Tumor response was evident in all subgroups of patients studied—those with pretreatment elevated lactate dehydrogenase levels (one of the strongest prognostic factors in metastatic melanoma), metastases to distant sites, and bulky, multifocal tumor burden. Based on these results, a phase 3 trial is now under way looking at the combination of these two medications vs either one alone.
In summary, targeted treatments are changing the paradigm of how common dermatologic conditions associated with significant morbidity and mortality are treated. Although implementation of the above treatments into everyday clinical practice is exciting, future studies surrounding each are needed to address unanswered issues, such as the optimal dosing and treatment schedules in terms of both disease response and inhibition of resistance, optimal patient/disease characteristics for use, and optimal drug treatment combinations. In the meantime, basic research still utilizing classic molecular biology techniques to uncover pathogenic disease mechanisms in even more detail is ongoing and hopefully will lead to development of even better targeted treatments or even cures for these diseases.
- Lyons TG, O’Kane GM, Kelly CM. Efficacy and safety of vismodegib: a new therapeutic agent in the treatment of basal cell carcinoma. Expert Opin Drug Saf 2014; 13:1125–1132.
- McCusker M, Basset-Sequin N, Dummer R, et al. Metastatic basal cell carcinoma: prognosis dependent on anatomic site and spread of disease. Eur J Cancer 2014; 50:774–783.
- Hahn H, Wicking C, Zaphiropoulous PG, et al. Mutations of the human homolog of Drosophila patched in the nevoid basal cell carcinoma syndrome. Cell 1996; 85:841–851.
- Aszterbaum M, Rothman A, Johnson RL, et al. Identification of mutations in the human PATCHED gene in sporadic basal cell carcinomas and in patients with the basal cell nevus syndrome. J Invest Dermatol 1998; 110:885–888.
- Ingham PW, Placzek M. Orchestrating ontogenesis: variations on a theme by sonic hedgehog. Nat Rev Genet 2006; 7:841–850.
- Robarge KD, Brunton SA, Castanedo GM, et al. GDC-0449-a potent inhibitor of the hedgehog pathway. Bioorg Med Chem Lett 2009; 19:5576–5581.
- Proctor AE, Thompson LA, O’Bryant CL. Vismodegib: an inhibitor of the Hedgehog signaling pathway in the treatment of basal cell carcinoma. Ann Pharmacother 2014; 48:99–106.
- Dessinioti C, Plaka M, Stratigos AJ. Vismodegib for the treatment of basal cell carcinoma: results and implications of the ERIVANCE BCC trial. Future Oncol 2014; 10:927–936.
- Sekulic A, Migden MR, Oro AE, et al. Efficacy and safety of vismodegib in advanced basal-cell carcinoma. N Engl J Med 2012; 366:2171–2179.
- Tang JY, Mackay-Wiggan JM, Aszterbaum M, et al. Inhibiting the hedgehog pathway in patients with basal-cell nevus syndrome. N Engl J Med 2012; 366:2180–2188.
- Brinkhuizen T, Reinders MG, van Geel M, et al. Acquired resistance to the Hedgehog pathway inhibitor vismodegib due to smoothened mutations in treatment of locally advanced basal cell carcinoma. J Am Acad Dermatol 2014; 71:1005–1008.
- Ally MS, Aasi S, Wysong A, et al. An investigator-initiated open-label clinical trial of vismodegib as a neoadjuvant to surgery for high-risk basal cell carcinoma. J Am Acad Dermatol 2014; 71:904–911.
- Rapp SR, Feldman SR, Exum ML, Fleischer AB Jr, Reboussin DM. Psoriasis causes as much disability as other major medical diseases. J Am Acad Dermatol 1999; 41:401–407.
- Gelfand JM, Niemann AL, Shin DB, Wang X, Margolis DJ, Troxel AB. Risk of myocardial infarction in patients with psoriasis. JAMA 2006; 296:1735–1741.
- Lynde CW, Poulin Y, Vender R, Bourcier M, Khalil S. Interleukin 17A: toward a new understanding of psoriasis pathogenesis. J Am Acad Dermatol 2014; 71:141–150.
- Tracey D, Klareskog L, Sasso EH, Salfeld JG, Tak PP. Tumor necrosis factor antagonist mechanisms of action: a comprehensive review. Pharmacol Ther 2008; 117:244–279.
- Nestle FL, Kaplan DH, Barker J. Psoriasis. N Engl J Med 2009; 361:496–509.
- Mentor A, Tyring SK, Gordon K, et al. Adalimumab therapy for moderate to severe psoriasis: a randomized, controlled phase III trial. J Am Acad Dermatol 2007; 58:106–115.
- Leonardi CL, Powers JL, Matheson RT, et al. Etanercept as monotherapy in patients with psoriasis. N Engl J Med 2003; 349:2014–2022.
- Reich K, Nestle FO, Papp K, et al. Infliximab induction and maintenance therapy for moderate-to-severe psoriasis: a phase III, multicentre, double-blind trial. Lancet 2005; 366:1367–1374.
- Leonardi CL, Kimball AB, Papp KA, et al. Efficacy and safety of ustekinumab, a human interleukin-12/23 monoclonal antibody, in patients with psoriasis: 76-week results from a randomised, double-blind, placebo-controlled trial (PHOENIX 1). Lancet 2008; 371:1665–1674.
- Papp KA, Langley RG, Lebwohl M, et al; PHOENIX 2 study investigators. Efficacy and safety of ustekinumab, a human interleukin-12/23 monoclonal antibody, in patients with psoriasis: 52-week results from a randomised, double-blind, placebo-controlled trial (PHOENIX 2). Lancet 2008; 371:1675–1684.
- Leonardi CL, Gordon KB. New and emerging therapies in psoriasis. Semin Cut Med Surg 2014; 33(suppl 2):S37–S41.
- Langley RG, Elewski BE, Lebwohl, et al for the ERASURE and FIXTURE Study Groups. Secukinumab in plaque psorisis—results of two phase 3 trials. N Engl J Med 2014; 371:326–338.
- Leonardi C, Matheson R, Zachariae C. Anti-interleukin-17 monoclonal antibody ixekizumab in chronic plaque psoriasis. N Engl J Med 2012; 366:1190–1199.
- Papp KA, Leonardi C, Menter A, et al. Brodalumab, an anti-interleukin-17-receptor antibody for psoriasis. N Engl J Med 2012; 366:1181–1189.
- McInnes IB, Sieper J, Braun J, et al. Efficacy and safety of secukinumab, a fully human anti-interleukin-17A monoclonal antibody, in patients with moderate-to-severe psoriatic arthritis: a 24-week, randomised, double-blind, placebo-controlled, phase II proof-of-concept trial. Ann Rheum Dis 2014; 73:349–356.
- Mease PJ, Genovese MC, Greenwald MW, et al. Brodalumab, an anti-IL17RA monoclonal antibody, in psoriatic arthritis. N Engl J Med 2014; 370:2295–2306.
- Girotti MR, Saturno G, Lorigan P, Marais R. No longer an untreatable disease: how targeted and immunotherapies have changed the management of melanoma patients. Molec Oncol 2014, 8:1140–1158.
- Saranga-Perry V, Ambe C, Zager JS, Kudchadkar RR. Recent developments in the medical and surgical treatment of melanoma. CA Canc J Clin 2014; 64:171–185.
- Shah DJ, Dronca RS. Latest advances in chemotherapeutic, targeted, and immune approaches in the treatment of metastatic melanoma. Mayo Clin Proc 2014; 89:504–519.
- Davies H, Ignell GR, Cox C, et al. Mutations of the BRAF gene in human cancer. Nature 2002; 417:949–954.
- Flaherty KT, Puzanov I, Kim KB, et al. Inhibition of mutated, activated BRAF in metastatic melanoma. N Engl J Med 2010; 363:809–819.
- Chapman PB, Hauschild A, Robert C. Improved survival with vemurafenib in melanoma with BRAF V600E mutation. N Engl J Med 2011; 364:2507–2516.
- Hauschild A, Grob JJ, Demidov LV, et al. Dabrafenib in BRAF-mutated metastatic melanoma: a multicentre, open-label, phase 3 randomised controlled trial. Lancet 2012; 380:358–365.
- Flaherty KT, Robert C, Hersey P, et al. Improved survival with MEK inhibition in BRAF-mutated melanoma. N Engl J Med 2012; 367:107–114.
- Larkin J, Ascierto PA, Dreno B, et al. Combined vemurafenib and cobimetinib in BRAF-mutated melanoma. N Engl J Med 2014; 371:1867–1876.
- Long GV, Stroyakovskiy D, Gogas H, et al. Combined BRAF and MEK inhibition versus BRAF inhibition alone in melanoma. N Eng J Med 2014; 371:1877–1888.
- Hodi FS, O’Day SJ, McDermott DF, Weber RW. Improved survival with ipilimumab in patients with metastatic melanoma. N Engl J Med 2010; 363:711–723.
- Robert C, Long GV, Brady B, et al. Nivolumab in previously untreated melanoma without BRAF mutation. N Engl J Med 2015; 372:320–330.
- Robert C, Ribas A, Wolchok JD, et al. Anti-programmed-death-receptor-1 treatment with pembrolizumab in ipilimumab-refractory advanced melanoma: a randomised dose-comparison cohort of a phase 1 trial. Lancet 2014; 384:1109–1117.
- Wolchok JD, Kluger H, Callahan MK, et al. Nivolumab plus ipilimumab in advanced melanoma. N Engl J Med 2013; 369:122–133.
New targeted therapies are changing the way patients with advanced dermatologic diseases are treated. Innovative molecular biology techniques developed as far back as the 1970s have engendered tremendous insight into the cellular and molecular pathogenesis of numerous diseases. Novel medications based on these insights are now bearing fruit, as directed biologic therapies that are revolutionizing clinical practice are increasingly becoming available.
This article reviews advances in targeted therapies for advanced basal cell carcinoma, psoriasis, and metastatic melanoma.
TARGETED THERAPY FOR BASAL CELL CARCINOMA
Case 1. A 56-year-old man presents with a progressively enlarging leg ulcer. Although it has been treated empirically for years as a venous stasis ulcer, biopsy reveals that it is basal cell carcinoma. Imaging shows muscle and tendon invasion, making surgical intervention short of amputation challenging (Figure 1). What are his options?
Basal cell carcinoma is the most common cancer in humans, accounting for 25% of all cancers and more than 2 million cases in the United States every year. In most cases, surgical excision is curative, but a subset of patients have inoperable, locally advanced, or metastatic disease that drastically limits treatment options. The median survival in metastatic basal cell carcinoma is 24 months, and conventional chemotherapy has not been shown to improve the prognosis.1,2
In addition to the burden of sporadic basal cell carcinoma, patients with the rare autosomal-dominant genetic disorder basal cell nevus syndrome (Gorlin syndrome) develop multiple basal cell lesions over their lifetime. The syndrome may also involve abnormalities of the skeletal system, genitourinary tract, and central nervous system, including development of medulloblastoma.
In Gorlin syndrome, basal cell carcinomas occur often and early; about half of white patients with the syndrome develop their first lesions by age 21, and 90% by age 35. The lesions occur in multiple numbers and can develop anywhere on the body, including on non–sun-exposed areas. Patients who have Gorlin syndrome need meticulous monitoring every 2 to 3 months so that basal cell lesions can be recognized early and treated before they become locally advanced. Keeping up with the numerous medical appointments and invasive treatments can be physically and mentally taxing for patients.
Specific pathway and mutations identified
In 1996, Gorlin syndrome was found to be caused by mutations of the human homolog of the PATCHED gene, which codes for a receptor in the “hedgehog” pathway.3 Two years later, the same mutations were found to be involved in many sporadic basal cell carcinomas, and we now believe that at least 85% of basal cell carcinomas involve abnormal activation of hedgehog pathway signaling.4,5
Vismodegib developed as targeted therapy
In 2009, Robarge et al6 described a potent inhibitor of the hedgehog pathway that was later optimized for potency and desirable pharmacologic traits, resulting in the drug vismodegib.7,8
Two phase 2 multicenter clinical trials9,10 of vismodegib were published in 2012. In the first, which was not randomized,9 33 patients with metastatic basal cell carcinoma and 63 patients with locally advanced disease were treated with vismodegib. Of those with metastatic disease, 30% achieved an objective response. Of those with locally advanced disease, 43% achieved an objective response and 21% achieved a complete response.
In the second trial,10 patients with Gorlin syndrome were randomized to either vismodegib (26 patients) or placebo (16 patients). After 8 months, the vismodegib group had developed significantly fewer new surgically eligible tumors (2 vs 29 per year), their tumors were smaller (change from baseline of the sum of the longest diameters –65% vs –11%), and they needed fewer surgeries (mean 0.31 vs 4.4 per patient). No tumors progressed in the treatment group. Results in some patients were dramatic, with complete healing of large ulcerative tumors. The trial was ended early in view of significant efficacy in the treatment group.
Based on these trials, the US Food and Drug Administration (FDA) approved vismodegib for treating metastatic and locally advanced basal cell carcinoma.
Resistance and adverse effects common
Unfortunately, vismodegib has significant drawbacks. About 20% of patients develop resistance, with tumors recurring after several months of therapy.11 Adverse effects most commonly reported were muscle spasms (68%), alopecia (63%), taste distortion (51%), weight loss (46%), and fatigue (36%). Although these effects were considered mild or moderate, they tended to persist, and almost every patient developed at least one. In the nonrandomized trial,9 more than 25% of patients discontinued treatment because of adverse effects, and more than half of patients did the same in the basal cell nevus syndrome trial.10
New uses may reduce shortcomings
Studies are under way to determine how best to use vismodegib.
One possibility is to use the drug for a few months to shrink tumors to the point that they become eligible for surgery. This is especially important for high-risk tumors, such as those near the eye or other vital structures. In 11 patients, Ally et al12 found that the surgical defect area was reduced by 27% from baseline after 4 months of treatment with vismodegib, allowing for curative surgery in some.
Another option is to combine vismodegib with other agents—either new ones on the horizon or existing nonspecific medications. For example, the antifungal itraconazole has been shown to inhibit hedgehog signaling and perhaps could be combined with vismodegib to increase response and reduce resistance.
Finally, a topical or intralesional form of vismodegib would be useful not only to reduce systemic toxicity, but also to increase efficacy when combined with other topical or systemic medications.
TARGETED THERAPY FOR PSORIASIS VULGARIS
Case 2. A 28-year-old woman presents with worsening psoriasis. About 35% of her body surface is involved, including the palms and soles, making it difficult for her to perform activities of daily living (Figure 2). What are her options?
Psoriasis is a chronic immune-mediated disease that affects up to 3% of people worldwide. In its moderate to severe forms, we recognize psoriasis as a systemic inflammatory disease that may adversely affect organ systems other than the skin. Commonly associated comorbid diseases include inflammatory (psoriatic) arthritis, cardiovascular disease, malignancies (eg, lymphoma), and inflammatory bowel disease. In addition, patients are well known to have significantly impaired quality of life because of low self-esteem, stigmatization affecting their social and work relationships, and, in up to 60%, clinical depression.13,14 The onset of psoriatic arthritis, particularly of erosive disease, is an important decision point for starting aggressive treatment, as joint destruction is irreversible.
Early targeted therapy aimed at TNF alpha, IL-12, and IL-23
Histologically, psoriasis involves thickening of the epidermis caused by hyperproliferation of keratinocytes. Based on this, prior to the 1980s, the dominant hypothesis concerning its pathogenesis was that it was caused by an inherent defect of keratinocytes. In the 1980s and 1990s, however, molecular research revealed that psoriasis was an immune-mediated disease caused by immunologic dysregulation predominantly involving T-helper 1 (Th-1) cells, with the inflammatory cytokines tumor necrosis factor (TNF) alpha, interferon gamma, interleukin (IL) 12, and IL-23 playing prominent roles.15 These findings led to the development and FDA approval of the first effective, targeted, psoriasis treatments, TNF-alpha inhibitors and the IL-12/23 inhibitor ustekinumab.
Etanercept, the first TNF-alpha inhibitor to become available, was approved in 2004 for moderate to severe psoriasis. In 2008, the IL-12/23 inhibitor ustekinumab was approved for the same indication. These drugs are efficacious, are generally safe, and have revolutionized the treatment of psoriasis and psoriatic arthritis, and they are now prescribed on a daily basis.16,17
In the clinical trials that led to approval of these drugs, the main outcome measure was the Psoriasis Area and Severity Index (PASI), a clinical scoring tool that assesses clinical aspects of psoriatic disease including body surface area involvement, degree of thickness, erythema, and scaling of psoriatic plaques. PASI scores range from 0 (no psoriasis) to 72 (most severe psoriasis). Achieving “PASI 75” indicates at least 75% improvement from the baseline score and represents the most common primary outcome measure in clinical trials assessing efficacy of new treatments. Up to 80% of patients who received currently available TNF-alpha inhibitors and ustekinumab in pivotal clinical trials achieved PASI 75 when assessed at 12 to 16 weeks after starting treatment. A moderate percentage of patients (19%–57%, depending on the trial) achieved 90% improvement (PASI 90), and a minority (up to 18%) achieved PASI 100, indicating complete clearing of their psoriasis.18–22
Newly developed therapies target IL-17A
In the mid-2000s, Th-17 cells were discovered, a new lineage of T cells distinct from Th-1 and Th-2 cells. Th-17 cells are characterized by their production of IL-17, a pro-inflammatory cytokine with six family members (IL-17A through IL-17F). Over the next few years, experiments revealed that Th-17 cells and IL-17A play key roles in psoriasis immunologic dysregulation.15 These findings led to a paradigm shift in hypotheses concerning psoriasis pathogenesis, with Th-17 cells and IL-17 replacing Th-1 cells and associated cytokines as dominant mediators of tissue damage.
Additionally, these findings led to new ideas for treatment. Three monoclonal antibodies that target IL-17 inhibition are currently under investigation. Secukinumab and ixekizumab bind to IL-17A and inhibit it from downstream signaling, whereas brodalumab binds to the IL-17A receptor, blocking all six IL-17 cytokines (IL-17A to IL-17F).23
Clinical trials of IL-17 inhibitors show excellent skin improvement
Secukinumab. In 2014, the results of two phase 3 trials of secukinumab were published.24
In the Efficacy and Safety of Subcutaneous Secukinumab for Moderate to Severe Chronic Plaque-type Psoriasis for up to 1 Year trial,24 patients were given either secukinumab 300 mg or 150 mg subcutaneously at defined time points; 82% and 72%, respectively, attained PASI 75 at 12 weeks.
Similar results were seen in the Safety and Efficacy of Secukinumab Compared to Etanercept in Subjects With Moderate to Severe, Chronic Plaque-Type Psoriasis study,24 in which PASI 75 was achieved by 77% of patients receiving secukinumab 300 mg, 67% of those receiving secukinumab 150 mg, and only 44% of those receiving etanercept 50 mg twice weekly at 12 weeks. Rates of infection with secukinumab and etanercept were similar.
The most striking results of these trials were that more than half of patients receiving the 300-mg dose achieved at least 90% improvement in their PASI score (PASI 90) by week 12, and in more than a quarter of patients the psoriasis completely cleared (PASI 100). These results were dramatically better than for etanercept (PASI 90 21%; PASI 100 4%).
Additionally, secukinumab worked fast. The median time to PASI 50 with secukinumab 300 mg was less than half that seen with etanercept (3 weeks vs 7 weeks).
Ixekizumab. In 2012, a phase 2 trial evaluated subcutaneous injections of ixekizumab in dosages ranging from 10 to 150 mg at defined intervals for 16 weeks.25 Of those receiving the highest dosage, 82% attained PASI 75 at 12 weeks, on par with what is noted in patients receiving TNF-alpha inhibitors and IL-12/23 inhibitors. Remarkably, however, almost three-quarters of patients (71%) achieved PASI 90, and 39% achieved PASI 100. Improvement in psoriasis was apparent as early as 1 week after injection.
Brodalumab. A 2012 phase 2 trial of various dosages of the IL-17 receptor inhibitor brodalumab26 also showed excellent PASI 75 achievement with the highest dosage (82%). Astonishingly, though, PASI 90 was achieved by 75% of patients, and PASI 100 by 62%.
Overall, although the percentages of patients achieving PASI 75 with the new IL-17 inhibitor drugs are comparable to those seen with TNF-alpha inhibitors and IL-12/23 inhibitors, the extraordinarily high percentages of patients who achieved PASI 90 and PASI 100 are unprecedented.18–22
Arthritis improvement not shown
Where the IL-17 inhibitors eventually settle within algorithms of psoriasis treatment largely depends on their efficacy in treating psoriatic arthritis compared with TNF-alpha inhibitors and IL-12/23 inhibitors. Joint inflammation is typically evaluated with the American College of Rheumatology (ACR) scoring tool, which in simple terms can be thought of as analogous to the PASI scoring tool for the skin. Although the ACR scoring tool was developed to assess joint inflammation in clinical trials for patients with rheumatoid arthritis, it is commonly used to assess improvement of psoriatic arthritis in clinical trials. The ACR tool involves assessing and scoring the number of swollen and tender joints, but also incorporates serologic assessment of acute-phase reactants (erythrocyte sedimentation rate or C-reactive protein level), patient and physician global assessment, pain, and function. ACR 20 implies roughly a 20% improvement in these criteria, whereas ACR 50 indicates 50% improvement, and so on.
Two phase 2 trials of IL-17 inhibitors for psoriatic arthritis have been published, one with secukinumab27 and one with brodalumab.28 Neither had impressive improvement in the ACR score vs TNF inhibitors—39% for ACR 20 at week 12 and less than 10% for ACR 70. Clinical trial design may have played a role, and phase 3 trials are under way for all three IL-17 inhibitors.
Adverse effects of IL-17 inhibitors
For the most part, adverse effects reported with the IL-17 inhibitors have been mild and similar to those reported with available biologic treatments for psoriasis. Adverse effects most commonly reported have been nasopharyngitis, upper respiratory infection, arthralgia, and mild injection-site reactions. In the future, attention will be paid to the rate of infections known to be associated with IL-17, mainly localized infections with Staphylococcus aureus and Candida species. Some patients have developed Candida esophagitis, but this appears to resolve with discontinuation of the drugs. Neutropenia has occurred, but very few patients have developed grade 3 (500–1,000 cells/mm3) or worse. All adverse effects were reversible with discontinuation of treatment.
Approval of secukinumab, and current studies of IL-17 inhibitors
On January 21, 2015, secukinumab was approved by the FDA for treatment of moderate to severe psoriasis vulgaris in adult patients and is now available by prescription.
More trials of IL-17 inhibitors for the treatment of psoriasis and psoriatic arthritis are under way and are at various phases at the time of this writing.23
TARGETED THERAPY FOR ADVANCED MELANOMA
Case 3. A 58-year-old man presents with an irregular pigmented lesion on his back. Biopsy shows malignant melanoma with an intense, chronic inflammatory infiltrate surrounding the tumor (Figure 3). The tumor was surgically excised with standard margins. Two years later, the patient developed multiple pigmented lesions on the face and complained of headache. Magnetic resonance imaging of the brain revealed multiple enhancing lesions consistent with metastatic melanoma (Figure 3). What are this patient’s options?
Melanoma is the fifth most common cancer in humans, with about 132,000 new cases diagnosed worldwide each year and 48,000 deaths from advanced disease. Its incidence has risen rapidly over the last few decades. Advanced disease has a poor prognosis, with the median overall survival less than 1 year and 5-year survival less than 10%.
Despite decades of research, a paucity of FDA-approved medications were available to treat advanced melanoma until recently. The alkylating agent dacarbazine was approved in 1975, interferon alpha in 1995, and high-dose IL-2 in 1998. Although some patients respond, studies have not shown significant improvement in survival with any of these medications.29–31
In 2002, Davies et al32 found that 50% to 65% of metastatic cutaneous melanomas have a mutation in the BRAF gene. Interestingly, 80% of these patients share a single specific mutation: substitution of glutamic acid for valine in codon 600 (BRAF V600E). The second most common mutation is a single substitution of a lysine for that same valine (BRAF V600K). Additionally, NRAS is mutated in about 20% of melanomas. These discoveries implicated a mitogen-activated protein kinase (MAPK) pathway (Figure 4) as playing a critical role in metastatic melanoma for a large percentage of patients.29
Based on this knowledge, several targeted therapies for melanoma have been developed, and some have been approved.
BRAF inhibitors—first success against melanoma
Vemurafenib. In 2010, Flaherty et al33 reported on a phase 1 and phase 2 clinical trial of vemurafenib (960 mg orally twice daily), a potent inhibitor of BRAF with the V600E mutation. They demonstrated a clinical benefit in 80% of patients with stage IV BRAF-mutant melanoma, an unprecedented response that opened the door to changes in the treatment of metastatic melanoma.
The phase 3 BRAF Inhibitor in Melanoma (BRIM)-3 clinical trial,34 published in 2011, randomized 675 previously untreated patients with advanced melanoma to either vemurafenib 960 mg orally twice daily or dacarbazine, the standard of care. The trial was terminated early when an interim analysis showed a significant overall advantage for vemurafenib (median progression-free survival 5.3 months vs 1.6 months for dacarbazine). Based on these results, vemurafenib was FDA-approved in August 2011 for use in patients with BRAF-mutant melanoma.
Dabrafenib. In a phase 3 clinical trial in 2012, Hauschild et al35 randomized 250 patients with BRAF (V600E)-mutated melanoma in a 3:1 ratio to receive either dabrafenib, a more potent second-generation BRAF inhibitor, or dacarbazine. Half of patients responded to dabrafenib, with a significantly improved progression-free survival rate (5.1 vs 2.7 months respectively), leading to FDA approval for its use in BRAF-mutant melanoma in May 2013.
Adverse effects common to vemurafenib and dabrafenib include rash, fatigue, fever, and joint pain. In addition, up to 25% of patients develop multiple secondary cutaneous squamous cell carcinomas and keratoacanthomas, usually within the first few months of therapy, which are believed to be caused by paradoxical activation of the MAPK pathway.
A more important problem with these medications is the development of resistance. Tumors typically progress again after a median progression-free survival of 6 to 7 months.
MEK inhibitors—another line of defense
Inhibitors of MEK—a serine-threonine kinase that is part of the same MAPK pathway involving BRAF—have been developed as well.
Trametinib. In 2012, trametinib, an allosteric MEK inhibitor, was used in an open-label phase 3 trial in 322 patients with advanced melanoma. Progression-free survival was 4.8 months for trametinib-treated patients compared with 1.5 months for the standard chemotherapy group (dacarbazine or paclitaxel).36 These results led to FDA approval of trametinib in May 2013 for treating BRAF-mutant melanoma.29
Cobimetinib is a second MEK inhibitor being evaluated alone and in combination with other targeted treatments for advanced melanoma.
Both MEK inhibitors have adverse effects similar to those seen with the BRAF inhibitors, including diarrhea, rash, fatigue, and edema. They also tend to cause asymptomatic elevated creatine kinase and transient retinopathy, reduced ejection fraction, and ventricular dysfunction. Unlike BRAF inhibitors, they are not associated with development of secondary cutaneous squamous cell carcinomas or keratoacanthomas. However, as with BRAF inhibitors, resistance is a problem with MEK inhibitors, with most patients relapsing less than a year after starting therapy.
Combination therapy improves outcomes
Possible mechanisms underlying resistance to these medications are being studied. A number of important factors appear to drive resistance, including expression of truncated BRAF proteins that do not bind the BRAF inhibitors and still activate downstream signaling, and amplification of BRAF to such a degree that it overwhelms the medications. This has led to the idea of combining BRAF inhibitors and MEK inhibitors to block the MAPK pathway at two points, potentially increasing the response and decreasing resistance.
Two trials have evaluated combinations of BRAF and MEK inhibitors in patients with advanced melanoma. Larkin et al37 conducted a phase 3 study evaluating combined vemurafenib (a BRAF inhibitor) and cobimetinib (a MEK inhibitor) vs combined vemurafenib and placebo. Survival with the combination therapy was 9.9 months vs 6.2 months with the single treatment.
The incidence of serious adverse effects was not significantly increased with the combination therapy, and keratoacanthomas, cutaneous squamous cell carcinomas, alopecia, and arthralgias were reduced compared with the vemurafenib and placebo group.
Another trial38 evaluating combined dabrafenib (a BRAF inhibitor) and trametinib (a MEK inhibitor) vs combined dabrafenib and placebo had similar findings: increased survival in the combined therapy group (9.3 months vs 8.8 months) and lower rates of squamous cell carcinoma (2% vs 9%).
In January 2014, the FDA approved the combination of BRAF and MEK inhibitors for the treatment of BRAF-mutant metastatic melanoma based on improved survival and generally reduced adverse effects.
IMMUNOTHERAPIES FOR NON-BRAF MELANOMA
Although BRAF and MEK inhibitors represent tremendous advances, their use is limited to the approximately 50% to 65% of patients with advanced melanoma who have BRAF V600 mutations. For others, only the traditional standard medications have been available until recently.
Two of those standard FDA-approved medications, interferon alpha-2b and IL-2, represent immunotherapies. Interferon alpha-2b up-regulates antigen presentation and increases antigen recognition by T cells. Overall, about 20% of patients in clinical trials have achieved responses.
IL-2 is a cytokine that increases T-cell proliferation and maturation into effector T cells. High-dose IL-2 has produced responses in 15% of patients, with a durable complete response in a small proportion.
Though success with these medications was modest, the fact that some patients responded to them indicates that immunotherapy could be a viable strategy for treating metastatic melanoma.30 This is underscored by the fact that some patients can mount an adaptive immune response specifically directed against antigenic proteins expressed in their tumors, resulting in expansion of cytotoxic T cells and control or even elimination of the malignancy.30
Tumors manipulate host immune checkpoints
Molecular biology has provided tremendous insight into tumor immunology over the past several decades, and we now recognize that a hallmark of cancer is escape from immune control.
Cancer cells contain a multitude of mutations that produce proteins that should be recognized by the immune system as foreign but in most individuals are not. This is because T-cell activity is down-regulated in cancer due to cancer cells’ ability to manipulate the host’s normal immunologic inhibitory pathways critical for maintaining self-tolerance.
In general, T-cell activation is initiated when an antigen-presenting cell presents an antigen to a T cell in a major histocompatibility complex-restricted manner. To prevent T cells from being activated by self-antigens and initiating autoimmunity, the interaction between antigen-presenting cells and T cells is regulated by checkpoints (Figure 5). First, for an antigen-presenting cell/T-cell interaction to result in T-cell activation, the T-cell receptor CD28 must bind CD80 on the antigen-presenting cell to drive a “positive” signal. Early in the interaction, the T-cell receptor CTLA-4 is up-regulated and competes with CD28 for binding of CD80. If CTLA-4, and not CD28, binds CD80, a “negative” signal is sent to the T cell and down-regulates it, making the interaction unproductive. Importantly, it is the CTLA-4:CD80 interaction that appears to be crucial for the ability of tumors to dampen T-cell responses to cancer cells.
Ipilimumab is a fully humanized monoclonal antibody that binds to CTLA-4, blocking its ability to bind to CD80 and thereby enhancing T-cell activation. In a phase 3 trial, Hodi et al39 evaluated its use in treating advanced melanoma, with some enrolled patients having failed IL-2 treatment. Patients receiving ipilimumab with or without a glycoprotein-100 peptide vaccine (gp100) had an overall survival benefit of 10.1 months compared with 6.4 months for patients treated with gp100 alone. At 24 months, the survival rate with ipilimumab alone was 23.5%, almost double that of patients receiving gp100 alone.
Ipilimumab received FDA approval for treatment of metastatic melanoma in March 2011. This, and the BRAF inhibitors, were the first drugs approved by the FDA for the treatment of advanced melanoma in more than a decade.
Common adverse effects of ipilimumab include fatigue, diarrhea, rash, and pruritus. As expected, given its mechanism of action, up to about 25% of patients experience severe autoimmune-related events that may variably manifest as colitis, rash, hepatitis, neuritis, hypothyroidism, hypopituitarism, and hypophysitis. Another problem with this medication is that a subset of patients do not respond.
Cancer cells disguised as normal cells
Cancer cells can also manipulate another immunologic checkpoint to evade attack by the host immune system (Figure 5). Cytotoxic T cells may recognize antigens on tumor cells and become activated and primed to directly destroy them. However, tumor cells, like normal cells express the programmed death ligands RTK-L1 and PD-L2. These ligands function to bind to the PD-1 receptor on activated T cells to indicate they are “self” and inhibit the cytotoxic T cells from destroying them.
Evasion of immune system attack by manipulating checkpoints involving CTLA-4 and PD-1 helps explain why malignancies can seemingly be associated with brisk inflammatory responses, such as the tumor in Case 3, yet progress and eventually metastasize (Figure 3).
Two medications—nivolumab and pembrolizumab—have been developed in an attempt to disrupt the ability of tumor cells to trick the immune system into accepting them as “self” by manipulating the PD-L1/PD-L2: PD-1 interaction. Both drugs are monoclonal antibodies that bind to PD-1 and, thus, effectively block the ability of PD-L1 or PD-L2 on tumor cells to bind these ligands and signal to activated T cells that they are “self.” This blocking allows T cells to then carry out their killing of tumor cells they initially recognize as foreign.
Nivolumab. In 2014, a phase 3 trial40 compared nivolumab and dacarbazine in patients with untreated advanced melanoma without a BRAF mutation. Objective response rates were 40.0% in the nivolumab group vs 13.9% in the dacarbazine group. This trial was stopped early because of significantly better survival rates in patients taking nivolumab compared with standard chemotherapy.
Interestingly, only 35% of patients who responded to nivolumab had evidence of PD-L1 expression on the surface of their tumor cells as assessed by immunohistochemical assay. Regardless of PD-L1 status, nivolumab-treated patients had improved overall survival compared with those treated with dacarbazine. The response rate with nivolumab was only slightly better in the subgroup of patients whose tumors expressed PD-L1 than in the subgroup without PD-L1.
On December 22, 2014, the FDA granted accelerated approval to nivolumab for the treatment of patients with unresectable or metastatic melanoma and disease progression following ipilimumab treatment and, if BRAF V600 mutation-positive, a BRAF inhibitor.
Pembrolizumab. Also in 2014, an open-label, randomized, phase 1b trial of pembrolizumab treatment at two different dosage schedules was conducted in patients with advanced melanoma that had become refractory either to ipilimumab or a BRAF inhibitor.41 Treatment with pembrolizumab had an objective response rate of 26% at both doses.
In September 2014, the FDA granted accelerated approval for the use of pembrolizumab to treat patients with unresectable or metastatic melanoma and disease progression following treatment with ipilimumab or a BRAF inhibitor.
Adverse effects of PD-1 inhibitors are similar to those seen with ipilimumab, the most common (occurring in at least 20%) being fatigue, cough, nausea, pruritus, rash, decreased appetite, constipation, muscle pain, and diarrhea. Serious effects from pembrolizumab (occurring in at least 2%) were kidney failure, dyspnea, pneumonia, and cellulitis. As seen with ipilimumab, clinically significant autoimmune adverse reactions occur with PD-1 inhibitors, including pneumonitis, colitis, hypophysitis, nephritis, and hepatitis.
Combination therapy under investigation
A phase 1 trial using combination therapy with both immune checkpoint inhibitors—nivolumab (anti-PD-1) and ipilimumab (anti-CTLA-4)—in patients with treatment-resistant metastatic melanoma was published in 2013.42 More than half of patients achieved objective responses, with tumor regression of at least 80% in those who had a response. Tumor response was evident in all subgroups of patients studied—those with pretreatment elevated lactate dehydrogenase levels (one of the strongest prognostic factors in metastatic melanoma), metastases to distant sites, and bulky, multifocal tumor burden. Based on these results, a phase 3 trial is now under way looking at the combination of these two medications vs either one alone.
In summary, targeted treatments are changing the paradigm of how common dermatologic conditions associated with significant morbidity and mortality are treated. Although implementation of the above treatments into everyday clinical practice is exciting, future studies surrounding each are needed to address unanswered issues, such as the optimal dosing and treatment schedules in terms of both disease response and inhibition of resistance, optimal patient/disease characteristics for use, and optimal drug treatment combinations. In the meantime, basic research still utilizing classic molecular biology techniques to uncover pathogenic disease mechanisms in even more detail is ongoing and hopefully will lead to development of even better targeted treatments or even cures for these diseases.
New targeted therapies are changing the way patients with advanced dermatologic diseases are treated. Innovative molecular biology techniques developed as far back as the 1970s have engendered tremendous insight into the cellular and molecular pathogenesis of numerous diseases. Novel medications based on these insights are now bearing fruit, as directed biologic therapies that are revolutionizing clinical practice are increasingly becoming available.
This article reviews advances in targeted therapies for advanced basal cell carcinoma, psoriasis, and metastatic melanoma.
TARGETED THERAPY FOR BASAL CELL CARCINOMA
Case 1. A 56-year-old man presents with a progressively enlarging leg ulcer. Although it has been treated empirically for years as a venous stasis ulcer, biopsy reveals that it is basal cell carcinoma. Imaging shows muscle and tendon invasion, making surgical intervention short of amputation challenging (Figure 1). What are his options?
Basal cell carcinoma is the most common cancer in humans, accounting for 25% of all cancers and more than 2 million cases in the United States every year. In most cases, surgical excision is curative, but a subset of patients have inoperable, locally advanced, or metastatic disease that drastically limits treatment options. The median survival in metastatic basal cell carcinoma is 24 months, and conventional chemotherapy has not been shown to improve the prognosis.1,2
In addition to the burden of sporadic basal cell carcinoma, patients with the rare autosomal-dominant genetic disorder basal cell nevus syndrome (Gorlin syndrome) develop multiple basal cell lesions over their lifetime. The syndrome may also involve abnormalities of the skeletal system, genitourinary tract, and central nervous system, including development of medulloblastoma.
In Gorlin syndrome, basal cell carcinomas occur often and early; about half of white patients with the syndrome develop their first lesions by age 21, and 90% by age 35. The lesions occur in multiple numbers and can develop anywhere on the body, including on non–sun-exposed areas. Patients who have Gorlin syndrome need meticulous monitoring every 2 to 3 months so that basal cell lesions can be recognized early and treated before they become locally advanced. Keeping up with the numerous medical appointments and invasive treatments can be physically and mentally taxing for patients.
Specific pathway and mutations identified
In 1996, Gorlin syndrome was found to be caused by mutations of the human homolog of the PATCHED gene, which codes for a receptor in the “hedgehog” pathway.3 Two years later, the same mutations were found to be involved in many sporadic basal cell carcinomas, and we now believe that at least 85% of basal cell carcinomas involve abnormal activation of hedgehog pathway signaling.4,5
Vismodegib developed as targeted therapy
In 2009, Robarge et al6 described a potent inhibitor of the hedgehog pathway that was later optimized for potency and desirable pharmacologic traits, resulting in the drug vismodegib.7,8
Two phase 2 multicenter clinical trials9,10 of vismodegib were published in 2012. In the first, which was not randomized,9 33 patients with metastatic basal cell carcinoma and 63 patients with locally advanced disease were treated with vismodegib. Of those with metastatic disease, 30% achieved an objective response. Of those with locally advanced disease, 43% achieved an objective response and 21% achieved a complete response.
In the second trial,10 patients with Gorlin syndrome were randomized to either vismodegib (26 patients) or placebo (16 patients). After 8 months, the vismodegib group had developed significantly fewer new surgically eligible tumors (2 vs 29 per year), their tumors were smaller (change from baseline of the sum of the longest diameters –65% vs –11%), and they needed fewer surgeries (mean 0.31 vs 4.4 per patient). No tumors progressed in the treatment group. Results in some patients were dramatic, with complete healing of large ulcerative tumors. The trial was ended early in view of significant efficacy in the treatment group.
Based on these trials, the US Food and Drug Administration (FDA) approved vismodegib for treating metastatic and locally advanced basal cell carcinoma.
Resistance and adverse effects common
Unfortunately, vismodegib has significant drawbacks. About 20% of patients develop resistance, with tumors recurring after several months of therapy.11 Adverse effects most commonly reported were muscle spasms (68%), alopecia (63%), taste distortion (51%), weight loss (46%), and fatigue (36%). Although these effects were considered mild or moderate, they tended to persist, and almost every patient developed at least one. In the nonrandomized trial,9 more than 25% of patients discontinued treatment because of adverse effects, and more than half of patients did the same in the basal cell nevus syndrome trial.10
New uses may reduce shortcomings
Studies are under way to determine how best to use vismodegib.
One possibility is to use the drug for a few months to shrink tumors to the point that they become eligible for surgery. This is especially important for high-risk tumors, such as those near the eye or other vital structures. In 11 patients, Ally et al12 found that the surgical defect area was reduced by 27% from baseline after 4 months of treatment with vismodegib, allowing for curative surgery in some.
Another option is to combine vismodegib with other agents—either new ones on the horizon or existing nonspecific medications. For example, the antifungal itraconazole has been shown to inhibit hedgehog signaling and perhaps could be combined with vismodegib to increase response and reduce resistance.
Finally, a topical or intralesional form of vismodegib would be useful not only to reduce systemic toxicity, but also to increase efficacy when combined with other topical or systemic medications.
TARGETED THERAPY FOR PSORIASIS VULGARIS
Case 2. A 28-year-old woman presents with worsening psoriasis. About 35% of her body surface is involved, including the palms and soles, making it difficult for her to perform activities of daily living (Figure 2). What are her options?
Psoriasis is a chronic immune-mediated disease that affects up to 3% of people worldwide. In its moderate to severe forms, we recognize psoriasis as a systemic inflammatory disease that may adversely affect organ systems other than the skin. Commonly associated comorbid diseases include inflammatory (psoriatic) arthritis, cardiovascular disease, malignancies (eg, lymphoma), and inflammatory bowel disease. In addition, patients are well known to have significantly impaired quality of life because of low self-esteem, stigmatization affecting their social and work relationships, and, in up to 60%, clinical depression.13,14 The onset of psoriatic arthritis, particularly of erosive disease, is an important decision point for starting aggressive treatment, as joint destruction is irreversible.
Early targeted therapy aimed at TNF alpha, IL-12, and IL-23
Histologically, psoriasis involves thickening of the epidermis caused by hyperproliferation of keratinocytes. Based on this, prior to the 1980s, the dominant hypothesis concerning its pathogenesis was that it was caused by an inherent defect of keratinocytes. In the 1980s and 1990s, however, molecular research revealed that psoriasis was an immune-mediated disease caused by immunologic dysregulation predominantly involving T-helper 1 (Th-1) cells, with the inflammatory cytokines tumor necrosis factor (TNF) alpha, interferon gamma, interleukin (IL) 12, and IL-23 playing prominent roles.15 These findings led to the development and FDA approval of the first effective, targeted, psoriasis treatments, TNF-alpha inhibitors and the IL-12/23 inhibitor ustekinumab.
Etanercept, the first TNF-alpha inhibitor to become available, was approved in 2004 for moderate to severe psoriasis. In 2008, the IL-12/23 inhibitor ustekinumab was approved for the same indication. These drugs are efficacious, are generally safe, and have revolutionized the treatment of psoriasis and psoriatic arthritis, and they are now prescribed on a daily basis.16,17
In the clinical trials that led to approval of these drugs, the main outcome measure was the Psoriasis Area and Severity Index (PASI), a clinical scoring tool that assesses clinical aspects of psoriatic disease including body surface area involvement, degree of thickness, erythema, and scaling of psoriatic plaques. PASI scores range from 0 (no psoriasis) to 72 (most severe psoriasis). Achieving “PASI 75” indicates at least 75% improvement from the baseline score and represents the most common primary outcome measure in clinical trials assessing efficacy of new treatments. Up to 80% of patients who received currently available TNF-alpha inhibitors and ustekinumab in pivotal clinical trials achieved PASI 75 when assessed at 12 to 16 weeks after starting treatment. A moderate percentage of patients (19%–57%, depending on the trial) achieved 90% improvement (PASI 90), and a minority (up to 18%) achieved PASI 100, indicating complete clearing of their psoriasis.18–22
Newly developed therapies target IL-17A
In the mid-2000s, Th-17 cells were discovered, a new lineage of T cells distinct from Th-1 and Th-2 cells. Th-17 cells are characterized by their production of IL-17, a pro-inflammatory cytokine with six family members (IL-17A through IL-17F). Over the next few years, experiments revealed that Th-17 cells and IL-17A play key roles in psoriasis immunologic dysregulation.15 These findings led to a paradigm shift in hypotheses concerning psoriasis pathogenesis, with Th-17 cells and IL-17 replacing Th-1 cells and associated cytokines as dominant mediators of tissue damage.
Additionally, these findings led to new ideas for treatment. Three monoclonal antibodies that target IL-17 inhibition are currently under investigation. Secukinumab and ixekizumab bind to IL-17A and inhibit it from downstream signaling, whereas brodalumab binds to the IL-17A receptor, blocking all six IL-17 cytokines (IL-17A to IL-17F).23
Clinical trials of IL-17 inhibitors show excellent skin improvement
Secukinumab. In 2014, the results of two phase 3 trials of secukinumab were published.24
In the Efficacy and Safety of Subcutaneous Secukinumab for Moderate to Severe Chronic Plaque-type Psoriasis for up to 1 Year trial,24 patients were given either secukinumab 300 mg or 150 mg subcutaneously at defined time points; 82% and 72%, respectively, attained PASI 75 at 12 weeks.
Similar results were seen in the Safety and Efficacy of Secukinumab Compared to Etanercept in Subjects With Moderate to Severe, Chronic Plaque-Type Psoriasis study,24 in which PASI 75 was achieved by 77% of patients receiving secukinumab 300 mg, 67% of those receiving secukinumab 150 mg, and only 44% of those receiving etanercept 50 mg twice weekly at 12 weeks. Rates of infection with secukinumab and etanercept were similar.
The most striking results of these trials were that more than half of patients receiving the 300-mg dose achieved at least 90% improvement in their PASI score (PASI 90) by week 12, and in more than a quarter of patients the psoriasis completely cleared (PASI 100). These results were dramatically better than for etanercept (PASI 90 21%; PASI 100 4%).
Additionally, secukinumab worked fast. The median time to PASI 50 with secukinumab 300 mg was less than half that seen with etanercept (3 weeks vs 7 weeks).
Ixekizumab. In 2012, a phase 2 trial evaluated subcutaneous injections of ixekizumab in dosages ranging from 10 to 150 mg at defined intervals for 16 weeks.25 Of those receiving the highest dosage, 82% attained PASI 75 at 12 weeks, on par with what is noted in patients receiving TNF-alpha inhibitors and IL-12/23 inhibitors. Remarkably, however, almost three-quarters of patients (71%) achieved PASI 90, and 39% achieved PASI 100. Improvement in psoriasis was apparent as early as 1 week after injection.
Brodalumab. A 2012 phase 2 trial of various dosages of the IL-17 receptor inhibitor brodalumab26 also showed excellent PASI 75 achievement with the highest dosage (82%). Astonishingly, though, PASI 90 was achieved by 75% of patients, and PASI 100 by 62%.
Overall, although the percentages of patients achieving PASI 75 with the new IL-17 inhibitor drugs are comparable to those seen with TNF-alpha inhibitors and IL-12/23 inhibitors, the extraordinarily high percentages of patients who achieved PASI 90 and PASI 100 are unprecedented.18–22
Arthritis improvement not shown
Where the IL-17 inhibitors eventually settle within algorithms of psoriasis treatment largely depends on their efficacy in treating psoriatic arthritis compared with TNF-alpha inhibitors and IL-12/23 inhibitors. Joint inflammation is typically evaluated with the American College of Rheumatology (ACR) scoring tool, which in simple terms can be thought of as analogous to the PASI scoring tool for the skin. Although the ACR scoring tool was developed to assess joint inflammation in clinical trials for patients with rheumatoid arthritis, it is commonly used to assess improvement of psoriatic arthritis in clinical trials. The ACR tool involves assessing and scoring the number of swollen and tender joints, but also incorporates serologic assessment of acute-phase reactants (erythrocyte sedimentation rate or C-reactive protein level), patient and physician global assessment, pain, and function. ACR 20 implies roughly a 20% improvement in these criteria, whereas ACR 50 indicates 50% improvement, and so on.
Two phase 2 trials of IL-17 inhibitors for psoriatic arthritis have been published, one with secukinumab27 and one with brodalumab.28 Neither had impressive improvement in the ACR score vs TNF inhibitors—39% for ACR 20 at week 12 and less than 10% for ACR 70. Clinical trial design may have played a role, and phase 3 trials are under way for all three IL-17 inhibitors.
Adverse effects of IL-17 inhibitors
For the most part, adverse effects reported with the IL-17 inhibitors have been mild and similar to those reported with available biologic treatments for psoriasis. Adverse effects most commonly reported have been nasopharyngitis, upper respiratory infection, arthralgia, and mild injection-site reactions. In the future, attention will be paid to the rate of infections known to be associated with IL-17, mainly localized infections with Staphylococcus aureus and Candida species. Some patients have developed Candida esophagitis, but this appears to resolve with discontinuation of the drugs. Neutropenia has occurred, but very few patients have developed grade 3 (500–1,000 cells/mm3) or worse. All adverse effects were reversible with discontinuation of treatment.
Approval of secukinumab, and current studies of IL-17 inhibitors
On January 21, 2015, secukinumab was approved by the FDA for treatment of moderate to severe psoriasis vulgaris in adult patients and is now available by prescription.
More trials of IL-17 inhibitors for the treatment of psoriasis and psoriatic arthritis are under way and are at various phases at the time of this writing.23
TARGETED THERAPY FOR ADVANCED MELANOMA
Case 3. A 58-year-old man presents with an irregular pigmented lesion on his back. Biopsy shows malignant melanoma with an intense, chronic inflammatory infiltrate surrounding the tumor (Figure 3). The tumor was surgically excised with standard margins. Two years later, the patient developed multiple pigmented lesions on the face and complained of headache. Magnetic resonance imaging of the brain revealed multiple enhancing lesions consistent with metastatic melanoma (Figure 3). What are this patient’s options?
Melanoma is the fifth most common cancer in humans, with about 132,000 new cases diagnosed worldwide each year and 48,000 deaths from advanced disease. Its incidence has risen rapidly over the last few decades. Advanced disease has a poor prognosis, with the median overall survival less than 1 year and 5-year survival less than 10%.
Despite decades of research, a paucity of FDA-approved medications were available to treat advanced melanoma until recently. The alkylating agent dacarbazine was approved in 1975, interferon alpha in 1995, and high-dose IL-2 in 1998. Although some patients respond, studies have not shown significant improvement in survival with any of these medications.29–31
In 2002, Davies et al32 found that 50% to 65% of metastatic cutaneous melanomas have a mutation in the BRAF gene. Interestingly, 80% of these patients share a single specific mutation: substitution of glutamic acid for valine in codon 600 (BRAF V600E). The second most common mutation is a single substitution of a lysine for that same valine (BRAF V600K). Additionally, NRAS is mutated in about 20% of melanomas. These discoveries implicated a mitogen-activated protein kinase (MAPK) pathway (Figure 4) as playing a critical role in metastatic melanoma for a large percentage of patients.29
Based on this knowledge, several targeted therapies for melanoma have been developed, and some have been approved.
BRAF inhibitors—first success against melanoma
Vemurafenib. In 2010, Flaherty et al33 reported on a phase 1 and phase 2 clinical trial of vemurafenib (960 mg orally twice daily), a potent inhibitor of BRAF with the V600E mutation. They demonstrated a clinical benefit in 80% of patients with stage IV BRAF-mutant melanoma, an unprecedented response that opened the door to changes in the treatment of metastatic melanoma.
The phase 3 BRAF Inhibitor in Melanoma (BRIM)-3 clinical trial,34 published in 2011, randomized 675 previously untreated patients with advanced melanoma to either vemurafenib 960 mg orally twice daily or dacarbazine, the standard of care. The trial was terminated early when an interim analysis showed a significant overall advantage for vemurafenib (median progression-free survival 5.3 months vs 1.6 months for dacarbazine). Based on these results, vemurafenib was FDA-approved in August 2011 for use in patients with BRAF-mutant melanoma.
Dabrafenib. In a phase 3 clinical trial in 2012, Hauschild et al35 randomized 250 patients with BRAF (V600E)-mutated melanoma in a 3:1 ratio to receive either dabrafenib, a more potent second-generation BRAF inhibitor, or dacarbazine. Half of patients responded to dabrafenib, with a significantly improved progression-free survival rate (5.1 vs 2.7 months respectively), leading to FDA approval for its use in BRAF-mutant melanoma in May 2013.
Adverse effects common to vemurafenib and dabrafenib include rash, fatigue, fever, and joint pain. In addition, up to 25% of patients develop multiple secondary cutaneous squamous cell carcinomas and keratoacanthomas, usually within the first few months of therapy, which are believed to be caused by paradoxical activation of the MAPK pathway.
A more important problem with these medications is the development of resistance. Tumors typically progress again after a median progression-free survival of 6 to 7 months.
MEK inhibitors—another line of defense
Inhibitors of MEK—a serine-threonine kinase that is part of the same MAPK pathway involving BRAF—have been developed as well.
Trametinib. In 2012, trametinib, an allosteric MEK inhibitor, was used in an open-label phase 3 trial in 322 patients with advanced melanoma. Progression-free survival was 4.8 months for trametinib-treated patients compared with 1.5 months for the standard chemotherapy group (dacarbazine or paclitaxel).36 These results led to FDA approval of trametinib in May 2013 for treating BRAF-mutant melanoma.29
Cobimetinib is a second MEK inhibitor being evaluated alone and in combination with other targeted treatments for advanced melanoma.
Both MEK inhibitors have adverse effects similar to those seen with the BRAF inhibitors, including diarrhea, rash, fatigue, and edema. They also tend to cause asymptomatic elevated creatine kinase and transient retinopathy, reduced ejection fraction, and ventricular dysfunction. Unlike BRAF inhibitors, they are not associated with development of secondary cutaneous squamous cell carcinomas or keratoacanthomas. However, as with BRAF inhibitors, resistance is a problem with MEK inhibitors, with most patients relapsing less than a year after starting therapy.
Combination therapy improves outcomes
Possible mechanisms underlying resistance to these medications are being studied. A number of important factors appear to drive resistance, including expression of truncated BRAF proteins that do not bind the BRAF inhibitors and still activate downstream signaling, and amplification of BRAF to such a degree that it overwhelms the medications. This has led to the idea of combining BRAF inhibitors and MEK inhibitors to block the MAPK pathway at two points, potentially increasing the response and decreasing resistance.
Two trials have evaluated combinations of BRAF and MEK inhibitors in patients with advanced melanoma. Larkin et al37 conducted a phase 3 study evaluating combined vemurafenib (a BRAF inhibitor) and cobimetinib (a MEK inhibitor) vs combined vemurafenib and placebo. Survival with the combination therapy was 9.9 months vs 6.2 months with the single treatment.
The incidence of serious adverse effects was not significantly increased with the combination therapy, and keratoacanthomas, cutaneous squamous cell carcinomas, alopecia, and arthralgias were reduced compared with the vemurafenib and placebo group.
Another trial38 evaluating combined dabrafenib (a BRAF inhibitor) and trametinib (a MEK inhibitor) vs combined dabrafenib and placebo had similar findings: increased survival in the combined therapy group (9.3 months vs 8.8 months) and lower rates of squamous cell carcinoma (2% vs 9%).
In January 2014, the FDA approved the combination of BRAF and MEK inhibitors for the treatment of BRAF-mutant metastatic melanoma based on improved survival and generally reduced adverse effects.
IMMUNOTHERAPIES FOR NON-BRAF MELANOMA
Although BRAF and MEK inhibitors represent tremendous advances, their use is limited to the approximately 50% to 65% of patients with advanced melanoma who have BRAF V600 mutations. For others, only the traditional standard medications have been available until recently.
Two of those standard FDA-approved medications, interferon alpha-2b and IL-2, represent immunotherapies. Interferon alpha-2b up-regulates antigen presentation and increases antigen recognition by T cells. Overall, about 20% of patients in clinical trials have achieved responses.
IL-2 is a cytokine that increases T-cell proliferation and maturation into effector T cells. High-dose IL-2 has produced responses in 15% of patients, with a durable complete response in a small proportion.
Though success with these medications was modest, the fact that some patients responded to them indicates that immunotherapy could be a viable strategy for treating metastatic melanoma.30 This is underscored by the fact that some patients can mount an adaptive immune response specifically directed against antigenic proteins expressed in their tumors, resulting in expansion of cytotoxic T cells and control or even elimination of the malignancy.30
Tumors manipulate host immune checkpoints
Molecular biology has provided tremendous insight into tumor immunology over the past several decades, and we now recognize that a hallmark of cancer is escape from immune control.
Cancer cells contain a multitude of mutations that produce proteins that should be recognized by the immune system as foreign but in most individuals are not. This is because T-cell activity is down-regulated in cancer due to cancer cells’ ability to manipulate the host’s normal immunologic inhibitory pathways critical for maintaining self-tolerance.
In general, T-cell activation is initiated when an antigen-presenting cell presents an antigen to a T cell in a major histocompatibility complex-restricted manner. To prevent T cells from being activated by self-antigens and initiating autoimmunity, the interaction between antigen-presenting cells and T cells is regulated by checkpoints (Figure 5). First, for an antigen-presenting cell/T-cell interaction to result in T-cell activation, the T-cell receptor CD28 must bind CD80 on the antigen-presenting cell to drive a “positive” signal. Early in the interaction, the T-cell receptor CTLA-4 is up-regulated and competes with CD28 for binding of CD80. If CTLA-4, and not CD28, binds CD80, a “negative” signal is sent to the T cell and down-regulates it, making the interaction unproductive. Importantly, it is the CTLA-4:CD80 interaction that appears to be crucial for the ability of tumors to dampen T-cell responses to cancer cells.
Ipilimumab is a fully humanized monoclonal antibody that binds to CTLA-4, blocking its ability to bind to CD80 and thereby enhancing T-cell activation. In a phase 3 trial, Hodi et al39 evaluated its use in treating advanced melanoma, with some enrolled patients having failed IL-2 treatment. Patients receiving ipilimumab with or without a glycoprotein-100 peptide vaccine (gp100) had an overall survival benefit of 10.1 months compared with 6.4 months for patients treated with gp100 alone. At 24 months, the survival rate with ipilimumab alone was 23.5%, almost double that of patients receiving gp100 alone.
Ipilimumab received FDA approval for treatment of metastatic melanoma in March 2011. This, and the BRAF inhibitors, were the first drugs approved by the FDA for the treatment of advanced melanoma in more than a decade.
Common adverse effects of ipilimumab include fatigue, diarrhea, rash, and pruritus. As expected, given its mechanism of action, up to about 25% of patients experience severe autoimmune-related events that may variably manifest as colitis, rash, hepatitis, neuritis, hypothyroidism, hypopituitarism, and hypophysitis. Another problem with this medication is that a subset of patients do not respond.
Cancer cells disguised as normal cells
Cancer cells can also manipulate another immunologic checkpoint to evade attack by the host immune system (Figure 5). Cytotoxic T cells may recognize antigens on tumor cells and become activated and primed to directly destroy them. However, tumor cells, like normal cells express the programmed death ligands RTK-L1 and PD-L2. These ligands function to bind to the PD-1 receptor on activated T cells to indicate they are “self” and inhibit the cytotoxic T cells from destroying them.
Evasion of immune system attack by manipulating checkpoints involving CTLA-4 and PD-1 helps explain why malignancies can seemingly be associated with brisk inflammatory responses, such as the tumor in Case 3, yet progress and eventually metastasize (Figure 3).
Two medications—nivolumab and pembrolizumab—have been developed in an attempt to disrupt the ability of tumor cells to trick the immune system into accepting them as “self” by manipulating the PD-L1/PD-L2: PD-1 interaction. Both drugs are monoclonal antibodies that bind to PD-1 and, thus, effectively block the ability of PD-L1 or PD-L2 on tumor cells to bind these ligands and signal to activated T cells that they are “self.” This blocking allows T cells to then carry out their killing of tumor cells they initially recognize as foreign.
Nivolumab. In 2014, a phase 3 trial40 compared nivolumab and dacarbazine in patients with untreated advanced melanoma without a BRAF mutation. Objective response rates were 40.0% in the nivolumab group vs 13.9% in the dacarbazine group. This trial was stopped early because of significantly better survival rates in patients taking nivolumab compared with standard chemotherapy.
Interestingly, only 35% of patients who responded to nivolumab had evidence of PD-L1 expression on the surface of their tumor cells as assessed by immunohistochemical assay. Regardless of PD-L1 status, nivolumab-treated patients had improved overall survival compared with those treated with dacarbazine. The response rate with nivolumab was only slightly better in the subgroup of patients whose tumors expressed PD-L1 than in the subgroup without PD-L1.
On December 22, 2014, the FDA granted accelerated approval to nivolumab for the treatment of patients with unresectable or metastatic melanoma and disease progression following ipilimumab treatment and, if BRAF V600 mutation-positive, a BRAF inhibitor.
Pembrolizumab. Also in 2014, an open-label, randomized, phase 1b trial of pembrolizumab treatment at two different dosage schedules was conducted in patients with advanced melanoma that had become refractory either to ipilimumab or a BRAF inhibitor.41 Treatment with pembrolizumab had an objective response rate of 26% at both doses.
In September 2014, the FDA granted accelerated approval for the use of pembrolizumab to treat patients with unresectable or metastatic melanoma and disease progression following treatment with ipilimumab or a BRAF inhibitor.
Adverse effects of PD-1 inhibitors are similar to those seen with ipilimumab, the most common (occurring in at least 20%) being fatigue, cough, nausea, pruritus, rash, decreased appetite, constipation, muscle pain, and diarrhea. Serious effects from pembrolizumab (occurring in at least 2%) were kidney failure, dyspnea, pneumonia, and cellulitis. As seen with ipilimumab, clinically significant autoimmune adverse reactions occur with PD-1 inhibitors, including pneumonitis, colitis, hypophysitis, nephritis, and hepatitis.
Combination therapy under investigation
A phase 1 trial using combination therapy with both immune checkpoint inhibitors—nivolumab (anti-PD-1) and ipilimumab (anti-CTLA-4)—in patients with treatment-resistant metastatic melanoma was published in 2013.42 More than half of patients achieved objective responses, with tumor regression of at least 80% in those who had a response. Tumor response was evident in all subgroups of patients studied—those with pretreatment elevated lactate dehydrogenase levels (one of the strongest prognostic factors in metastatic melanoma), metastases to distant sites, and bulky, multifocal tumor burden. Based on these results, a phase 3 trial is now under way looking at the combination of these two medications vs either one alone.
In summary, targeted treatments are changing the paradigm of how common dermatologic conditions associated with significant morbidity and mortality are treated. Although implementation of the above treatments into everyday clinical practice is exciting, future studies surrounding each are needed to address unanswered issues, such as the optimal dosing and treatment schedules in terms of both disease response and inhibition of resistance, optimal patient/disease characteristics for use, and optimal drug treatment combinations. In the meantime, basic research still utilizing classic molecular biology techniques to uncover pathogenic disease mechanisms in even more detail is ongoing and hopefully will lead to development of even better targeted treatments or even cures for these diseases.
- Lyons TG, O’Kane GM, Kelly CM. Efficacy and safety of vismodegib: a new therapeutic agent in the treatment of basal cell carcinoma. Expert Opin Drug Saf 2014; 13:1125–1132.
- McCusker M, Basset-Sequin N, Dummer R, et al. Metastatic basal cell carcinoma: prognosis dependent on anatomic site and spread of disease. Eur J Cancer 2014; 50:774–783.
- Hahn H, Wicking C, Zaphiropoulous PG, et al. Mutations of the human homolog of Drosophila patched in the nevoid basal cell carcinoma syndrome. Cell 1996; 85:841–851.
- Aszterbaum M, Rothman A, Johnson RL, et al. Identification of mutations in the human PATCHED gene in sporadic basal cell carcinomas and in patients with the basal cell nevus syndrome. J Invest Dermatol 1998; 110:885–888.
- Ingham PW, Placzek M. Orchestrating ontogenesis: variations on a theme by sonic hedgehog. Nat Rev Genet 2006; 7:841–850.
- Robarge KD, Brunton SA, Castanedo GM, et al. GDC-0449-a potent inhibitor of the hedgehog pathway. Bioorg Med Chem Lett 2009; 19:5576–5581.
- Proctor AE, Thompson LA, O’Bryant CL. Vismodegib: an inhibitor of the Hedgehog signaling pathway in the treatment of basal cell carcinoma. Ann Pharmacother 2014; 48:99–106.
- Dessinioti C, Plaka M, Stratigos AJ. Vismodegib for the treatment of basal cell carcinoma: results and implications of the ERIVANCE BCC trial. Future Oncol 2014; 10:927–936.
- Sekulic A, Migden MR, Oro AE, et al. Efficacy and safety of vismodegib in advanced basal-cell carcinoma. N Engl J Med 2012; 366:2171–2179.
- Tang JY, Mackay-Wiggan JM, Aszterbaum M, et al. Inhibiting the hedgehog pathway in patients with basal-cell nevus syndrome. N Engl J Med 2012; 366:2180–2188.
- Brinkhuizen T, Reinders MG, van Geel M, et al. Acquired resistance to the Hedgehog pathway inhibitor vismodegib due to smoothened mutations in treatment of locally advanced basal cell carcinoma. J Am Acad Dermatol 2014; 71:1005–1008.
- Ally MS, Aasi S, Wysong A, et al. An investigator-initiated open-label clinical trial of vismodegib as a neoadjuvant to surgery for high-risk basal cell carcinoma. J Am Acad Dermatol 2014; 71:904–911.
- Rapp SR, Feldman SR, Exum ML, Fleischer AB Jr, Reboussin DM. Psoriasis causes as much disability as other major medical diseases. J Am Acad Dermatol 1999; 41:401–407.
- Gelfand JM, Niemann AL, Shin DB, Wang X, Margolis DJ, Troxel AB. Risk of myocardial infarction in patients with psoriasis. JAMA 2006; 296:1735–1741.
- Lynde CW, Poulin Y, Vender R, Bourcier M, Khalil S. Interleukin 17A: toward a new understanding of psoriasis pathogenesis. J Am Acad Dermatol 2014; 71:141–150.
- Tracey D, Klareskog L, Sasso EH, Salfeld JG, Tak PP. Tumor necrosis factor antagonist mechanisms of action: a comprehensive review. Pharmacol Ther 2008; 117:244–279.
- Nestle FL, Kaplan DH, Barker J. Psoriasis. N Engl J Med 2009; 361:496–509.
- Mentor A, Tyring SK, Gordon K, et al. Adalimumab therapy for moderate to severe psoriasis: a randomized, controlled phase III trial. J Am Acad Dermatol 2007; 58:106–115.
- Leonardi CL, Powers JL, Matheson RT, et al. Etanercept as monotherapy in patients with psoriasis. N Engl J Med 2003; 349:2014–2022.
- Reich K, Nestle FO, Papp K, et al. Infliximab induction and maintenance therapy for moderate-to-severe psoriasis: a phase III, multicentre, double-blind trial. Lancet 2005; 366:1367–1374.
- Leonardi CL, Kimball AB, Papp KA, et al. Efficacy and safety of ustekinumab, a human interleukin-12/23 monoclonal antibody, in patients with psoriasis: 76-week results from a randomised, double-blind, placebo-controlled trial (PHOENIX 1). Lancet 2008; 371:1665–1674.
- Papp KA, Langley RG, Lebwohl M, et al; PHOENIX 2 study investigators. Efficacy and safety of ustekinumab, a human interleukin-12/23 monoclonal antibody, in patients with psoriasis: 52-week results from a randomised, double-blind, placebo-controlled trial (PHOENIX 2). Lancet 2008; 371:1675–1684.
- Leonardi CL, Gordon KB. New and emerging therapies in psoriasis. Semin Cut Med Surg 2014; 33(suppl 2):S37–S41.
- Langley RG, Elewski BE, Lebwohl, et al for the ERASURE and FIXTURE Study Groups. Secukinumab in plaque psorisis—results of two phase 3 trials. N Engl J Med 2014; 371:326–338.
- Leonardi C, Matheson R, Zachariae C. Anti-interleukin-17 monoclonal antibody ixekizumab in chronic plaque psoriasis. N Engl J Med 2012; 366:1190–1199.
- Papp KA, Leonardi C, Menter A, et al. Brodalumab, an anti-interleukin-17-receptor antibody for psoriasis. N Engl J Med 2012; 366:1181–1189.
- McInnes IB, Sieper J, Braun J, et al. Efficacy and safety of secukinumab, a fully human anti-interleukin-17A monoclonal antibody, in patients with moderate-to-severe psoriatic arthritis: a 24-week, randomised, double-blind, placebo-controlled, phase II proof-of-concept trial. Ann Rheum Dis 2014; 73:349–356.
- Mease PJ, Genovese MC, Greenwald MW, et al. Brodalumab, an anti-IL17RA monoclonal antibody, in psoriatic arthritis. N Engl J Med 2014; 370:2295–2306.
- Girotti MR, Saturno G, Lorigan P, Marais R. No longer an untreatable disease: how targeted and immunotherapies have changed the management of melanoma patients. Molec Oncol 2014, 8:1140–1158.
- Saranga-Perry V, Ambe C, Zager JS, Kudchadkar RR. Recent developments in the medical and surgical treatment of melanoma. CA Canc J Clin 2014; 64:171–185.
- Shah DJ, Dronca RS. Latest advances in chemotherapeutic, targeted, and immune approaches in the treatment of metastatic melanoma. Mayo Clin Proc 2014; 89:504–519.
- Davies H, Ignell GR, Cox C, et al. Mutations of the BRAF gene in human cancer. Nature 2002; 417:949–954.
- Flaherty KT, Puzanov I, Kim KB, et al. Inhibition of mutated, activated BRAF in metastatic melanoma. N Engl J Med 2010; 363:809–819.
- Chapman PB, Hauschild A, Robert C. Improved survival with vemurafenib in melanoma with BRAF V600E mutation. N Engl J Med 2011; 364:2507–2516.
- Hauschild A, Grob JJ, Demidov LV, et al. Dabrafenib in BRAF-mutated metastatic melanoma: a multicentre, open-label, phase 3 randomised controlled trial. Lancet 2012; 380:358–365.
- Flaherty KT, Robert C, Hersey P, et al. Improved survival with MEK inhibition in BRAF-mutated melanoma. N Engl J Med 2012; 367:107–114.
- Larkin J, Ascierto PA, Dreno B, et al. Combined vemurafenib and cobimetinib in BRAF-mutated melanoma. N Engl J Med 2014; 371:1867–1876.
- Long GV, Stroyakovskiy D, Gogas H, et al. Combined BRAF and MEK inhibition versus BRAF inhibition alone in melanoma. N Eng J Med 2014; 371:1877–1888.
- Hodi FS, O’Day SJ, McDermott DF, Weber RW. Improved survival with ipilimumab in patients with metastatic melanoma. N Engl J Med 2010; 363:711–723.
- Robert C, Long GV, Brady B, et al. Nivolumab in previously untreated melanoma without BRAF mutation. N Engl J Med 2015; 372:320–330.
- Robert C, Ribas A, Wolchok JD, et al. Anti-programmed-death-receptor-1 treatment with pembrolizumab in ipilimumab-refractory advanced melanoma: a randomised dose-comparison cohort of a phase 1 trial. Lancet 2014; 384:1109–1117.
- Wolchok JD, Kluger H, Callahan MK, et al. Nivolumab plus ipilimumab in advanced melanoma. N Engl J Med 2013; 369:122–133.
- Lyons TG, O’Kane GM, Kelly CM. Efficacy and safety of vismodegib: a new therapeutic agent in the treatment of basal cell carcinoma. Expert Opin Drug Saf 2014; 13:1125–1132.
- McCusker M, Basset-Sequin N, Dummer R, et al. Metastatic basal cell carcinoma: prognosis dependent on anatomic site and spread of disease. Eur J Cancer 2014; 50:774–783.
- Hahn H, Wicking C, Zaphiropoulous PG, et al. Mutations of the human homolog of Drosophila patched in the nevoid basal cell carcinoma syndrome. Cell 1996; 85:841–851.
- Aszterbaum M, Rothman A, Johnson RL, et al. Identification of mutations in the human PATCHED gene in sporadic basal cell carcinomas and in patients with the basal cell nevus syndrome. J Invest Dermatol 1998; 110:885–888.
- Ingham PW, Placzek M. Orchestrating ontogenesis: variations on a theme by sonic hedgehog. Nat Rev Genet 2006; 7:841–850.
- Robarge KD, Brunton SA, Castanedo GM, et al. GDC-0449-a potent inhibitor of the hedgehog pathway. Bioorg Med Chem Lett 2009; 19:5576–5581.
- Proctor AE, Thompson LA, O’Bryant CL. Vismodegib: an inhibitor of the Hedgehog signaling pathway in the treatment of basal cell carcinoma. Ann Pharmacother 2014; 48:99–106.
- Dessinioti C, Plaka M, Stratigos AJ. Vismodegib for the treatment of basal cell carcinoma: results and implications of the ERIVANCE BCC trial. Future Oncol 2014; 10:927–936.
- Sekulic A, Migden MR, Oro AE, et al. Efficacy and safety of vismodegib in advanced basal-cell carcinoma. N Engl J Med 2012; 366:2171–2179.
- Tang JY, Mackay-Wiggan JM, Aszterbaum M, et al. Inhibiting the hedgehog pathway in patients with basal-cell nevus syndrome. N Engl J Med 2012; 366:2180–2188.
- Brinkhuizen T, Reinders MG, van Geel M, et al. Acquired resistance to the Hedgehog pathway inhibitor vismodegib due to smoothened mutations in treatment of locally advanced basal cell carcinoma. J Am Acad Dermatol 2014; 71:1005–1008.
- Ally MS, Aasi S, Wysong A, et al. An investigator-initiated open-label clinical trial of vismodegib as a neoadjuvant to surgery for high-risk basal cell carcinoma. J Am Acad Dermatol 2014; 71:904–911.
- Rapp SR, Feldman SR, Exum ML, Fleischer AB Jr, Reboussin DM. Psoriasis causes as much disability as other major medical diseases. J Am Acad Dermatol 1999; 41:401–407.
- Gelfand JM, Niemann AL, Shin DB, Wang X, Margolis DJ, Troxel AB. Risk of myocardial infarction in patients with psoriasis. JAMA 2006; 296:1735–1741.
- Lynde CW, Poulin Y, Vender R, Bourcier M, Khalil S. Interleukin 17A: toward a new understanding of psoriasis pathogenesis. J Am Acad Dermatol 2014; 71:141–150.
- Tracey D, Klareskog L, Sasso EH, Salfeld JG, Tak PP. Tumor necrosis factor antagonist mechanisms of action: a comprehensive review. Pharmacol Ther 2008; 117:244–279.
- Nestle FL, Kaplan DH, Barker J. Psoriasis. N Engl J Med 2009; 361:496–509.
- Mentor A, Tyring SK, Gordon K, et al. Adalimumab therapy for moderate to severe psoriasis: a randomized, controlled phase III trial. J Am Acad Dermatol 2007; 58:106–115.
- Leonardi CL, Powers JL, Matheson RT, et al. Etanercept as monotherapy in patients with psoriasis. N Engl J Med 2003; 349:2014–2022.
- Reich K, Nestle FO, Papp K, et al. Infliximab induction and maintenance therapy for moderate-to-severe psoriasis: a phase III, multicentre, double-blind trial. Lancet 2005; 366:1367–1374.
- Leonardi CL, Kimball AB, Papp KA, et al. Efficacy and safety of ustekinumab, a human interleukin-12/23 monoclonal antibody, in patients with psoriasis: 76-week results from a randomised, double-blind, placebo-controlled trial (PHOENIX 1). Lancet 2008; 371:1665–1674.
- Papp KA, Langley RG, Lebwohl M, et al; PHOENIX 2 study investigators. Efficacy and safety of ustekinumab, a human interleukin-12/23 monoclonal antibody, in patients with psoriasis: 52-week results from a randomised, double-blind, placebo-controlled trial (PHOENIX 2). Lancet 2008; 371:1675–1684.
- Leonardi CL, Gordon KB. New and emerging therapies in psoriasis. Semin Cut Med Surg 2014; 33(suppl 2):S37–S41.
- Langley RG, Elewski BE, Lebwohl, et al for the ERASURE and FIXTURE Study Groups. Secukinumab in plaque psorisis—results of two phase 3 trials. N Engl J Med 2014; 371:326–338.
- Leonardi C, Matheson R, Zachariae C. Anti-interleukin-17 monoclonal antibody ixekizumab in chronic plaque psoriasis. N Engl J Med 2012; 366:1190–1199.
- Papp KA, Leonardi C, Menter A, et al. Brodalumab, an anti-interleukin-17-receptor antibody for psoriasis. N Engl J Med 2012; 366:1181–1189.
- McInnes IB, Sieper J, Braun J, et al. Efficacy and safety of secukinumab, a fully human anti-interleukin-17A monoclonal antibody, in patients with moderate-to-severe psoriatic arthritis: a 24-week, randomised, double-blind, placebo-controlled, phase II proof-of-concept trial. Ann Rheum Dis 2014; 73:349–356.
- Mease PJ, Genovese MC, Greenwald MW, et al. Brodalumab, an anti-IL17RA monoclonal antibody, in psoriatic arthritis. N Engl J Med 2014; 370:2295–2306.
- Girotti MR, Saturno G, Lorigan P, Marais R. No longer an untreatable disease: how targeted and immunotherapies have changed the management of melanoma patients. Molec Oncol 2014, 8:1140–1158.
- Saranga-Perry V, Ambe C, Zager JS, Kudchadkar RR. Recent developments in the medical and surgical treatment of melanoma. CA Canc J Clin 2014; 64:171–185.
- Shah DJ, Dronca RS. Latest advances in chemotherapeutic, targeted, and immune approaches in the treatment of metastatic melanoma. Mayo Clin Proc 2014; 89:504–519.
- Davies H, Ignell GR, Cox C, et al. Mutations of the BRAF gene in human cancer. Nature 2002; 417:949–954.
- Flaherty KT, Puzanov I, Kim KB, et al. Inhibition of mutated, activated BRAF in metastatic melanoma. N Engl J Med 2010; 363:809–819.
- Chapman PB, Hauschild A, Robert C. Improved survival with vemurafenib in melanoma with BRAF V600E mutation. N Engl J Med 2011; 364:2507–2516.
- Hauschild A, Grob JJ, Demidov LV, et al. Dabrafenib in BRAF-mutated metastatic melanoma: a multicentre, open-label, phase 3 randomised controlled trial. Lancet 2012; 380:358–365.
- Flaherty KT, Robert C, Hersey P, et al. Improved survival with MEK inhibition in BRAF-mutated melanoma. N Engl J Med 2012; 367:107–114.
- Larkin J, Ascierto PA, Dreno B, et al. Combined vemurafenib and cobimetinib in BRAF-mutated melanoma. N Engl J Med 2014; 371:1867–1876.
- Long GV, Stroyakovskiy D, Gogas H, et al. Combined BRAF and MEK inhibition versus BRAF inhibition alone in melanoma. N Eng J Med 2014; 371:1877–1888.
- Hodi FS, O’Day SJ, McDermott DF, Weber RW. Improved survival with ipilimumab in patients with metastatic melanoma. N Engl J Med 2010; 363:711–723.
- Robert C, Long GV, Brady B, et al. Nivolumab in previously untreated melanoma without BRAF mutation. N Engl J Med 2015; 372:320–330.
- Robert C, Ribas A, Wolchok JD, et al. Anti-programmed-death-receptor-1 treatment with pembrolizumab in ipilimumab-refractory advanced melanoma: a randomised dose-comparison cohort of a phase 1 trial. Lancet 2014; 384:1109–1117.
- Wolchok JD, Kluger H, Callahan MK, et al. Nivolumab plus ipilimumab in advanced melanoma. N Engl J Med 2013; 369:122–133.
KEY POINTS
- Vismodegib, an inhibitor of the “hedgehog” pathway, dramatically shrinks basal cell carcinomas, but resistance and adverse effects remain troublesome. Using it to shrink tumors to operable size may be its best future role.
- Th-17 cells and interleukin 17 are now thought to play central roles in the pathogenesis of psoriasis. Clinical trials of new drugs that block interleukin 17 show striking improvement in skin manifestations with few side effects. Benefits in psoriatic arthritis have not yet been shown.
- About half of patients with melanoma harbor BRAF mutations, and new treatments that target this pathway have improved survival rates. For melanoma not involving BRAF mutations, a better understanding of how tumors evade immune control has led to improved immunotherapies. These targeted medications mark the first major advancements in metastatic melanoma treatment in decades.
Stand by me! Reducing the risk of injurious falls in older adults
Falls and fall-related injuries are common in older adults Every year, 30% of those who are 65 and older fall,1 and the consequences are potentially serious. Falls are the primary cause of hip fracture, which requires an extensive period of rehabilitation. However, rehabilitation does not always restore the older adult to his or her preinjury functional state. In fact, at 6 to 12 months after a hip fracture, 22% to 75% of elderly patients have not recovered their prefracture ambulatory or functional status.2
Falls are also the most common cause of traumatic brain injury in older adults,3 often resulting in long-term cognitive and emotional problems and pain that compromise quality of life. Falls can be fatal and in fact are the leading cause of death from injury in older adults.4
Practitioners can reduce fall-related injury5 and potentially improve quality of life by screening older adults yearly, performing a focused history and examination when necessary, and implementing evidence-based interventions.
RISK FACTORS
A single identifiable factor may account for only a small portion of the fall risk. Falls in older adults are, in general, multifactorial and can be caused by medical conditions (eg, sarcopenia, particularly of the lower limbs, vision loss, urinary incontinence, neuropathies), cognitive impairment, medications such as psychotropic drugs, and home hazards such as area rugs, extension cords, and dimly lit stairways.
The strongest predictors of falls are a recent fall and the presence of a gait or balance disorder.6
SCREENING TESTS
Joint guidelines from the American Geriatrics Society and British Geriatrics Society,7 published in 2011, recommend that practitioners screen older adults yearly for fall risk by asking two questions: “Have you fallen in the past year?” and “Are you having difficulty with gait or balance?” A negative response to both questions suggests a low risk of falling in the near future. Patients with two or more falls, a balance or gait problem (subjective or objective), or history of a fall requiring medical attention should undergo a focused history and physical examination plus a multifactorial risk assessment.
A report of one fall without injury should prompt a simple office-based test of balance. Examples of tests include the Get Up and Go, the Timed Up and Go, and the One-Legged Stance (the Unipedal Stance).
In the Get Up and Go test, patients sit comfortably in a chair with a straight back. They rise from the chair, stand still, walk a short distance (about 3 meters), turn around, walk back to the chair, and sit down.8 The clinician notes any deviation from a confident, smooth performance.
In the Timed Up and Go test, the clinician records the time it takes for the patient to rise from a hardback chair, walk 10 feet (3 meters), turn, return to the chair, and sit down.9 Most older adults complete this test in less than 10 seconds. Taking longer than 14 seconds is associated with a high risk of falls.10
For the One-Legged Stance test, the clinician asks the patient to stand on one leg. A patient without significant balance issues is able to stand for at least 5 seconds.11
Figure 1 summarizes the approach for a community-dwelling patient who presents to the outpatient setting. A complete multifactorial risk assessment may require a dedicated appointment or referral to a specialist such as a geriatrician, physiatrist, or neurologist.
WHAT INFORMATION DOES A FOCUSED HISTORY INCLUDE?
The fall-focused history includes:
A detailed description of the circumstances of the fall or falls, symptoms (such as dizziness), and injuries or other consequences of the fall.7
A medication review. Table 1 includes commonly prescribed drug classes associated with increased fall risk.12 Be especially vigilant for eyedrops used to treat glaucoma (some can potentiate bradycardia) and for psychotropic drugs.
Drug regimens with a high psychotropic burden can be identified with the Drug Burden Index13 or the Anticholinergic Risk Scale,14 but these scales are cumbersome and are usually used only as part of a research study. The updated Beers criteria15 and use of a structured medication review such as the START and STOPP algorithms16 can help prune unnecessary, inappropriate, and high-risk medications such as:
- Selective serotonin reuptake inhibitors in the absence of current major depression. These drugs increase the risk of falls and decrease bone density.17
- Proton pump inhibitors in the absence of a true indication for this drug class to treat reflux. Drugs in this class reduce bone density and increase the risk of hip fracture after 1 year of continuous use18
- Cholinesterase inhibitors in the absence of demonstrated benefit to dementia symptoms for the particular patient. Drugs in this class are associated with falls, hip fracture, bradycardia, and possible need for pacemaker placement.19
Review of activities of daily living (ADLs). A functional assessment of the patient’s ability to complete ADLs helps identify targets for therapy. Assess whether the patient is afraid of falling and, if so, what impact this fear has on ADLs. This can help determine whether the fear protects the patient from performing risky tasks, or harms the patient by contributing to deconditioning.
Medical conditions. Consider chronic conditions that can impair mobility and increase fall risk. These include urinary incontinence, cognitive impairment (eg, dementia), neuropathy, degenerative neurologic conditions such as Parkinson disease, and degenerative arthritis. Osteoporosis increases the risk of fracture in a fall. Vitamin D deficiency increases both fall and fracture risk.20
PHYSICAL EXAMINATION FINDINGS
Assess the patient’s vision, proprioception, reflexes, and cortical, extrapyramidal, and cerebellar function.7
Perform a detailed assessment of the patient’s gait, balance, and mobility. Assess the lower extremities for joint and nerve function, muscle strength, and range of motion.7 The use of brain imaging, if appropriate, is guided by gait abnormalities. Unexpected findings such as neuropathy may require referrals for further evaluation.
Examine the patient’s feet and footwear for signs of poor fit and for styles that may be inappropriate for someone at risk of falling, such as high heels.
Conduct a cardiovascular examination. In addition to assessing heart rate and rhythm and checking for heart murmurs, evaluate the patient for postural changes in heart rate and blood pressure. Wait at least 2 minutes before asking the patient to change position from supine to seated and from seated to standing. A longer interval (3 to 5 minutes) can be used depending on the patient’s history. For example, an older adult reporting a syncopal episode standing by the kitchen sink may need a longer standing interval prior to blood pressure measurement than an older adult who falls right after standing up from a chair.
If there is a strong suspicion that an orthostatic condition contributed to a fall but it is not possible to elicit orthostasis in the office, it may be necessary to refer the patient for tilt-table testing. If the circumstances suggest that pressure along the neck, or turning the neck, contributed to a fall, referral for carotid sinus stimulation may be appropriate. If there is a concern that a brady- or tachyarrhythmia contributed to the fall, a referral for 24- or 48-hour Holter monitoring or a 30-day loop monitor may be indicated.
Assess the patient’s mental status. Cognitive impairment itself is an independent predictor of falls7 because it can reduce processing speed and impair executive function.21 Executive dysfunction may contribute to falls by causing problems with multitasking, drug compliance, and judgment. The presence and severity of cognitive impairment may affect recommendation options (see below), so the assessment should include a screening test. Consider using the Mini-Cog, which requires the patient to recall three words and draw an analog clock (Figure 2).22
Some cognitive screening tests validated for use in the general older population include the General Practitioner Assessment of Cognition and the Memory Impairment Screen.23 More involved cognitive testing such as the Folstein Mini-Mental State Examination, Montreal Cognitive Assessment, and the Saint Louis University Mental Status Examination are routinely performed in a geriatric or neurologic setting. The Folstein is a proprietary test; the other two are not.
Conditions such as circulatory disease, chronic obstructive pulmonary disease, depression, and arthritis are associated with a higher risk of falling, even with adjustment for drug use and other potential confounding factors.24
A brief mood assessment is part of the multifactorial assessment because mood disorders in older adults can lead to deconditioning, drug noncompliance, and other conditions that lead to falls and fall-related injuries. Options for screening include the Geriatric Depression Scale (15 or 30 questions) and the Patient Health Questionnaire (the PHQ-2 or the PHQ-9).7
WHAT ARE THE EVIDENCE-BASED INTERVENTIONS?
In general, interventions are chosen according to the risks identified by the assessment; multiple interventions are usually necessary. It is ineffective to identify risk factors without providing intervention.25
Specific interventions with recommendation levels A and B are listed in Table 2.7 Level A interventions are specifically supported by strong evidence and should be recommended. Of note, although vitamin D3 may not be bioequivalent to vitamin D2, studies in older adults have not consistently found a clinically different outcome, and either may be supplemented in the community-dwelling elderly. Except for vitamin D, these interventions target community-dwelling older adults who are cognitively intact.
Home assessments are effective in high-risk patients, such as those with poor vision and those who were recently hospitalized. The goal is to improve safety, particularly during patient transfers, with education and training provided by an occupational or physical therapist or other geriatric specialist. The benefit of home assessment and environmental modification is greater when combined with other strategies and in general should not be implemented alone.
Exercise is an important intervention. The number needed to treat (NNT) to prevent one fall in older people over the course of at least 12 weeks is 16.26 This compares favorably with interventions that are commonly used in the general population, such as aspirin therapy as secondary prevention for cardiovascular disease (NNT for 1 year = 50)27 and statin therapy to prevent one death from a cardiovascular event over 5 years in people with known heart disease (NNT = 83).28
Exercise recommendations should be customized to the patient. The amount and type of exercise depends on the patient’s baseline physical activity, medication use including antiplatelet and anticoagulant therapy, home environment, cardiac and pulmonary reserve, vision and hearing deficits, and comorbidities including neuropathy and arthritis.
The well-known risks associated with exercise include myocardial infarction and cardiac arrest, as well as falls and fractures. However, the benefits extend beyond fall risk and include improvements in physical function, glycemic control, cardiopulmonary reserve, bone density, arthritic pain, mood, and cognition. Exercise can also help manage weight, reduce sarcopenia, and increase opportunities for socialization. In most positive trials, the exercise interventions lasted longer than 12 weeks, had variable intensity, and occurred 1 to 3 times per week.
The American College of Sports Medicine recommends that older adults perform aerobic exercise 3 to 5 times per week, 20 to 60 minutes per session (the lower ranges are for frail elderly patients).29 It also recommends resistance training 2 to 4 days per week, 20 to 45 minutes per session, depending on the patient’s level of frailty and conditioning.30 Most older adults do not exercise enough.
Interventions listed at the bottom of Table 2 do not, in general, have enough evidence to support or discourage their use; these are level C recommendations. However, these interventions may be considered for certain individuals. For example, older adults with diabetic neuropathy are often unaware of their foot position when they walk. Additionally, those with diabetic neuropathy may have slower generation of ankle and knee strength compared with age-matched controls. These patients may benefit from targeted physical therapy to strengthen ankle and knee extensors and to retrain stride and speed to improve both gait and safety awareness.
Patients who wear shoes that fit poorly, have high heels, or are not laced or buckled have a higher risk of falls.31 Consider recommending footwear that has a firm, low, rubber heel and a sole with a large surface contact area, which may help reduce the risk of falling.32 Advise patients to wear shoes when they are at home and to avoid using slippers and going barefoot.33
Cataract surgery, another level C intervention, is associated with fewer fall-related injuries, particularly hip fracture.34 Noncataract vision interventions (such as exchanging progressive or bifocal lenses for single-lens glasses) may be effective in select patients if distorted vision in the lower fields of view increases the risk of falling, particularly outdoors.35
INTERVENTIONS FOR SPECIAL POPULATIONS
Falls occur more frequently in mobile residents of long-term care facilities than in community-dwelling adults.7 Institutional residents are older and more frail, have more cognitive impairment, and are prescribed more medications. Half of long-term care residents fall at least once a year.7
The data support giving combined calcium and vitamin D supplementation to older adults in long-term care facilities to reduce fracture rates.36 The NNT to prevent one hip fracture is about 111.37 Hip protectors in this setting may reduce the risk of a hip fracture but also may increase the risk of a pelvic fracture. They do not alter the risk of falling.38
Collaborative interventions can help reduce the fall risk in older adults in the nursing home.39 Input from medical, psychosocial, nursing, podiatric, dietary, and therapy services can be solicited and incorporated into an individualized fall prevention program. The program can also include modifications in the environment to improve safety and reduce fall risk.
The benefits of exercise in reducing injurious falls in long-term care is less clear than in the community, likely because of the heterogeneity of both the long-term care population and the studied interventions. Exercise has other benefits, however. It maintains a person’s ability to complete ADLs, improves mood, reduces hyperglycemia, and improves quality of life. Some studies have found a greater risk of falling with exercise therapy as independence increased.40 However, a meta-analysis in 2013 found that exercise interventions, ranging from 3 to 24 months and consisting mainly of balance and resistance training, reduced the risk of falls by 23%.41 Mixing several types of exercises was helpful. Studies of a longer duration with exercise sessions at least 2 to 3 times per week demonstrated the most benefit.41 There was no statistically significant reduction in fracture risk in this meta-analysis,41 although, possibly, more participants would have been needed for a longer period to demonstrate a benefit. Additionally, no study combined osteoporosis treatment with exercise interventions.
WHAT EVIDENCE EXISTS FOR PATIENTS WITH COGNITIVE IMPAIRMENT?
Currently, there are no specific evidence-based recommendations for fall prevention in community-dwelling older adults with cognitive impairment and dementia.7 Cognitively impaired adults are typically excluded from community studies of fall prevention. The one study that specifically investigated community-dwelling adults with cognitive impairment was not able to demonstrate a fall reduction with multifactorial intervention.42
PREVENTING FALLS IN ELDERLY PATIENTS WHO RECENTLY HAD A STROKE
Falls are common in patients who have had a cerebrovascular event. Up to 7% of patients fall in the first week after a stroke. In the year after a stroke, 55% to 75% of patients experience a fall.43 Falls account for the most common medical complication after a stroke.44
Several small studies found that vitamin D supplementation after a stroke reduced both the rate of falls and the number of people who fall.45 Additional interventions such as exercise, medication, and visual aids have been studied, but there is little evidence to support their use. Mobile patients who have lower-extremity hemiparesis after a stroke may develop osteoporosis in the affected limb, so evaluation and appropriate pharmacologic therapy may be considered.
- Tromp AM, Pluijm SM, Smit JH, Deeg DJ, Bouter LM, Lips P. Fall-risk screening test: a prospective study on predictors for falls in community-dwelling elderly. J Clin Epidemiol 2001; 54:837–844.
- Binder EF, Brown M, Sinacore DR, Steger-May K, Yarasheski KE, Schechtman KB. Effects of extended outpatient rehabilitation after hip fracture: a randomized controlled trial. JAMA 2004; 292:837–846.
- Sterling DA, O’Connor JA, Bonadies J. Geriatric falls: injury severity is high and disproportionate to mechanism. J Trauma 2001; 50:116–119.
- Centers for Disease Control and Prevention (CDC). Web-based Injury Statistics Query and Reporting System (WISQARSTM). www.cdc.gov/injury/wisqars. Accessed April 8, 2015.
- Tinetti ME, Baker DI, King M, et al. Effect of dissemination of evidence in reducing injuries from falls. N Engl J Med 2008; 359:252–261.
- Ganz DA, Bao Y, Shekelle PG, Rubenstein LZ. Will my patient fall? JAMA 2007; 297:77–86.
- Panel on Prevention of Falls in Older Persons, American Geriatrics Society and British Geriatrics Society. Summary of the Updated American Geriatrics Society/British Geriatrics Society clinical practice guideline for prevention of falls in older persons. J Am Geriatr Soc 2011; 59:148–157.
- Mathias S, Nayak US, Isaacs B. Balance in elderly patients: the “get-up and go” test. Arch Phys Med Rehabil 1986; 67:387–389.
- Podsiadlo D, Richardson S. The timed “Up & Go”: a test of basic functional mobility for frail elderly persons. J Am Geriatr Soc 1991; 39:142–148.
- Shumway-Cook A, Brauer S, Woollacott M. Predicting the probability for falls in community-dwelling older adults using the Timed Up & Go Test. Phys Ther 2000; 80:896–903.
- Springer BA, Marin R, Cyhan T, Roberts H, Gill NW. Normative values for the unipedal stance test with eyes open and closed. J Geriatr Phys Ther 2007; 30:8–15.
- Leipzig RM, Cumming RG, Tinetti ME. Drugs and falls in older people: a systematic review and meta-analysis: I. Psychotropic drugs. J Am Geriatr Soc 1999; 47:30–39.
- Hilmer SN, Mager DE, Simonsick EM, et al. A drug burden index to define the functional burden of medications in older people. Arch Intern Med 2007; 167:781–787.
- Rudolph JL, Salow MJ, Angelini MC, McGlinchey RE. The anticholinergic risk scale and anticholinergic adverse effects in older persons. Arch Intern Med 2008; 168:508–513.
- American Geriatrics Society 2012 Beers Criteria Update Expert Panel. American Geriatrics Society updated Beers Criteria for potentially inappropriate medication use in older adults. J Am Geriatr Soc 2012; 60:616–631.
- Gallagher P, Ryan C, Byrne S, Kennedy J, O’Mahony D. STOPP (Screening Tool of Older Person’s Prescriptions) and START (Screening Tool to Alert doctors to Right Treatment). Consensus validation. Int J Clin Pharmacol Ther 2008; 46:72–83.
- Sterke CS, Ziere G, van Beeck EF, Looman CW, van der Cammen TJ. Dose-response relationship between selective serotonin re-uptake inhibitors and injurious falls: a study in nursing home residents with dementia. Br J Clin Pharmacol 2012; 73:812–820.
- Khalili H, Huang ES, Jacobson BC, Camargo CA Jr, Feskanich D, Chan AT. Use of proton pump inhibitors and risk of hip fracture in relation to dietary and lifestyle factors: a prospective cohort study. BMJ 2012; 344:e372.
- Gill SS, Anderson GM, Fischer HD, et al. Syncope and its consequences in patients with dementia receiving cholinesterase inhibitors: a population-based cohort study. Arch Intern Med 2009; 169:867–873.
- Janssen HC, Samson MM, Verhaar HJ. Vitamin D deficiency, muscle function, and falls in elderly people. Am J Clin Nutr 2002; 75:611–615.
- Muir SW, Gopaul K, Montero Odasso MM. The role of cognitive impairment in fall risk among older adults: a systematic review and meta-analysis. Age Ageing 2012; 41:299–308.
- Borson S, Scanlan J, Brush M, Vitaliano P, Dokmak A. The Mini-Cog: a cognitive ‘vital signs’ measure for dementia screening in multi-lingual elderly. Int J Geriatr Psychiatry 2000; 15:1021–1027.
- Cordell CB, Borson S, Boustani M, et al; Medicare Detection of Cognitive Impairment Workgroup. Alzheimer’s Association recommendations for operationalizing the detection of cognitive impairment during the Medicare Annual Wellness Visit in a primary care setting. Alzheimers Dement 2013; 9:141–150.
- Lawlor DA, Patel R, Ebrahim S. Association between falls in elderly women and chronic diseases and drug use: cross sectional study. BMJ 2003; 327:712–717.
- Day L, Fildes B, Gordon I, Fitzharris M, Flamer H, Lord S. Randomised factorial trial of falls prevention among older people living in their own homes. BMJ 2002; 325:128.
- Chang JT, Morton SC, Rubenstein LZ, et al. Interventions for the prevention of falls in older adults: systematic review and meta-analysis of randomised clinical trials. BMJ 2004; 328:680.
- Antithrombotic Trialists’ (ATT) Collaboration; Baigent C, Blackwell L, Collins R, et al. Aspirin in the primary and secondary prevention of vascular disease: collaborative meta-analysis of individual participant data from randomised trials. Lancet 2009; 373:1849–1860.
- Baigent C, Keech A, Kearney PM, et al; Cholesterol Treatment Trialists’ (CTT) Collaborators. Efficacy and safety of cholesterol-lowering treatment: prospective meta-analysis of data from 90,056 participants in 14 randomised trials of statins. Lancet 2005; 366:1267–1278.
- Mazzeo RS; American College of Sports Medicine (ACSM). Exercise and the older adult. ACSM Current Comment. www.acsm.org/docs/current-comments/exerciseandtheolderadult.pdf. Accessed April 8, 2015.
- Willoughby DS; American College of Sports Medicine (ACSM). Resistance training and the older adult. ACSM Current Comment. www.acsm.org/docs/current-comments/resistancetrainingandtheoa.pdf. Accessed April 8, 2015.
- Tencer AF, Koepsell TD, Wolf ME, et al. Biomechanical properties of shoes and risk of falls in older adults. J Am Geriatr Soc 2004; 52:1840–1846.
- Lord SR, Bashford GM. Shoe characteristics and balance in older women. J Am Geriatr Soc 1996; 44:429–433.
- Kelsey JL, Procter-Gray E, Nguyen US, Li W, Kiel DP, Hannan MT. Footwear and falls in the home among older individuals in the MOBILIZE Boston Study. Footwear Sci 2010; 2:123–129.
- Tseng VL, Yu F, Lum F, Coleman AL. Risk of fractures following cataract surgery in Medicare beneficiaries. JAMA 2012; 308:493–501.
- Cumming RG, Ivers R, Clemson L, et al. Improving vision to prevent falls in frail older people: a randomized trial. J Am Geriatr Soc 2007; 55:175–181.
- Bischoff-Ferrari HA, Dawson-Hughes B, Willett WC, et al. Effect of vitamin D on falls: a meta-analysis. JAMA 2004; 291:1999–2006.
- Avenell A, Mak JCS, O’Connell D. Vitamin D and vitamin D analogues for preventing fractures in postmenopausal women and older men. Cochrane Database Syst Rev 2014; 4:CD000227.
- Santesso N, Carrasco-Labra A, Brignardello-Petersen R. Hip protectors for preventing hip fractures in older people. Cochrane Database Syst Rev 2014; 3:CD001255.
- Messinger-Rapport B, Dumas LG. Falls in the nursing home: a collaborative approach. Nurs Clin North Am 2009; 44:187–195.
- Faber MJ, Bosscher RJ, Chin A, Paw MJ, van Wieringen PC. Effects of exercise programs on falls and mobility in frail and pre-frail older adults: a multicenter randomized controlled trial. Arch Phys Med Rehabil 2006; 87:885–896.
- Silva RB, Eslick GD, Duque G. Exercise for falls and fracture prevention in long term care facilities: a systematic review and meta-analysis. J Am Med Dir Assoc 2013; 14:685–689.e2.
- Shaw FE, Bond J, Richardson DA, et al. Multifactorial intervention after a fall in older people with cognitive impairment and dementia presenting to the accident and emergency department: randomised controlled trial. BMJ 2003; 326:73.
- Ashburn A, Hyndman D, Pickering R, Yardley L, Harris S. Predicting people with stroke at risk of falls. Age Ageing 2008; 37:270–276.
- Davenport RJ, Dennis MS, Wellwood I, Warlow CP. Complications after acute stroke. Stroke 1996; 27:415–420.
- Verheyden GS, Weerdesteyn V, Pickering RM, et al. Interventions for preventing falls in people after stroke. Cochrane Database Syst Rev 2013; 5:CD008728.
Falls and fall-related injuries are common in older adults Every year, 30% of those who are 65 and older fall,1 and the consequences are potentially serious. Falls are the primary cause of hip fracture, which requires an extensive period of rehabilitation. However, rehabilitation does not always restore the older adult to his or her preinjury functional state. In fact, at 6 to 12 months after a hip fracture, 22% to 75% of elderly patients have not recovered their prefracture ambulatory or functional status.2
Falls are also the most common cause of traumatic brain injury in older adults,3 often resulting in long-term cognitive and emotional problems and pain that compromise quality of life. Falls can be fatal and in fact are the leading cause of death from injury in older adults.4
Practitioners can reduce fall-related injury5 and potentially improve quality of life by screening older adults yearly, performing a focused history and examination when necessary, and implementing evidence-based interventions.
RISK FACTORS
A single identifiable factor may account for only a small portion of the fall risk. Falls in older adults are, in general, multifactorial and can be caused by medical conditions (eg, sarcopenia, particularly of the lower limbs, vision loss, urinary incontinence, neuropathies), cognitive impairment, medications such as psychotropic drugs, and home hazards such as area rugs, extension cords, and dimly lit stairways.
The strongest predictors of falls are a recent fall and the presence of a gait or balance disorder.6
SCREENING TESTS
Joint guidelines from the American Geriatrics Society and British Geriatrics Society,7 published in 2011, recommend that practitioners screen older adults yearly for fall risk by asking two questions: “Have you fallen in the past year?” and “Are you having difficulty with gait or balance?” A negative response to both questions suggests a low risk of falling in the near future. Patients with two or more falls, a balance or gait problem (subjective or objective), or history of a fall requiring medical attention should undergo a focused history and physical examination plus a multifactorial risk assessment.
A report of one fall without injury should prompt a simple office-based test of balance. Examples of tests include the Get Up and Go, the Timed Up and Go, and the One-Legged Stance (the Unipedal Stance).
In the Get Up and Go test, patients sit comfortably in a chair with a straight back. They rise from the chair, stand still, walk a short distance (about 3 meters), turn around, walk back to the chair, and sit down.8 The clinician notes any deviation from a confident, smooth performance.
In the Timed Up and Go test, the clinician records the time it takes for the patient to rise from a hardback chair, walk 10 feet (3 meters), turn, return to the chair, and sit down.9 Most older adults complete this test in less than 10 seconds. Taking longer than 14 seconds is associated with a high risk of falls.10
For the One-Legged Stance test, the clinician asks the patient to stand on one leg. A patient without significant balance issues is able to stand for at least 5 seconds.11
Figure 1 summarizes the approach for a community-dwelling patient who presents to the outpatient setting. A complete multifactorial risk assessment may require a dedicated appointment or referral to a specialist such as a geriatrician, physiatrist, or neurologist.
WHAT INFORMATION DOES A FOCUSED HISTORY INCLUDE?
The fall-focused history includes:
A detailed description of the circumstances of the fall or falls, symptoms (such as dizziness), and injuries or other consequences of the fall.7
A medication review. Table 1 includes commonly prescribed drug classes associated with increased fall risk.12 Be especially vigilant for eyedrops used to treat glaucoma (some can potentiate bradycardia) and for psychotropic drugs.
Drug regimens with a high psychotropic burden can be identified with the Drug Burden Index13 or the Anticholinergic Risk Scale,14 but these scales are cumbersome and are usually used only as part of a research study. The updated Beers criteria15 and use of a structured medication review such as the START and STOPP algorithms16 can help prune unnecessary, inappropriate, and high-risk medications such as:
- Selective serotonin reuptake inhibitors in the absence of current major depression. These drugs increase the risk of falls and decrease bone density.17
- Proton pump inhibitors in the absence of a true indication for this drug class to treat reflux. Drugs in this class reduce bone density and increase the risk of hip fracture after 1 year of continuous use18
- Cholinesterase inhibitors in the absence of demonstrated benefit to dementia symptoms for the particular patient. Drugs in this class are associated with falls, hip fracture, bradycardia, and possible need for pacemaker placement.19
Review of activities of daily living (ADLs). A functional assessment of the patient’s ability to complete ADLs helps identify targets for therapy. Assess whether the patient is afraid of falling and, if so, what impact this fear has on ADLs. This can help determine whether the fear protects the patient from performing risky tasks, or harms the patient by contributing to deconditioning.
Medical conditions. Consider chronic conditions that can impair mobility and increase fall risk. These include urinary incontinence, cognitive impairment (eg, dementia), neuropathy, degenerative neurologic conditions such as Parkinson disease, and degenerative arthritis. Osteoporosis increases the risk of fracture in a fall. Vitamin D deficiency increases both fall and fracture risk.20
PHYSICAL EXAMINATION FINDINGS
Assess the patient’s vision, proprioception, reflexes, and cortical, extrapyramidal, and cerebellar function.7
Perform a detailed assessment of the patient’s gait, balance, and mobility. Assess the lower extremities for joint and nerve function, muscle strength, and range of motion.7 The use of brain imaging, if appropriate, is guided by gait abnormalities. Unexpected findings such as neuropathy may require referrals for further evaluation.
Examine the patient’s feet and footwear for signs of poor fit and for styles that may be inappropriate for someone at risk of falling, such as high heels.
Conduct a cardiovascular examination. In addition to assessing heart rate and rhythm and checking for heart murmurs, evaluate the patient for postural changes in heart rate and blood pressure. Wait at least 2 minutes before asking the patient to change position from supine to seated and from seated to standing. A longer interval (3 to 5 minutes) can be used depending on the patient’s history. For example, an older adult reporting a syncopal episode standing by the kitchen sink may need a longer standing interval prior to blood pressure measurement than an older adult who falls right after standing up from a chair.
If there is a strong suspicion that an orthostatic condition contributed to a fall but it is not possible to elicit orthostasis in the office, it may be necessary to refer the patient for tilt-table testing. If the circumstances suggest that pressure along the neck, or turning the neck, contributed to a fall, referral for carotid sinus stimulation may be appropriate. If there is a concern that a brady- or tachyarrhythmia contributed to the fall, a referral for 24- or 48-hour Holter monitoring or a 30-day loop monitor may be indicated.
Assess the patient’s mental status. Cognitive impairment itself is an independent predictor of falls7 because it can reduce processing speed and impair executive function.21 Executive dysfunction may contribute to falls by causing problems with multitasking, drug compliance, and judgment. The presence and severity of cognitive impairment may affect recommendation options (see below), so the assessment should include a screening test. Consider using the Mini-Cog, which requires the patient to recall three words and draw an analog clock (Figure 2).22
Some cognitive screening tests validated for use in the general older population include the General Practitioner Assessment of Cognition and the Memory Impairment Screen.23 More involved cognitive testing such as the Folstein Mini-Mental State Examination, Montreal Cognitive Assessment, and the Saint Louis University Mental Status Examination are routinely performed in a geriatric or neurologic setting. The Folstein is a proprietary test; the other two are not.
Conditions such as circulatory disease, chronic obstructive pulmonary disease, depression, and arthritis are associated with a higher risk of falling, even with adjustment for drug use and other potential confounding factors.24
A brief mood assessment is part of the multifactorial assessment because mood disorders in older adults can lead to deconditioning, drug noncompliance, and other conditions that lead to falls and fall-related injuries. Options for screening include the Geriatric Depression Scale (15 or 30 questions) and the Patient Health Questionnaire (the PHQ-2 or the PHQ-9).7
WHAT ARE THE EVIDENCE-BASED INTERVENTIONS?
In general, interventions are chosen according to the risks identified by the assessment; multiple interventions are usually necessary. It is ineffective to identify risk factors without providing intervention.25
Specific interventions with recommendation levels A and B are listed in Table 2.7 Level A interventions are specifically supported by strong evidence and should be recommended. Of note, although vitamin D3 may not be bioequivalent to vitamin D2, studies in older adults have not consistently found a clinically different outcome, and either may be supplemented in the community-dwelling elderly. Except for vitamin D, these interventions target community-dwelling older adults who are cognitively intact.
Home assessments are effective in high-risk patients, such as those with poor vision and those who were recently hospitalized. The goal is to improve safety, particularly during patient transfers, with education and training provided by an occupational or physical therapist or other geriatric specialist. The benefit of home assessment and environmental modification is greater when combined with other strategies and in general should not be implemented alone.
Exercise is an important intervention. The number needed to treat (NNT) to prevent one fall in older people over the course of at least 12 weeks is 16.26 This compares favorably with interventions that are commonly used in the general population, such as aspirin therapy as secondary prevention for cardiovascular disease (NNT for 1 year = 50)27 and statin therapy to prevent one death from a cardiovascular event over 5 years in people with known heart disease (NNT = 83).28
Exercise recommendations should be customized to the patient. The amount and type of exercise depends on the patient’s baseline physical activity, medication use including antiplatelet and anticoagulant therapy, home environment, cardiac and pulmonary reserve, vision and hearing deficits, and comorbidities including neuropathy and arthritis.
The well-known risks associated with exercise include myocardial infarction and cardiac arrest, as well as falls and fractures. However, the benefits extend beyond fall risk and include improvements in physical function, glycemic control, cardiopulmonary reserve, bone density, arthritic pain, mood, and cognition. Exercise can also help manage weight, reduce sarcopenia, and increase opportunities for socialization. In most positive trials, the exercise interventions lasted longer than 12 weeks, had variable intensity, and occurred 1 to 3 times per week.
The American College of Sports Medicine recommends that older adults perform aerobic exercise 3 to 5 times per week, 20 to 60 minutes per session (the lower ranges are for frail elderly patients).29 It also recommends resistance training 2 to 4 days per week, 20 to 45 minutes per session, depending on the patient’s level of frailty and conditioning.30 Most older adults do not exercise enough.
Interventions listed at the bottom of Table 2 do not, in general, have enough evidence to support or discourage their use; these are level C recommendations. However, these interventions may be considered for certain individuals. For example, older adults with diabetic neuropathy are often unaware of their foot position when they walk. Additionally, those with diabetic neuropathy may have slower generation of ankle and knee strength compared with age-matched controls. These patients may benefit from targeted physical therapy to strengthen ankle and knee extensors and to retrain stride and speed to improve both gait and safety awareness.
Patients who wear shoes that fit poorly, have high heels, or are not laced or buckled have a higher risk of falls.31 Consider recommending footwear that has a firm, low, rubber heel and a sole with a large surface contact area, which may help reduce the risk of falling.32 Advise patients to wear shoes when they are at home and to avoid using slippers and going barefoot.33
Cataract surgery, another level C intervention, is associated with fewer fall-related injuries, particularly hip fracture.34 Noncataract vision interventions (such as exchanging progressive or bifocal lenses for single-lens glasses) may be effective in select patients if distorted vision in the lower fields of view increases the risk of falling, particularly outdoors.35
INTERVENTIONS FOR SPECIAL POPULATIONS
Falls occur more frequently in mobile residents of long-term care facilities than in community-dwelling adults.7 Institutional residents are older and more frail, have more cognitive impairment, and are prescribed more medications. Half of long-term care residents fall at least once a year.7
The data support giving combined calcium and vitamin D supplementation to older adults in long-term care facilities to reduce fracture rates.36 The NNT to prevent one hip fracture is about 111.37 Hip protectors in this setting may reduce the risk of a hip fracture but also may increase the risk of a pelvic fracture. They do not alter the risk of falling.38
Collaborative interventions can help reduce the fall risk in older adults in the nursing home.39 Input from medical, psychosocial, nursing, podiatric, dietary, and therapy services can be solicited and incorporated into an individualized fall prevention program. The program can also include modifications in the environment to improve safety and reduce fall risk.
The benefits of exercise in reducing injurious falls in long-term care is less clear than in the community, likely because of the heterogeneity of both the long-term care population and the studied interventions. Exercise has other benefits, however. It maintains a person’s ability to complete ADLs, improves mood, reduces hyperglycemia, and improves quality of life. Some studies have found a greater risk of falling with exercise therapy as independence increased.40 However, a meta-analysis in 2013 found that exercise interventions, ranging from 3 to 24 months and consisting mainly of balance and resistance training, reduced the risk of falls by 23%.41 Mixing several types of exercises was helpful. Studies of a longer duration with exercise sessions at least 2 to 3 times per week demonstrated the most benefit.41 There was no statistically significant reduction in fracture risk in this meta-analysis,41 although, possibly, more participants would have been needed for a longer period to demonstrate a benefit. Additionally, no study combined osteoporosis treatment with exercise interventions.
WHAT EVIDENCE EXISTS FOR PATIENTS WITH COGNITIVE IMPAIRMENT?
Currently, there are no specific evidence-based recommendations for fall prevention in community-dwelling older adults with cognitive impairment and dementia.7 Cognitively impaired adults are typically excluded from community studies of fall prevention. The one study that specifically investigated community-dwelling adults with cognitive impairment was not able to demonstrate a fall reduction with multifactorial intervention.42
PREVENTING FALLS IN ELDERLY PATIENTS WHO RECENTLY HAD A STROKE
Falls are common in patients who have had a cerebrovascular event. Up to 7% of patients fall in the first week after a stroke. In the year after a stroke, 55% to 75% of patients experience a fall.43 Falls account for the most common medical complication after a stroke.44
Several small studies found that vitamin D supplementation after a stroke reduced both the rate of falls and the number of people who fall.45 Additional interventions such as exercise, medication, and visual aids have been studied, but there is little evidence to support their use. Mobile patients who have lower-extremity hemiparesis after a stroke may develop osteoporosis in the affected limb, so evaluation and appropriate pharmacologic therapy may be considered.
Falls and fall-related injuries are common in older adults Every year, 30% of those who are 65 and older fall,1 and the consequences are potentially serious. Falls are the primary cause of hip fracture, which requires an extensive period of rehabilitation. However, rehabilitation does not always restore the older adult to his or her preinjury functional state. In fact, at 6 to 12 months after a hip fracture, 22% to 75% of elderly patients have not recovered their prefracture ambulatory or functional status.2
Falls are also the most common cause of traumatic brain injury in older adults,3 often resulting in long-term cognitive and emotional problems and pain that compromise quality of life. Falls can be fatal and in fact are the leading cause of death from injury in older adults.4
Practitioners can reduce fall-related injury5 and potentially improve quality of life by screening older adults yearly, performing a focused history and examination when necessary, and implementing evidence-based interventions.
RISK FACTORS
A single identifiable factor may account for only a small portion of the fall risk. Falls in older adults are, in general, multifactorial and can be caused by medical conditions (eg, sarcopenia, particularly of the lower limbs, vision loss, urinary incontinence, neuropathies), cognitive impairment, medications such as psychotropic drugs, and home hazards such as area rugs, extension cords, and dimly lit stairways.
The strongest predictors of falls are a recent fall and the presence of a gait or balance disorder.6
SCREENING TESTS
Joint guidelines from the American Geriatrics Society and British Geriatrics Society,7 published in 2011, recommend that practitioners screen older adults yearly for fall risk by asking two questions: “Have you fallen in the past year?” and “Are you having difficulty with gait or balance?” A negative response to both questions suggests a low risk of falling in the near future. Patients with two or more falls, a balance or gait problem (subjective or objective), or history of a fall requiring medical attention should undergo a focused history and physical examination plus a multifactorial risk assessment.
A report of one fall without injury should prompt a simple office-based test of balance. Examples of tests include the Get Up and Go, the Timed Up and Go, and the One-Legged Stance (the Unipedal Stance).
In the Get Up and Go test, patients sit comfortably in a chair with a straight back. They rise from the chair, stand still, walk a short distance (about 3 meters), turn around, walk back to the chair, and sit down.8 The clinician notes any deviation from a confident, smooth performance.
In the Timed Up and Go test, the clinician records the time it takes for the patient to rise from a hardback chair, walk 10 feet (3 meters), turn, return to the chair, and sit down.9 Most older adults complete this test in less than 10 seconds. Taking longer than 14 seconds is associated with a high risk of falls.10
For the One-Legged Stance test, the clinician asks the patient to stand on one leg. A patient without significant balance issues is able to stand for at least 5 seconds.11
Figure 1 summarizes the approach for a community-dwelling patient who presents to the outpatient setting. A complete multifactorial risk assessment may require a dedicated appointment or referral to a specialist such as a geriatrician, physiatrist, or neurologist.
WHAT INFORMATION DOES A FOCUSED HISTORY INCLUDE?
The fall-focused history includes:
A detailed description of the circumstances of the fall or falls, symptoms (such as dizziness), and injuries or other consequences of the fall.7
A medication review. Table 1 includes commonly prescribed drug classes associated with increased fall risk.12 Be especially vigilant for eyedrops used to treat glaucoma (some can potentiate bradycardia) and for psychotropic drugs.
Drug regimens with a high psychotropic burden can be identified with the Drug Burden Index13 or the Anticholinergic Risk Scale,14 but these scales are cumbersome and are usually used only as part of a research study. The updated Beers criteria15 and use of a structured medication review such as the START and STOPP algorithms16 can help prune unnecessary, inappropriate, and high-risk medications such as:
- Selective serotonin reuptake inhibitors in the absence of current major depression. These drugs increase the risk of falls and decrease bone density.17
- Proton pump inhibitors in the absence of a true indication for this drug class to treat reflux. Drugs in this class reduce bone density and increase the risk of hip fracture after 1 year of continuous use18
- Cholinesterase inhibitors in the absence of demonstrated benefit to dementia symptoms for the particular patient. Drugs in this class are associated with falls, hip fracture, bradycardia, and possible need for pacemaker placement.19
Review of activities of daily living (ADLs). A functional assessment of the patient’s ability to complete ADLs helps identify targets for therapy. Assess whether the patient is afraid of falling and, if so, what impact this fear has on ADLs. This can help determine whether the fear protects the patient from performing risky tasks, or harms the patient by contributing to deconditioning.
Medical conditions. Consider chronic conditions that can impair mobility and increase fall risk. These include urinary incontinence, cognitive impairment (eg, dementia), neuropathy, degenerative neurologic conditions such as Parkinson disease, and degenerative arthritis. Osteoporosis increases the risk of fracture in a fall. Vitamin D deficiency increases both fall and fracture risk.20
PHYSICAL EXAMINATION FINDINGS
Assess the patient’s vision, proprioception, reflexes, and cortical, extrapyramidal, and cerebellar function.7
Perform a detailed assessment of the patient’s gait, balance, and mobility. Assess the lower extremities for joint and nerve function, muscle strength, and range of motion.7 The use of brain imaging, if appropriate, is guided by gait abnormalities. Unexpected findings such as neuropathy may require referrals for further evaluation.
Examine the patient’s feet and footwear for signs of poor fit and for styles that may be inappropriate for someone at risk of falling, such as high heels.
Conduct a cardiovascular examination. In addition to assessing heart rate and rhythm and checking for heart murmurs, evaluate the patient for postural changes in heart rate and blood pressure. Wait at least 2 minutes before asking the patient to change position from supine to seated and from seated to standing. A longer interval (3 to 5 minutes) can be used depending on the patient’s history. For example, an older adult reporting a syncopal episode standing by the kitchen sink may need a longer standing interval prior to blood pressure measurement than an older adult who falls right after standing up from a chair.
If there is a strong suspicion that an orthostatic condition contributed to a fall but it is not possible to elicit orthostasis in the office, it may be necessary to refer the patient for tilt-table testing. If the circumstances suggest that pressure along the neck, or turning the neck, contributed to a fall, referral for carotid sinus stimulation may be appropriate. If there is a concern that a brady- or tachyarrhythmia contributed to the fall, a referral for 24- or 48-hour Holter monitoring or a 30-day loop monitor may be indicated.
Assess the patient’s mental status. Cognitive impairment itself is an independent predictor of falls7 because it can reduce processing speed and impair executive function.21 Executive dysfunction may contribute to falls by causing problems with multitasking, drug compliance, and judgment. The presence and severity of cognitive impairment may affect recommendation options (see below), so the assessment should include a screening test. Consider using the Mini-Cog, which requires the patient to recall three words and draw an analog clock (Figure 2).22
Some cognitive screening tests validated for use in the general older population include the General Practitioner Assessment of Cognition and the Memory Impairment Screen.23 More involved cognitive testing such as the Folstein Mini-Mental State Examination, Montreal Cognitive Assessment, and the Saint Louis University Mental Status Examination are routinely performed in a geriatric or neurologic setting. The Folstein is a proprietary test; the other two are not.
Conditions such as circulatory disease, chronic obstructive pulmonary disease, depression, and arthritis are associated with a higher risk of falling, even with adjustment for drug use and other potential confounding factors.24
A brief mood assessment is part of the multifactorial assessment because mood disorders in older adults can lead to deconditioning, drug noncompliance, and other conditions that lead to falls and fall-related injuries. Options for screening include the Geriatric Depression Scale (15 or 30 questions) and the Patient Health Questionnaire (the PHQ-2 or the PHQ-9).7
WHAT ARE THE EVIDENCE-BASED INTERVENTIONS?
In general, interventions are chosen according to the risks identified by the assessment; multiple interventions are usually necessary. It is ineffective to identify risk factors without providing intervention.25
Specific interventions with recommendation levels A and B are listed in Table 2.7 Level A interventions are specifically supported by strong evidence and should be recommended. Of note, although vitamin D3 may not be bioequivalent to vitamin D2, studies in older adults have not consistently found a clinically different outcome, and either may be supplemented in the community-dwelling elderly. Except for vitamin D, these interventions target community-dwelling older adults who are cognitively intact.
Home assessments are effective in high-risk patients, such as those with poor vision and those who were recently hospitalized. The goal is to improve safety, particularly during patient transfers, with education and training provided by an occupational or physical therapist or other geriatric specialist. The benefit of home assessment and environmental modification is greater when combined with other strategies and in general should not be implemented alone.
Exercise is an important intervention. The number needed to treat (NNT) to prevent one fall in older people over the course of at least 12 weeks is 16.26 This compares favorably with interventions that are commonly used in the general population, such as aspirin therapy as secondary prevention for cardiovascular disease (NNT for 1 year = 50)27 and statin therapy to prevent one death from a cardiovascular event over 5 years in people with known heart disease (NNT = 83).28
Exercise recommendations should be customized to the patient. The amount and type of exercise depends on the patient’s baseline physical activity, medication use including antiplatelet and anticoagulant therapy, home environment, cardiac and pulmonary reserve, vision and hearing deficits, and comorbidities including neuropathy and arthritis.
The well-known risks associated with exercise include myocardial infarction and cardiac arrest, as well as falls and fractures. However, the benefits extend beyond fall risk and include improvements in physical function, glycemic control, cardiopulmonary reserve, bone density, arthritic pain, mood, and cognition. Exercise can also help manage weight, reduce sarcopenia, and increase opportunities for socialization. In most positive trials, the exercise interventions lasted longer than 12 weeks, had variable intensity, and occurred 1 to 3 times per week.
The American College of Sports Medicine recommends that older adults perform aerobic exercise 3 to 5 times per week, 20 to 60 minutes per session (the lower ranges are for frail elderly patients).29 It also recommends resistance training 2 to 4 days per week, 20 to 45 minutes per session, depending on the patient’s level of frailty and conditioning.30 Most older adults do not exercise enough.
Interventions listed at the bottom of Table 2 do not, in general, have enough evidence to support or discourage their use; these are level C recommendations. However, these interventions may be considered for certain individuals. For example, older adults with diabetic neuropathy are often unaware of their foot position when they walk. Additionally, those with diabetic neuropathy may have slower generation of ankle and knee strength compared with age-matched controls. These patients may benefit from targeted physical therapy to strengthen ankle and knee extensors and to retrain stride and speed to improve both gait and safety awareness.
Patients who wear shoes that fit poorly, have high heels, or are not laced or buckled have a higher risk of falls.31 Consider recommending footwear that has a firm, low, rubber heel and a sole with a large surface contact area, which may help reduce the risk of falling.32 Advise patients to wear shoes when they are at home and to avoid using slippers and going barefoot.33
Cataract surgery, another level C intervention, is associated with fewer fall-related injuries, particularly hip fracture.34 Noncataract vision interventions (such as exchanging progressive or bifocal lenses for single-lens glasses) may be effective in select patients if distorted vision in the lower fields of view increases the risk of falling, particularly outdoors.35
INTERVENTIONS FOR SPECIAL POPULATIONS
Falls occur more frequently in mobile residents of long-term care facilities than in community-dwelling adults.7 Institutional residents are older and more frail, have more cognitive impairment, and are prescribed more medications. Half of long-term care residents fall at least once a year.7
The data support giving combined calcium and vitamin D supplementation to older adults in long-term care facilities to reduce fracture rates.36 The NNT to prevent one hip fracture is about 111.37 Hip protectors in this setting may reduce the risk of a hip fracture but also may increase the risk of a pelvic fracture. They do not alter the risk of falling.38
Collaborative interventions can help reduce the fall risk in older adults in the nursing home.39 Input from medical, psychosocial, nursing, podiatric, dietary, and therapy services can be solicited and incorporated into an individualized fall prevention program. The program can also include modifications in the environment to improve safety and reduce fall risk.
The benefits of exercise in reducing injurious falls in long-term care is less clear than in the community, likely because of the heterogeneity of both the long-term care population and the studied interventions. Exercise has other benefits, however. It maintains a person’s ability to complete ADLs, improves mood, reduces hyperglycemia, and improves quality of life. Some studies have found a greater risk of falling with exercise therapy as independence increased.40 However, a meta-analysis in 2013 found that exercise interventions, ranging from 3 to 24 months and consisting mainly of balance and resistance training, reduced the risk of falls by 23%.41 Mixing several types of exercises was helpful. Studies of a longer duration with exercise sessions at least 2 to 3 times per week demonstrated the most benefit.41 There was no statistically significant reduction in fracture risk in this meta-analysis,41 although, possibly, more participants would have been needed for a longer period to demonstrate a benefit. Additionally, no study combined osteoporosis treatment with exercise interventions.
WHAT EVIDENCE EXISTS FOR PATIENTS WITH COGNITIVE IMPAIRMENT?
Currently, there are no specific evidence-based recommendations for fall prevention in community-dwelling older adults with cognitive impairment and dementia.7 Cognitively impaired adults are typically excluded from community studies of fall prevention. The one study that specifically investigated community-dwelling adults with cognitive impairment was not able to demonstrate a fall reduction with multifactorial intervention.42
PREVENTING FALLS IN ELDERLY PATIENTS WHO RECENTLY HAD A STROKE
Falls are common in patients who have had a cerebrovascular event. Up to 7% of patients fall in the first week after a stroke. In the year after a stroke, 55% to 75% of patients experience a fall.43 Falls account for the most common medical complication after a stroke.44
Several small studies found that vitamin D supplementation after a stroke reduced both the rate of falls and the number of people who fall.45 Additional interventions such as exercise, medication, and visual aids have been studied, but there is little evidence to support their use. Mobile patients who have lower-extremity hemiparesis after a stroke may develop osteoporosis in the affected limb, so evaluation and appropriate pharmacologic therapy may be considered.
- Tromp AM, Pluijm SM, Smit JH, Deeg DJ, Bouter LM, Lips P. Fall-risk screening test: a prospective study on predictors for falls in community-dwelling elderly. J Clin Epidemiol 2001; 54:837–844.
- Binder EF, Brown M, Sinacore DR, Steger-May K, Yarasheski KE, Schechtman KB. Effects of extended outpatient rehabilitation after hip fracture: a randomized controlled trial. JAMA 2004; 292:837–846.
- Sterling DA, O’Connor JA, Bonadies J. Geriatric falls: injury severity is high and disproportionate to mechanism. J Trauma 2001; 50:116–119.
- Centers for Disease Control and Prevention (CDC). Web-based Injury Statistics Query and Reporting System (WISQARSTM). www.cdc.gov/injury/wisqars. Accessed April 8, 2015.
- Tinetti ME, Baker DI, King M, et al. Effect of dissemination of evidence in reducing injuries from falls. N Engl J Med 2008; 359:252–261.
- Ganz DA, Bao Y, Shekelle PG, Rubenstein LZ. Will my patient fall? JAMA 2007; 297:77–86.
- Panel on Prevention of Falls in Older Persons, American Geriatrics Society and British Geriatrics Society. Summary of the Updated American Geriatrics Society/British Geriatrics Society clinical practice guideline for prevention of falls in older persons. J Am Geriatr Soc 2011; 59:148–157.
- Mathias S, Nayak US, Isaacs B. Balance in elderly patients: the “get-up and go” test. Arch Phys Med Rehabil 1986; 67:387–389.
- Podsiadlo D, Richardson S. The timed “Up & Go”: a test of basic functional mobility for frail elderly persons. J Am Geriatr Soc 1991; 39:142–148.
- Shumway-Cook A, Brauer S, Woollacott M. Predicting the probability for falls in community-dwelling older adults using the Timed Up & Go Test. Phys Ther 2000; 80:896–903.
- Springer BA, Marin R, Cyhan T, Roberts H, Gill NW. Normative values for the unipedal stance test with eyes open and closed. J Geriatr Phys Ther 2007; 30:8–15.
- Leipzig RM, Cumming RG, Tinetti ME. Drugs and falls in older people: a systematic review and meta-analysis: I. Psychotropic drugs. J Am Geriatr Soc 1999; 47:30–39.
- Hilmer SN, Mager DE, Simonsick EM, et al. A drug burden index to define the functional burden of medications in older people. Arch Intern Med 2007; 167:781–787.
- Rudolph JL, Salow MJ, Angelini MC, McGlinchey RE. The anticholinergic risk scale and anticholinergic adverse effects in older persons. Arch Intern Med 2008; 168:508–513.
- American Geriatrics Society 2012 Beers Criteria Update Expert Panel. American Geriatrics Society updated Beers Criteria for potentially inappropriate medication use in older adults. J Am Geriatr Soc 2012; 60:616–631.
- Gallagher P, Ryan C, Byrne S, Kennedy J, O’Mahony D. STOPP (Screening Tool of Older Person’s Prescriptions) and START (Screening Tool to Alert doctors to Right Treatment). Consensus validation. Int J Clin Pharmacol Ther 2008; 46:72–83.
- Sterke CS, Ziere G, van Beeck EF, Looman CW, van der Cammen TJ. Dose-response relationship between selective serotonin re-uptake inhibitors and injurious falls: a study in nursing home residents with dementia. Br J Clin Pharmacol 2012; 73:812–820.
- Khalili H, Huang ES, Jacobson BC, Camargo CA Jr, Feskanich D, Chan AT. Use of proton pump inhibitors and risk of hip fracture in relation to dietary and lifestyle factors: a prospective cohort study. BMJ 2012; 344:e372.
- Gill SS, Anderson GM, Fischer HD, et al. Syncope and its consequences in patients with dementia receiving cholinesterase inhibitors: a population-based cohort study. Arch Intern Med 2009; 169:867–873.
- Janssen HC, Samson MM, Verhaar HJ. Vitamin D deficiency, muscle function, and falls in elderly people. Am J Clin Nutr 2002; 75:611–615.
- Muir SW, Gopaul K, Montero Odasso MM. The role of cognitive impairment in fall risk among older adults: a systematic review and meta-analysis. Age Ageing 2012; 41:299–308.
- Borson S, Scanlan J, Brush M, Vitaliano P, Dokmak A. The Mini-Cog: a cognitive ‘vital signs’ measure for dementia screening in multi-lingual elderly. Int J Geriatr Psychiatry 2000; 15:1021–1027.
- Cordell CB, Borson S, Boustani M, et al; Medicare Detection of Cognitive Impairment Workgroup. Alzheimer’s Association recommendations for operationalizing the detection of cognitive impairment during the Medicare Annual Wellness Visit in a primary care setting. Alzheimers Dement 2013; 9:141–150.
- Lawlor DA, Patel R, Ebrahim S. Association between falls in elderly women and chronic diseases and drug use: cross sectional study. BMJ 2003; 327:712–717.
- Day L, Fildes B, Gordon I, Fitzharris M, Flamer H, Lord S. Randomised factorial trial of falls prevention among older people living in their own homes. BMJ 2002; 325:128.
- Chang JT, Morton SC, Rubenstein LZ, et al. Interventions for the prevention of falls in older adults: systematic review and meta-analysis of randomised clinical trials. BMJ 2004; 328:680.
- Antithrombotic Trialists’ (ATT) Collaboration; Baigent C, Blackwell L, Collins R, et al. Aspirin in the primary and secondary prevention of vascular disease: collaborative meta-analysis of individual participant data from randomised trials. Lancet 2009; 373:1849–1860.
- Baigent C, Keech A, Kearney PM, et al; Cholesterol Treatment Trialists’ (CTT) Collaborators. Efficacy and safety of cholesterol-lowering treatment: prospective meta-analysis of data from 90,056 participants in 14 randomised trials of statins. Lancet 2005; 366:1267–1278.
- Mazzeo RS; American College of Sports Medicine (ACSM). Exercise and the older adult. ACSM Current Comment. www.acsm.org/docs/current-comments/exerciseandtheolderadult.pdf. Accessed April 8, 2015.
- Willoughby DS; American College of Sports Medicine (ACSM). Resistance training and the older adult. ACSM Current Comment. www.acsm.org/docs/current-comments/resistancetrainingandtheoa.pdf. Accessed April 8, 2015.
- Tencer AF, Koepsell TD, Wolf ME, et al. Biomechanical properties of shoes and risk of falls in older adults. J Am Geriatr Soc 2004; 52:1840–1846.
- Lord SR, Bashford GM. Shoe characteristics and balance in older women. J Am Geriatr Soc 1996; 44:429–433.
- Kelsey JL, Procter-Gray E, Nguyen US, Li W, Kiel DP, Hannan MT. Footwear and falls in the home among older individuals in the MOBILIZE Boston Study. Footwear Sci 2010; 2:123–129.
- Tseng VL, Yu F, Lum F, Coleman AL. Risk of fractures following cataract surgery in Medicare beneficiaries. JAMA 2012; 308:493–501.
- Cumming RG, Ivers R, Clemson L, et al. Improving vision to prevent falls in frail older people: a randomized trial. J Am Geriatr Soc 2007; 55:175–181.
- Bischoff-Ferrari HA, Dawson-Hughes B, Willett WC, et al. Effect of vitamin D on falls: a meta-analysis. JAMA 2004; 291:1999–2006.
- Avenell A, Mak JCS, O’Connell D. Vitamin D and vitamin D analogues for preventing fractures in postmenopausal women and older men. Cochrane Database Syst Rev 2014; 4:CD000227.
- Santesso N, Carrasco-Labra A, Brignardello-Petersen R. Hip protectors for preventing hip fractures in older people. Cochrane Database Syst Rev 2014; 3:CD001255.
- Messinger-Rapport B, Dumas LG. Falls in the nursing home: a collaborative approach. Nurs Clin North Am 2009; 44:187–195.
- Faber MJ, Bosscher RJ, Chin A, Paw MJ, van Wieringen PC. Effects of exercise programs on falls and mobility in frail and pre-frail older adults: a multicenter randomized controlled trial. Arch Phys Med Rehabil 2006; 87:885–896.
- Silva RB, Eslick GD, Duque G. Exercise for falls and fracture prevention in long term care facilities: a systematic review and meta-analysis. J Am Med Dir Assoc 2013; 14:685–689.e2.
- Shaw FE, Bond J, Richardson DA, et al. Multifactorial intervention after a fall in older people with cognitive impairment and dementia presenting to the accident and emergency department: randomised controlled trial. BMJ 2003; 326:73.
- Ashburn A, Hyndman D, Pickering R, Yardley L, Harris S. Predicting people with stroke at risk of falls. Age Ageing 2008; 37:270–276.
- Davenport RJ, Dennis MS, Wellwood I, Warlow CP. Complications after acute stroke. Stroke 1996; 27:415–420.
- Verheyden GS, Weerdesteyn V, Pickering RM, et al. Interventions for preventing falls in people after stroke. Cochrane Database Syst Rev 2013; 5:CD008728.
- Tromp AM, Pluijm SM, Smit JH, Deeg DJ, Bouter LM, Lips P. Fall-risk screening test: a prospective study on predictors for falls in community-dwelling elderly. J Clin Epidemiol 2001; 54:837–844.
- Binder EF, Brown M, Sinacore DR, Steger-May K, Yarasheski KE, Schechtman KB. Effects of extended outpatient rehabilitation after hip fracture: a randomized controlled trial. JAMA 2004; 292:837–846.
- Sterling DA, O’Connor JA, Bonadies J. Geriatric falls: injury severity is high and disproportionate to mechanism. J Trauma 2001; 50:116–119.
- Centers for Disease Control and Prevention (CDC). Web-based Injury Statistics Query and Reporting System (WISQARSTM). www.cdc.gov/injury/wisqars. Accessed April 8, 2015.
- Tinetti ME, Baker DI, King M, et al. Effect of dissemination of evidence in reducing injuries from falls. N Engl J Med 2008; 359:252–261.
- Ganz DA, Bao Y, Shekelle PG, Rubenstein LZ. Will my patient fall? JAMA 2007; 297:77–86.
- Panel on Prevention of Falls in Older Persons, American Geriatrics Society and British Geriatrics Society. Summary of the Updated American Geriatrics Society/British Geriatrics Society clinical practice guideline for prevention of falls in older persons. J Am Geriatr Soc 2011; 59:148–157.
- Mathias S, Nayak US, Isaacs B. Balance in elderly patients: the “get-up and go” test. Arch Phys Med Rehabil 1986; 67:387–389.
- Podsiadlo D, Richardson S. The timed “Up & Go”: a test of basic functional mobility for frail elderly persons. J Am Geriatr Soc 1991; 39:142–148.
- Shumway-Cook A, Brauer S, Woollacott M. Predicting the probability for falls in community-dwelling older adults using the Timed Up & Go Test. Phys Ther 2000; 80:896–903.
- Springer BA, Marin R, Cyhan T, Roberts H, Gill NW. Normative values for the unipedal stance test with eyes open and closed. J Geriatr Phys Ther 2007; 30:8–15.
- Leipzig RM, Cumming RG, Tinetti ME. Drugs and falls in older people: a systematic review and meta-analysis: I. Psychotropic drugs. J Am Geriatr Soc 1999; 47:30–39.
- Hilmer SN, Mager DE, Simonsick EM, et al. A drug burden index to define the functional burden of medications in older people. Arch Intern Med 2007; 167:781–787.
- Rudolph JL, Salow MJ, Angelini MC, McGlinchey RE. The anticholinergic risk scale and anticholinergic adverse effects in older persons. Arch Intern Med 2008; 168:508–513.
- American Geriatrics Society 2012 Beers Criteria Update Expert Panel. American Geriatrics Society updated Beers Criteria for potentially inappropriate medication use in older adults. J Am Geriatr Soc 2012; 60:616–631.
- Gallagher P, Ryan C, Byrne S, Kennedy J, O’Mahony D. STOPP (Screening Tool of Older Person’s Prescriptions) and START (Screening Tool to Alert doctors to Right Treatment). Consensus validation. Int J Clin Pharmacol Ther 2008; 46:72–83.
- Sterke CS, Ziere G, van Beeck EF, Looman CW, van der Cammen TJ. Dose-response relationship between selective serotonin re-uptake inhibitors and injurious falls: a study in nursing home residents with dementia. Br J Clin Pharmacol 2012; 73:812–820.
- Khalili H, Huang ES, Jacobson BC, Camargo CA Jr, Feskanich D, Chan AT. Use of proton pump inhibitors and risk of hip fracture in relation to dietary and lifestyle factors: a prospective cohort study. BMJ 2012; 344:e372.
- Gill SS, Anderson GM, Fischer HD, et al. Syncope and its consequences in patients with dementia receiving cholinesterase inhibitors: a population-based cohort study. Arch Intern Med 2009; 169:867–873.
- Janssen HC, Samson MM, Verhaar HJ. Vitamin D deficiency, muscle function, and falls in elderly people. Am J Clin Nutr 2002; 75:611–615.
- Muir SW, Gopaul K, Montero Odasso MM. The role of cognitive impairment in fall risk among older adults: a systematic review and meta-analysis. Age Ageing 2012; 41:299–308.
- Borson S, Scanlan J, Brush M, Vitaliano P, Dokmak A. The Mini-Cog: a cognitive ‘vital signs’ measure for dementia screening in multi-lingual elderly. Int J Geriatr Psychiatry 2000; 15:1021–1027.
- Cordell CB, Borson S, Boustani M, et al; Medicare Detection of Cognitive Impairment Workgroup. Alzheimer’s Association recommendations for operationalizing the detection of cognitive impairment during the Medicare Annual Wellness Visit in a primary care setting. Alzheimers Dement 2013; 9:141–150.
- Lawlor DA, Patel R, Ebrahim S. Association between falls in elderly women and chronic diseases and drug use: cross sectional study. BMJ 2003; 327:712–717.
- Day L, Fildes B, Gordon I, Fitzharris M, Flamer H, Lord S. Randomised factorial trial of falls prevention among older people living in their own homes. BMJ 2002; 325:128.
- Chang JT, Morton SC, Rubenstein LZ, et al. Interventions for the prevention of falls in older adults: systematic review and meta-analysis of randomised clinical trials. BMJ 2004; 328:680.
- Antithrombotic Trialists’ (ATT) Collaboration; Baigent C, Blackwell L, Collins R, et al. Aspirin in the primary and secondary prevention of vascular disease: collaborative meta-analysis of individual participant data from randomised trials. Lancet 2009; 373:1849–1860.
- Baigent C, Keech A, Kearney PM, et al; Cholesterol Treatment Trialists’ (CTT) Collaborators. Efficacy and safety of cholesterol-lowering treatment: prospective meta-analysis of data from 90,056 participants in 14 randomised trials of statins. Lancet 2005; 366:1267–1278.
- Mazzeo RS; American College of Sports Medicine (ACSM). Exercise and the older adult. ACSM Current Comment. www.acsm.org/docs/current-comments/exerciseandtheolderadult.pdf. Accessed April 8, 2015.
- Willoughby DS; American College of Sports Medicine (ACSM). Resistance training and the older adult. ACSM Current Comment. www.acsm.org/docs/current-comments/resistancetrainingandtheoa.pdf. Accessed April 8, 2015.
- Tencer AF, Koepsell TD, Wolf ME, et al. Biomechanical properties of shoes and risk of falls in older adults. J Am Geriatr Soc 2004; 52:1840–1846.
- Lord SR, Bashford GM. Shoe characteristics and balance in older women. J Am Geriatr Soc 1996; 44:429–433.
- Kelsey JL, Procter-Gray E, Nguyen US, Li W, Kiel DP, Hannan MT. Footwear and falls in the home among older individuals in the MOBILIZE Boston Study. Footwear Sci 2010; 2:123–129.
- Tseng VL, Yu F, Lum F, Coleman AL. Risk of fractures following cataract surgery in Medicare beneficiaries. JAMA 2012; 308:493–501.
- Cumming RG, Ivers R, Clemson L, et al. Improving vision to prevent falls in frail older people: a randomized trial. J Am Geriatr Soc 2007; 55:175–181.
- Bischoff-Ferrari HA, Dawson-Hughes B, Willett WC, et al. Effect of vitamin D on falls: a meta-analysis. JAMA 2004; 291:1999–2006.
- Avenell A, Mak JCS, O’Connell D. Vitamin D and vitamin D analogues for preventing fractures in postmenopausal women and older men. Cochrane Database Syst Rev 2014; 4:CD000227.
- Santesso N, Carrasco-Labra A, Brignardello-Petersen R. Hip protectors for preventing hip fractures in older people. Cochrane Database Syst Rev 2014; 3:CD001255.
- Messinger-Rapport B, Dumas LG. Falls in the nursing home: a collaborative approach. Nurs Clin North Am 2009; 44:187–195.
- Faber MJ, Bosscher RJ, Chin A, Paw MJ, van Wieringen PC. Effects of exercise programs on falls and mobility in frail and pre-frail older adults: a multicenter randomized controlled trial. Arch Phys Med Rehabil 2006; 87:885–896.
- Silva RB, Eslick GD, Duque G. Exercise for falls and fracture prevention in long term care facilities: a systematic review and meta-analysis. J Am Med Dir Assoc 2013; 14:685–689.e2.
- Shaw FE, Bond J, Richardson DA, et al. Multifactorial intervention after a fall in older people with cognitive impairment and dementia presenting to the accident and emergency department: randomised controlled trial. BMJ 2003; 326:73.
- Ashburn A, Hyndman D, Pickering R, Yardley L, Harris S. Predicting people with stroke at risk of falls. Age Ageing 2008; 37:270–276.
- Davenport RJ, Dennis MS, Wellwood I, Warlow CP. Complications after acute stroke. Stroke 1996; 27:415–420.
- Verheyden GS, Weerdesteyn V, Pickering RM, et al. Interventions for preventing falls in people after stroke. Cochrane Database Syst Rev 2013; 5:CD008728.
KEY POINTS
- Practitioners can reduce fall-related injury by screening older adults yearly with questions about problems with balance and gait, performing a focused history and examination when necessary, and implementing evidence-based interventions.
- Cognitive impairment itself is an independent predictor of falls because it can reduce processing speed and impair executive function.
- An exercise program with resistance, balance, and gait training is usually prescribed to patients at high risk, along with a home assessment and withdrawal or minimization of psychoactive and antipsychotic medications.
- Combined calcium and vitamin D supplements should be given to most older adults in long-term care facilities to reduce fracture rates.
- There are no specific evidence-based recommendations for fall prevention in community-living older adults with cognitive impairment or dementia.
A 79-year-old with acute portal vein thrombosis
A 79-year-old man presented with chills and fever. He had a history of polymyalgia rheumatica and had been tapered off corticosteroids 1 month before admission. One week before he presented, he had developed generalized myalgia, chills, and fatigue. A cortisol stimulation test at that time was normal, prednisone was restarted, and his symptoms had improved. But 1 day before he presented, the chills had returned, this time with fever. Laboratory testing at an outpatient clinic had revealed abnormal liver enzyme levels.
On the day he presented, he felt worse, with persistent chills, fever, and vague lower abdominal pain, but he denied nausea, vomiting, changes in bowel habits, melena, hematochezia, and hematemesis. He was admitted for additional evaluation.
His medical history also included coronary artery disease (for which he had undergone coronary artery bypass grafting), hypertension, stable liver cysts, and gout. He had no known inflammatory bowel disease and no recent abdominal surgery. His medications included prednisone, atorvastatin, atenolol, aspirin, niacin, and cholecalciferol. He had no history of smoking, significant drinking, or use of illicit drugs. He had no respiratory or cardiac symptoms or neurologic symptoms consistent with a transient ischemic attack or stroke. He denied any rashes.
On admission, he was febrile, with temperatures reaching 102˚F (38.9˚C). His blood pressure was 137/63 mm Hg, pulse 54 beats per minute, respiration rate 18 breaths per minute, and oxygen saturation 97% on room air. A harsh systolic murmur was noted on physical examination. His abdomen was nondistended, nontender, and without bruits.
Laboratory testing (Table 1) revealed leukocytosis, anemia, mildly abnormal aminotransferase levels, elevated alkaline phosphatase, and markedly elevated C-reactive protein.
A full workup for fever was performed, including blood and urine cultures; chest radiography; contrast-enhanced computed tomography (CT) of the chest, abdomen, and pelvis; magnetic resonance imaging (MRI) of the abdomen; and colonoscopy. No source of infection—bacterial, viral, or fungal—was found. However, CT revealed new extensive thrombosis of the right portal vein and its branches (Figure 1).
CLINICAL PRESENTATION
1. Which of the following is least consistent with the clinical presentation of acute portal vein thrombosis?
- Abdominal pain
- Fever and chills
- Hematemesis
- Leukocytosis
- Absence of symptoms
Of these signs and symptoms, hematemesis is the least likely to be associated with acute portal vein thrombosis, although it can be associated with chronic cases.
Symptoms of portal vein thrombosis
Portal vein thrombosis causes extrahepatic obstruction of the portal venous system, which provides two-thirds of the total hepatic blood flow.
Acute. Often, thrombotic occlusion of the portal vein produces no acute symptoms because of immediate, compensatory vasodilation of the hepatic arterial system.1 Additionally, in the ensuing days, the thrombus becomes an organized collagenous plug, and collateral veins develop to bypass the blocked vein and maintain portal perfusion in a process called cavernous transformation.1,2 Thus, many patients have no symptoms.
If symptoms occur, portal vein thrombosis can initially present as transient abdominal pain with fever, as seen in this patient.3 Many patients with acute portal vein thrombosis experience abdominal pain due to intra-abdominal sepsis, also referred to as pylephlebitis.2,4 High, spiking fevers and chills also occur, caused by infected thrombi associated with intra-abdominal infections such as appendicitis, diverticulitis, and pancreatitis.5,6
Chronic. In contrast, symptomatic chronic portal vein thrombosis commonly presents with sequelae of portal hypertension, most notably gastrointestinal bleeding. Hematemesis from ruptured esophageal varices is the most frequent reason for seeking medical attention, though varices also develop in the stomach, duodenum, jejunum, gallbladder, and bile ducts.2,7 Abdominal pain is less common in chronic portal vein thrombosis unless the thrombus extends into the mesenteric veins and causes bowel ischemia or infarction. Long-standing portal vein thrombosis may also lead to dilated venous collaterals that compress large bile ducts, resulting in portal cholangiopathy.1,8
Portal vein thrombosis may present as acute intestinal ischemia and bowel infarction, though this is uncommon. This is generally seen with extensive occlusive portal vein thrombosis and concomitant mesenteric venous thrombosis.1,2
Other symptoms that are common but nonspecific are nausea, vomiting, diarrhea, weight loss, and anorexia.2
Signs of portal vein thrombosis
On examination, patients with acute portal vein thrombosis have minimal physical signs unless they have other contributing conditions. For example, acute portal vein thrombosis can result in abdominal distention secondary to ileus, or guarding and ascites secondary to intestinal infarction.3,9
Some patients with chronic portal vein thrombosis also have normal physical findings, but many have signs. Splenomegaly is seen in 75% to 100% of patients.2,7 Hepatomegaly, abdominal tenderness, and low-grade fever are common as well.2,10 Ascites is usually not present without underlying cirrhosis; however, mild and transient ascites can develop immediately after the thrombotic event before the patient develops collateral circulation.2
Laboratory testing for portal vein thrombosis
Laboratory test results are typically unremarkable. Liver function tests show preserved hepatic function but may reveal mild increases in aspartate aminotransferase (AST), alanine aminotransferase (ALT), alkaline phosphatase, and bilirubin.2,10
In acute cases, elevations of acute-phase reactant levels can occur.9 Leukocytosis and blood cultures growing Bacteroides species are seen in septic cases or pylephlebitis.11,12 There may be mild anemia, particularly after a recent bleeding episode, or mild leukopenia and thrombocytopenia due to hypersplenism. Suspicion of an underlying myeloproliferative disorder is high if thrombocytosis is present.2
DIAGNOSIS
2. All of the following would be appropriate initial diagnostic studies for portal vein thrombosis except which one?
- Doppler ultrasonography
- Contrast-enhanced CT
- Contrast-enhanced MRI
- Angiography
Portal vein thrombosis is most often diagnosed with noninvasive techniques, namely Doppler ultrasonography, CT, and MRI—not angiography.
Ultrasonography can reveal an echogenic thrombus in the vessel lumen with distention of the portal vein proximal to the occlusion and extensive collateral vessels. Plain ultrasonography fails to reveal the thrombus in up to one-third of patients. However, duplex ultrasonography with color flow Doppler imaging can confirm partial or complete absence of flow in the vein with 89% sensitivity and 92% specificity.13,14
On contrast-enhanced CT, the thrombus appears as a filling defect within the portal venous segment. Complete occlusion of the vein may produce a “train track” appearance due to contrast around the vessel.10 Without contrast, the clot will appear as hyperattenuating material in the portal vein, but contrast-enhanced imaging may be necessary to differentiate the thrombus from the vessel wall.15 Gas within the portal venous system is specific for pylephlebitis.4 Evidence of cavernous transformation is seen in chronic portal vein thrombosis.
Contrast-enhanced magnetic resonance angiography can also be used to evaluate patency and flow direction. In addition, it provides detailed anatomic information about the entire portal venous system, including the intrahepatic portal vessels, which is limited in CT imaging.2,10 CT and MRI can also help to identify predisposing conditions (eg, intra-abdominal infection, hepatocellular carcinoma) and complications (eg, intestinal infarction) associated with portal vein thrombosis.
Angiography can be considered if noninvasive techniques are inconclusive but is generally not necessary, given the increased use of CT and MRI.
In our patient, abdominal CT revealed occlusive thrombosis of the right portal vein and its branches (Figure 1). The left and main portal veins were patent. There was no evidence of intra-abdominal infection or infarction.
FINDING THE CAUSE
3. Which of the following is not a common cause
of portal vein thrombosis?
- A hypercoagulable state
- Immune deficiency
- Intra-abdominal infection
- Malignancy
- Portal hypertension
Once portal vein thrombosis has been diagnosed, the cause should be identified (Table 2). The differential diagnosis is broad, including both local factors (eg, injury to the portal vein, local inflammation, infection) and general factors (eg, inherited and acquired hypercoagulable conditions). Thrombophilias are identified in 60% of patients with portal vein thrombosis and local factors in 40%.7 Moreover, the etiology is often multifactorial. However, immune deficiency is not a common cause.
Hypercoagulability
Prothrombotic disorders can be either inherited or acquired.
Inherited deficiencies in the natural anticoagulants antithrombin, protein C, and protein S are associated with a high risk of thrombosis but have a low prevalence in the general population. In the setting of liver abnormalities, familial testing may be helpful to distinguish inherited causes of portal vein thrombosis from defective liver function as a consequence of portal vein thrombosis. The factor V Leiden mutation (G1691A) and the G20210A mutation in the prothrombin gene are more prevalent (> 2%) but generally confer a lower thrombosis risk.16 The prothrombin gene mutation G20210A is the most common risk factor for portal vein thrombosis, with prevalence of 2% to 22% in adults with nonmalignant, noncirrhotic portal vein thrombosis.3
Hyperhomocysteinemia due to a methylene tetrahydrofolate reductase (MTHFR) mutation (C677T) is another inherited associated risk factor for portal vein thrombosis, but hyperhomocysteinemia can also arise as a complication of portal vein thrombosis-related liver disease.3
Acquired prothrombotic disorders, particularly myeloproliferative diseases, are found in 22% to 48% of cases of portal vein thrombosis. Many young patients with myeloproliferative disorders present with portal vein thrombosis as the first symptom, and testing for the G1849T point mutation in JAK2 can make the diagnosis.17 Splenectomy with underlying myeloproliferative disorder confers a particularly high risk for portal vein thrombosis.18
Other thrombophilic disorders including antiphospholipid antibody syndrome, paroxysmal nocturnal hemoglobinuria, and malignancy can contribute to portal vein thrombosis.3 Pregnancy and oral contraceptive use have also been associated with hypercoagulability, and cessation of oral estrogen is recommended in such cases. The risk may be further increased in patients on oral contraceptives who have a previously unrecognized hypercoagulable state.3
Inflammation and infection
Inflammation and infection are local risk factors for portal vein thrombosis. Acute portal vein thrombosis has been associated with intra-abdominal infections (eg, appendicitis, cholecystitis) and with inflammatory conditions such as inflammatory bowel disease and pancreatitis.16,19 From 3% to 5% of all portal vein thrombosis cases result from pancreatitis, either from a single acute episode or from repeat inflammation of chronic pancreatitis.10 Portal vein thrombosis in the setting of inflammatory bowel disease can occur even when the disease is in remission, particularly in ulcerative colitis.20,21
Injury to the portal venous system
Abdominal surgery, particularly splenectomy, portosystemic shunting, colectomy, and blunt abdominal trauma can cause injury to the portal venous system, resulting in portal vein thrombosis. This is usually seen only in patients with portal hypertension, an underlying prothrombotic condition such as myeloproliferative disease, or inflammatory bowel disease.10,19,22
Impaired portal vein flow
Cirrhosis and malignancy are major risk factors for portal vein thrombosis. In case series, cirrhosis was found in 24% to 32% of patients with portal vein thrombosis.2,23 However, the overall prevalence of portal vein thrombosis in cirrhotic patients varies widely, from 0.6% to 28%, depending on the degree of cirrhosis.10
The pathogenesis of portal vein thrombosis in cirrhosis is unclear but may be multifactorial. Decreased portal blood flow (with subsequent stasis) and periportal lymphangitis and fibrosis are thought to stimulate thrombus formation.3,10 Additionally, patients with advanced cirrhosis are prothrombotic because of reduced hepatic synthesis of antithrombin, protein C, protein S, and coagulation factors.
Malignancy is associated with 21% to 24% of cases of portal vein thrombosis in adults, with pancreatic cancer and hepatocellular carcinoma being the most common.2,3 Others include cholangiocarcinoma and carcinomas of the stomach, lung, prostate, uterus, and kidney. Cancer causes portal vein thrombosis through a combination of tumor invasion into the portal vein, extrinsic compression by the tumor, periportal fibrosis following surgery or radiation, and hypercoagulability secondary to malignancy.9,16,24
Idiopathic portal vein thrombosis
Portal vein thrombosis is usually caused by one or more of the underlying factors mentioned above but is idiopathic in 8% to 15% of cases.10
Back to our patient
The cause of this patient’s portal vein thrombosis is unclear. He did not have a history of cirrhosis, inflammatory bowel disease, trauma, or abdominal surgery. His febrile illness could have precipitated the formation of a thrombus, but no definitive source of infection or inflammation was discovered. His workup was negative for pancreatitis, appendicitis, cholecystitis, diverticulitis, and prostatitis. No occult malignancy was found. It is also possible that his fever was the result of the thrombosis.
A full hypercoagulability panel revealed no striking abnormalities. He did have elevated fibrinogen and factor VIII levels that were consistent with an acute-phase reaction, along with an elevated erythrocyte sedimentation rate (> 90 mm/hr) and C-reactive protein level. Aside from the portal vein thrombosis, no potential source of inflammation could be identified.
Mildly reduced levels of antithrombin III activity were attributed to enoxaparin therapy and ultimately normalized on repeated testing. The patient had very minimally elevated titers of anticardiolipin immunoglobulin G (1:10 GPL) and anti-beta-2 glycoprotein immunoglobulin M (21 SMU), which were not thought to be significant. Tests for lupus anticoagulant, prothrombin gene mutation, activated protein C resistance, and JAK2 mutation were negative.
TREATMENT
4. Treatment of symptomatic portal vein thrombosis generally includes which two of the following?
- Anticoagulation
- Intravenous gamma globulin
- Broad-spectrum antibiotics
Anticoagulant therapy
Treatment of acute, symptomatic portal vein thrombosis involves anticoagulant therapy to prevent extension of the thrombus and, ultimately, to allow for recanalization of the obstructed veins. Anticoagulant therapy is initially intravenous unfractionated heparin or subcutaneous low-molecular-weight heparin, eventually bridged to an oral agent such as warfarin.3,9 Currently, there are inadequate data on the use of oral or parenteral factor Xa inhibitors or direct thrombin inhibitors in the treatment of this disease.
When started immediately, anticoagulation therapy is associated with complete recanalization in 38.3% and partial recanalization in 14% of patients presenting with complete thrombosis. Without anticoagulation, spontaneous recanalization is unusual.25
Although the optimal duration of anticoagulant therapy is unclear, a minimum of 3 months is generally recommended.9,26 If a hypercoagulable state is present or if the portal vein thrombosis is unprovoked (eg, by surgery, trauma, or an intra-abdominal infection), long-term treatment should be considered.26
Experience with thrombolytic therapy or mechanical recanalization has been limited, but the use of catheter-based techniques for pharmacomechanical thrombolysis has been reported.27–29 Transjugular intrahepatic portosystemic shunting is also an alternative to anticoagulation, but its role in treating portal vein thrombosis is complicated by technical difficulties of the procedure, postoperative complications, and recurrent occlusion of the shunt.25
Currently, there are no data comparing the risk-benefit ratio of early anticoagulation and that of invasive procedures. These more aggressive treatments are generally considered only when there is extensive thrombosis or ascites (which are both predictive factors of poor response to anticoagulation alone) and in patients for whom anticoagulation has failed.3 Surgical thrombectomy is rarely indicated, typically only in instances in which laparotomy is being performed for suspected bowel infarction.3
Antibiotics
In addition to anticoagulation, broad-spectrum antibiotics covering gram-negative and anaerobic bacteria are indicated for those cases of portal vein thrombosis associated with underlying infection.9
For chronic cases, the goals of management are to prevent and treat gastroesophageal variceal bleeding and to prevent recurrent thrombosis.9 Nonselective beta-blockers (eg, propranolol) and endoscopic band ligation have shown evidence of reducing the incidence of recurrent bleeding and prolonging survival in retrospective studies.9,30,31 Long-term anticoagulation is generally indicated to prevent further thrombosis and to increase the likelihood of recanalization only for patients with a permanent prothrombotic condition.9 In patients with clinically significant portal hypertension, the benefit of continued anticoagulation therapy must be weighed against the risk of esophageal and gastric variceal bleeding.
There is controversy regarding how to manage portal vein thrombosis that is incidentally identified and asymptomatic (eg, if it is discovered on an imaging study for another indication). Current guidelines recommend against anticoagulation in patients with incidentally discovered and asymptomatic splanchnic vein thrombosis, including portal vein thrombosis.26
Intravenous gamma globulin is not part of the treatment.
CASE CONTINUED
The patient’s presenting symptoms of fever, chills, and abdominal pain completely resolved after a course of antibiotic therapy. The erythrocyte sedimentation rate subsequently normalized and factor VIII activity improved. We believed that an underlying infectious or inflammatory process had contributed to the development of portal vein thrombosis, though the specific cause could not be identified. The patient was treated with enoxaparin 1 mg/kg twice a day and transitioned to warfarin.
Magnetic resonance venography done 3 months after diagnosis showed persistent right portal vein thrombosis that was largely unchanged. Anticoagulation was continued for 1 year with no change in his portal vein thrombosis on sequential imaging and was subsequently discontinued. To date, no malignancy or infectious process has been found, and the patient continues to do well 2 years later.
- Ponziani FR, Zocco MA, Campanale C, et al. Portal vein thrombosis: insight into physiopathology, diagnosis, and treatment. World J Gastroenterol 2010; 16:143–155.
- Cohen J, Edelman RR, Chopra S. Portal vein thrombosis: a review. Am J Med 1992; 92:173–182.
- Primignani M. Portal vein thrombosis, revisited. Dig Liver Dis 2010; 42:163–170.
- Condat B, Valla D. Nonmalignant portal vein thrombosis in adults. Nat Clin Pract Gastroenterol Hepatol 2006; 3:505–515.
- Condat B, Pessione F, Helene Denninger M, Hillaire S, Valla D. Recent portal or mesenteric venous thrombosis: increased recognition and frequent recanalization on anticoagulant therapy. Hepatology 2000; 32:466–470.
- Sheen CL, Lamparelli H, Milne A, Green I, Ramage JK. Clinical features, diagnosis and outcome of acute portal vein thrombosis. QJM 2000; 93:531–534.
- Sogaard KK, Astrup LB, Vilstrup H, Gronbaek H. Portal vein thrombosis; risk factors, clinical presentation and treatment. BMC Gastroenterol 2007; 7:34.
- Llop E, de Juan C, Seijo S, et al. Portal cholangiopathy: radiological classification and natural history. Gut 2011; 60:853–860.
- DeLeve LD, Valla DC, Garcia-Tsao G; American Association for the Study of Liver Diseases. Vascular disorders of the liver. Hepatology 2009; 49:1729–1764.
- Sobhonslidsuk A, Reddy KR. Portal vein thrombosis: a concise review. Am J Gastroenterol 2002; 97:535–541.
- Ni YH, Wang NC, Peng MY, Chou YY, Chang FY. Bacteroides fragilis bacteremia associated with portal vein and superior mesentery vein thrombosis secondary to antithrombin III and protein C deficiency: a case report. J Microbiol Immunol Infect 2002; 35:255–258.
- Trum J, Valla D, Cohen G, et al. Bacteroides bacteraemia of undetermined origin: strong association with portal vein thrombosis and cryptogenic pylephlebitis. Eur J Gastroenterol Hepatol 1993; 5:655–659.
- Ueno N, Sasaki A, Tomiyama T, Tano S, Kimura K. Color Doppler ultrasonography in the diagnosis of cavernous transformation of the portal vein. J Clin Ultrasound 1997; 25:227–233.
- Tessler FN, Gehring BJ, Gomes AS, et al. Diagnosis of portal vein thrombosis: value of color Doppler imaging. AJR Am J Roentgenol 1991; 157:293–296.
- Hidajat N, Stobbe H, Griesshaber V, Felix R, Schroder RJ. Imaging and radiological interventions of portal vein thrombosis. Acta Radiol 2005; 46:336–343.
- Valla DC, Condat B. Portal vein thrombosis in adults: pathophysiology, pathogenesis and management. J Hepatol 2000; 32:865–871.
- Kralovics R, Passamonti F, Buser AS, et al. A gain-of-function mutation of JAK2 in myeloproliferative disorders. N Engl J Med 2005; 352:1779–1790.
- Krauth MT, Lechner K, Neugebauer EA, Pabinger I. The postoperative splenic/portal vein thrombosis after splenectomy and its prevention—an unresolved issue. Haematologica 2008; 93:1227–1232.
- Sinagra E, Aragona E, Romano C, et al. The role of portal vein thrombosis in the clinical course of inflammatory bowel diseases: report on three cases and review of the literature. Gastroenterol Res Pract 2012; 2012:916428.
- Maconi G, Bolzacchini E, Dell’Era A, Russo U, Ardizzone S, de Franchis R. Portal vein thrombosis in inflammatory bowel diseases: a single-center case series. J Crohns Colitis 2012; 6:362–367.
- Jackson LM, O’Gorman PJ, O’Connell J, Cronin CC, Cotter KP, Shanahan F. Thrombosis in inflammatory bowel disease: clinical setting, procoagulant profile and factor V Leiden. QJM 1997; 90:183–188.
- Eguchi A, Hashizume M, Kitano S, Tanoue K, Wada H, Sugimachi K. High rate of portal thrombosis after splenectomy in patients with esophageal varices and idiopathic portal hypertension. Arch Surg 1991; 126:752–755.
- Ogren M, Bergqvist D, Björck M, Acosta S, Eriksson H, Sternby NH. Portal vein thrombosis: prevalence, patient characteristics and lifetime risk: a population study based on 23,796 consecutive autopsies. World J Gastroenterol 2006; 12:2115–2119.
- Falanga A, Marchetti M, Vignoli A. Coagulation and cancer: biological and clinical aspects. J Thromb Haemost 2013; 11:223–233.
- Congly SE, Lee SS. Portal vein thrombosis: should anticoagulation be used? Curr Gastroenterol Rep 2013; 15:306.
- Kearon C, Akl EA, Comerota AJ, et al; American College of Chest Physicians. Antithrombotic therapy for VTE disease: Antithrombotic therapy and prevention of thrombosis, 9th ed: American College of Chest Physicians Evidence-Based Clinical Practice Guidelines. Chest 2012; 141(suppl 2):e419S–e494S.
- Uflacker R. Applications of percutaneous mechanical thrombectomy in transjugular intrahepatic portosystemic shunt and portal vein thrombosis. Tech Vasc Interv Radiol 2003; 6:59–69.
- Takahashi N, Kuroki K, Yanaga K. Percutaneous transhepatic mechanical thrombectomy for acute mesenteric venous thrombosis. J Endovasc Ther 2005; 12:508–511.
- Lopera JE, Correa G, Brazzini A, et al. Percutaneous transhepatic treatment of symptomatic mesenteric venous thrombosis. J Vasc Surg 2002; 36:1058–1061.
- Orr DW, Harrison PM, Devlin J, et al. Chronic mesenteric venous thrombosis: evaluation and determinants of survival during long-term follow-up. Clin Gastroenterol Hepatol 2007; 5:80–86.
- Condat B, Pessione F, Hillaire S, et al. Current outcome of portal vein thrombosis in adults: risk and benefit of anticoagulant therapy. Gastroenterology 2001; 120:490–497.
A 79-year-old man presented with chills and fever. He had a history of polymyalgia rheumatica and had been tapered off corticosteroids 1 month before admission. One week before he presented, he had developed generalized myalgia, chills, and fatigue. A cortisol stimulation test at that time was normal, prednisone was restarted, and his symptoms had improved. But 1 day before he presented, the chills had returned, this time with fever. Laboratory testing at an outpatient clinic had revealed abnormal liver enzyme levels.
On the day he presented, he felt worse, with persistent chills, fever, and vague lower abdominal pain, but he denied nausea, vomiting, changes in bowel habits, melena, hematochezia, and hematemesis. He was admitted for additional evaluation.
His medical history also included coronary artery disease (for which he had undergone coronary artery bypass grafting), hypertension, stable liver cysts, and gout. He had no known inflammatory bowel disease and no recent abdominal surgery. His medications included prednisone, atorvastatin, atenolol, aspirin, niacin, and cholecalciferol. He had no history of smoking, significant drinking, or use of illicit drugs. He had no respiratory or cardiac symptoms or neurologic symptoms consistent with a transient ischemic attack or stroke. He denied any rashes.
On admission, he was febrile, with temperatures reaching 102˚F (38.9˚C). His blood pressure was 137/63 mm Hg, pulse 54 beats per minute, respiration rate 18 breaths per minute, and oxygen saturation 97% on room air. A harsh systolic murmur was noted on physical examination. His abdomen was nondistended, nontender, and without bruits.
Laboratory testing (Table 1) revealed leukocytosis, anemia, mildly abnormal aminotransferase levels, elevated alkaline phosphatase, and markedly elevated C-reactive protein.
A full workup for fever was performed, including blood and urine cultures; chest radiography; contrast-enhanced computed tomography (CT) of the chest, abdomen, and pelvis; magnetic resonance imaging (MRI) of the abdomen; and colonoscopy. No source of infection—bacterial, viral, or fungal—was found. However, CT revealed new extensive thrombosis of the right portal vein and its branches (Figure 1).
CLINICAL PRESENTATION
1. Which of the following is least consistent with the clinical presentation of acute portal vein thrombosis?
- Abdominal pain
- Fever and chills
- Hematemesis
- Leukocytosis
- Absence of symptoms
Of these signs and symptoms, hematemesis is the least likely to be associated with acute portal vein thrombosis, although it can be associated with chronic cases.
Symptoms of portal vein thrombosis
Portal vein thrombosis causes extrahepatic obstruction of the portal venous system, which provides two-thirds of the total hepatic blood flow.
Acute. Often, thrombotic occlusion of the portal vein produces no acute symptoms because of immediate, compensatory vasodilation of the hepatic arterial system.1 Additionally, in the ensuing days, the thrombus becomes an organized collagenous plug, and collateral veins develop to bypass the blocked vein and maintain portal perfusion in a process called cavernous transformation.1,2 Thus, many patients have no symptoms.
If symptoms occur, portal vein thrombosis can initially present as transient abdominal pain with fever, as seen in this patient.3 Many patients with acute portal vein thrombosis experience abdominal pain due to intra-abdominal sepsis, also referred to as pylephlebitis.2,4 High, spiking fevers and chills also occur, caused by infected thrombi associated with intra-abdominal infections such as appendicitis, diverticulitis, and pancreatitis.5,6
Chronic. In contrast, symptomatic chronic portal vein thrombosis commonly presents with sequelae of portal hypertension, most notably gastrointestinal bleeding. Hematemesis from ruptured esophageal varices is the most frequent reason for seeking medical attention, though varices also develop in the stomach, duodenum, jejunum, gallbladder, and bile ducts.2,7 Abdominal pain is less common in chronic portal vein thrombosis unless the thrombus extends into the mesenteric veins and causes bowel ischemia or infarction. Long-standing portal vein thrombosis may also lead to dilated venous collaterals that compress large bile ducts, resulting in portal cholangiopathy.1,8
Portal vein thrombosis may present as acute intestinal ischemia and bowel infarction, though this is uncommon. This is generally seen with extensive occlusive portal vein thrombosis and concomitant mesenteric venous thrombosis.1,2
Other symptoms that are common but nonspecific are nausea, vomiting, diarrhea, weight loss, and anorexia.2
Signs of portal vein thrombosis
On examination, patients with acute portal vein thrombosis have minimal physical signs unless they have other contributing conditions. For example, acute portal vein thrombosis can result in abdominal distention secondary to ileus, or guarding and ascites secondary to intestinal infarction.3,9
Some patients with chronic portal vein thrombosis also have normal physical findings, but many have signs. Splenomegaly is seen in 75% to 100% of patients.2,7 Hepatomegaly, abdominal tenderness, and low-grade fever are common as well.2,10 Ascites is usually not present without underlying cirrhosis; however, mild and transient ascites can develop immediately after the thrombotic event before the patient develops collateral circulation.2
Laboratory testing for portal vein thrombosis
Laboratory test results are typically unremarkable. Liver function tests show preserved hepatic function but may reveal mild increases in aspartate aminotransferase (AST), alanine aminotransferase (ALT), alkaline phosphatase, and bilirubin.2,10
In acute cases, elevations of acute-phase reactant levels can occur.9 Leukocytosis and blood cultures growing Bacteroides species are seen in septic cases or pylephlebitis.11,12 There may be mild anemia, particularly after a recent bleeding episode, or mild leukopenia and thrombocytopenia due to hypersplenism. Suspicion of an underlying myeloproliferative disorder is high if thrombocytosis is present.2
DIAGNOSIS
2. All of the following would be appropriate initial diagnostic studies for portal vein thrombosis except which one?
- Doppler ultrasonography
- Contrast-enhanced CT
- Contrast-enhanced MRI
- Angiography
Portal vein thrombosis is most often diagnosed with noninvasive techniques, namely Doppler ultrasonography, CT, and MRI—not angiography.
Ultrasonography can reveal an echogenic thrombus in the vessel lumen with distention of the portal vein proximal to the occlusion and extensive collateral vessels. Plain ultrasonography fails to reveal the thrombus in up to one-third of patients. However, duplex ultrasonography with color flow Doppler imaging can confirm partial or complete absence of flow in the vein with 89% sensitivity and 92% specificity.13,14
On contrast-enhanced CT, the thrombus appears as a filling defect within the portal venous segment. Complete occlusion of the vein may produce a “train track” appearance due to contrast around the vessel.10 Without contrast, the clot will appear as hyperattenuating material in the portal vein, but contrast-enhanced imaging may be necessary to differentiate the thrombus from the vessel wall.15 Gas within the portal venous system is specific for pylephlebitis.4 Evidence of cavernous transformation is seen in chronic portal vein thrombosis.
Contrast-enhanced magnetic resonance angiography can also be used to evaluate patency and flow direction. In addition, it provides detailed anatomic information about the entire portal venous system, including the intrahepatic portal vessels, which is limited in CT imaging.2,10 CT and MRI can also help to identify predisposing conditions (eg, intra-abdominal infection, hepatocellular carcinoma) and complications (eg, intestinal infarction) associated with portal vein thrombosis.
Angiography can be considered if noninvasive techniques are inconclusive but is generally not necessary, given the increased use of CT and MRI.
In our patient, abdominal CT revealed occlusive thrombosis of the right portal vein and its branches (Figure 1). The left and main portal veins were patent. There was no evidence of intra-abdominal infection or infarction.
FINDING THE CAUSE
3. Which of the following is not a common cause
of portal vein thrombosis?
- A hypercoagulable state
- Immune deficiency
- Intra-abdominal infection
- Malignancy
- Portal hypertension
Once portal vein thrombosis has been diagnosed, the cause should be identified (Table 2). The differential diagnosis is broad, including both local factors (eg, injury to the portal vein, local inflammation, infection) and general factors (eg, inherited and acquired hypercoagulable conditions). Thrombophilias are identified in 60% of patients with portal vein thrombosis and local factors in 40%.7 Moreover, the etiology is often multifactorial. However, immune deficiency is not a common cause.
Hypercoagulability
Prothrombotic disorders can be either inherited or acquired.
Inherited deficiencies in the natural anticoagulants antithrombin, protein C, and protein S are associated with a high risk of thrombosis but have a low prevalence in the general population. In the setting of liver abnormalities, familial testing may be helpful to distinguish inherited causes of portal vein thrombosis from defective liver function as a consequence of portal vein thrombosis. The factor V Leiden mutation (G1691A) and the G20210A mutation in the prothrombin gene are more prevalent (> 2%) but generally confer a lower thrombosis risk.16 The prothrombin gene mutation G20210A is the most common risk factor for portal vein thrombosis, with prevalence of 2% to 22% in adults with nonmalignant, noncirrhotic portal vein thrombosis.3
Hyperhomocysteinemia due to a methylene tetrahydrofolate reductase (MTHFR) mutation (C677T) is another inherited associated risk factor for portal vein thrombosis, but hyperhomocysteinemia can also arise as a complication of portal vein thrombosis-related liver disease.3
Acquired prothrombotic disorders, particularly myeloproliferative diseases, are found in 22% to 48% of cases of portal vein thrombosis. Many young patients with myeloproliferative disorders present with portal vein thrombosis as the first symptom, and testing for the G1849T point mutation in JAK2 can make the diagnosis.17 Splenectomy with underlying myeloproliferative disorder confers a particularly high risk for portal vein thrombosis.18
Other thrombophilic disorders including antiphospholipid antibody syndrome, paroxysmal nocturnal hemoglobinuria, and malignancy can contribute to portal vein thrombosis.3 Pregnancy and oral contraceptive use have also been associated with hypercoagulability, and cessation of oral estrogen is recommended in such cases. The risk may be further increased in patients on oral contraceptives who have a previously unrecognized hypercoagulable state.3
Inflammation and infection
Inflammation and infection are local risk factors for portal vein thrombosis. Acute portal vein thrombosis has been associated with intra-abdominal infections (eg, appendicitis, cholecystitis) and with inflammatory conditions such as inflammatory bowel disease and pancreatitis.16,19 From 3% to 5% of all portal vein thrombosis cases result from pancreatitis, either from a single acute episode or from repeat inflammation of chronic pancreatitis.10 Portal vein thrombosis in the setting of inflammatory bowel disease can occur even when the disease is in remission, particularly in ulcerative colitis.20,21
Injury to the portal venous system
Abdominal surgery, particularly splenectomy, portosystemic shunting, colectomy, and blunt abdominal trauma can cause injury to the portal venous system, resulting in portal vein thrombosis. This is usually seen only in patients with portal hypertension, an underlying prothrombotic condition such as myeloproliferative disease, or inflammatory bowel disease.10,19,22
Impaired portal vein flow
Cirrhosis and malignancy are major risk factors for portal vein thrombosis. In case series, cirrhosis was found in 24% to 32% of patients with portal vein thrombosis.2,23 However, the overall prevalence of portal vein thrombosis in cirrhotic patients varies widely, from 0.6% to 28%, depending on the degree of cirrhosis.10
The pathogenesis of portal vein thrombosis in cirrhosis is unclear but may be multifactorial. Decreased portal blood flow (with subsequent stasis) and periportal lymphangitis and fibrosis are thought to stimulate thrombus formation.3,10 Additionally, patients with advanced cirrhosis are prothrombotic because of reduced hepatic synthesis of antithrombin, protein C, protein S, and coagulation factors.
Malignancy is associated with 21% to 24% of cases of portal vein thrombosis in adults, with pancreatic cancer and hepatocellular carcinoma being the most common.2,3 Others include cholangiocarcinoma and carcinomas of the stomach, lung, prostate, uterus, and kidney. Cancer causes portal vein thrombosis through a combination of tumor invasion into the portal vein, extrinsic compression by the tumor, periportal fibrosis following surgery or radiation, and hypercoagulability secondary to malignancy.9,16,24
Idiopathic portal vein thrombosis
Portal vein thrombosis is usually caused by one or more of the underlying factors mentioned above but is idiopathic in 8% to 15% of cases.10
Back to our patient
The cause of this patient’s portal vein thrombosis is unclear. He did not have a history of cirrhosis, inflammatory bowel disease, trauma, or abdominal surgery. His febrile illness could have precipitated the formation of a thrombus, but no definitive source of infection or inflammation was discovered. His workup was negative for pancreatitis, appendicitis, cholecystitis, diverticulitis, and prostatitis. No occult malignancy was found. It is also possible that his fever was the result of the thrombosis.
A full hypercoagulability panel revealed no striking abnormalities. He did have elevated fibrinogen and factor VIII levels that were consistent with an acute-phase reaction, along with an elevated erythrocyte sedimentation rate (> 90 mm/hr) and C-reactive protein level. Aside from the portal vein thrombosis, no potential source of inflammation could be identified.
Mildly reduced levels of antithrombin III activity were attributed to enoxaparin therapy and ultimately normalized on repeated testing. The patient had very minimally elevated titers of anticardiolipin immunoglobulin G (1:10 GPL) and anti-beta-2 glycoprotein immunoglobulin M (21 SMU), which were not thought to be significant. Tests for lupus anticoagulant, prothrombin gene mutation, activated protein C resistance, and JAK2 mutation were negative.
TREATMENT
4. Treatment of symptomatic portal vein thrombosis generally includes which two of the following?
- Anticoagulation
- Intravenous gamma globulin
- Broad-spectrum antibiotics
Anticoagulant therapy
Treatment of acute, symptomatic portal vein thrombosis involves anticoagulant therapy to prevent extension of the thrombus and, ultimately, to allow for recanalization of the obstructed veins. Anticoagulant therapy is initially intravenous unfractionated heparin or subcutaneous low-molecular-weight heparin, eventually bridged to an oral agent such as warfarin.3,9 Currently, there are inadequate data on the use of oral or parenteral factor Xa inhibitors or direct thrombin inhibitors in the treatment of this disease.
When started immediately, anticoagulation therapy is associated with complete recanalization in 38.3% and partial recanalization in 14% of patients presenting with complete thrombosis. Without anticoagulation, spontaneous recanalization is unusual.25
Although the optimal duration of anticoagulant therapy is unclear, a minimum of 3 months is generally recommended.9,26 If a hypercoagulable state is present or if the portal vein thrombosis is unprovoked (eg, by surgery, trauma, or an intra-abdominal infection), long-term treatment should be considered.26
Experience with thrombolytic therapy or mechanical recanalization has been limited, but the use of catheter-based techniques for pharmacomechanical thrombolysis has been reported.27–29 Transjugular intrahepatic portosystemic shunting is also an alternative to anticoagulation, but its role in treating portal vein thrombosis is complicated by technical difficulties of the procedure, postoperative complications, and recurrent occlusion of the shunt.25
Currently, there are no data comparing the risk-benefit ratio of early anticoagulation and that of invasive procedures. These more aggressive treatments are generally considered only when there is extensive thrombosis or ascites (which are both predictive factors of poor response to anticoagulation alone) and in patients for whom anticoagulation has failed.3 Surgical thrombectomy is rarely indicated, typically only in instances in which laparotomy is being performed for suspected bowel infarction.3
Antibiotics
In addition to anticoagulation, broad-spectrum antibiotics covering gram-negative and anaerobic bacteria are indicated for those cases of portal vein thrombosis associated with underlying infection.9
For chronic cases, the goals of management are to prevent and treat gastroesophageal variceal bleeding and to prevent recurrent thrombosis.9 Nonselective beta-blockers (eg, propranolol) and endoscopic band ligation have shown evidence of reducing the incidence of recurrent bleeding and prolonging survival in retrospective studies.9,30,31 Long-term anticoagulation is generally indicated to prevent further thrombosis and to increase the likelihood of recanalization only for patients with a permanent prothrombotic condition.9 In patients with clinically significant portal hypertension, the benefit of continued anticoagulation therapy must be weighed against the risk of esophageal and gastric variceal bleeding.
There is controversy regarding how to manage portal vein thrombosis that is incidentally identified and asymptomatic (eg, if it is discovered on an imaging study for another indication). Current guidelines recommend against anticoagulation in patients with incidentally discovered and asymptomatic splanchnic vein thrombosis, including portal vein thrombosis.26
Intravenous gamma globulin is not part of the treatment.
CASE CONTINUED
The patient’s presenting symptoms of fever, chills, and abdominal pain completely resolved after a course of antibiotic therapy. The erythrocyte sedimentation rate subsequently normalized and factor VIII activity improved. We believed that an underlying infectious or inflammatory process had contributed to the development of portal vein thrombosis, though the specific cause could not be identified. The patient was treated with enoxaparin 1 mg/kg twice a day and transitioned to warfarin.
Magnetic resonance venography done 3 months after diagnosis showed persistent right portal vein thrombosis that was largely unchanged. Anticoagulation was continued for 1 year with no change in his portal vein thrombosis on sequential imaging and was subsequently discontinued. To date, no malignancy or infectious process has been found, and the patient continues to do well 2 years later.
A 79-year-old man presented with chills and fever. He had a history of polymyalgia rheumatica and had been tapered off corticosteroids 1 month before admission. One week before he presented, he had developed generalized myalgia, chills, and fatigue. A cortisol stimulation test at that time was normal, prednisone was restarted, and his symptoms had improved. But 1 day before he presented, the chills had returned, this time with fever. Laboratory testing at an outpatient clinic had revealed abnormal liver enzyme levels.
On the day he presented, he felt worse, with persistent chills, fever, and vague lower abdominal pain, but he denied nausea, vomiting, changes in bowel habits, melena, hematochezia, and hematemesis. He was admitted for additional evaluation.
His medical history also included coronary artery disease (for which he had undergone coronary artery bypass grafting), hypertension, stable liver cysts, and gout. He had no known inflammatory bowel disease and no recent abdominal surgery. His medications included prednisone, atorvastatin, atenolol, aspirin, niacin, and cholecalciferol. He had no history of smoking, significant drinking, or use of illicit drugs. He had no respiratory or cardiac symptoms or neurologic symptoms consistent with a transient ischemic attack or stroke. He denied any rashes.
On admission, he was febrile, with temperatures reaching 102˚F (38.9˚C). His blood pressure was 137/63 mm Hg, pulse 54 beats per minute, respiration rate 18 breaths per minute, and oxygen saturation 97% on room air. A harsh systolic murmur was noted on physical examination. His abdomen was nondistended, nontender, and without bruits.
Laboratory testing (Table 1) revealed leukocytosis, anemia, mildly abnormal aminotransferase levels, elevated alkaline phosphatase, and markedly elevated C-reactive protein.
A full workup for fever was performed, including blood and urine cultures; chest radiography; contrast-enhanced computed tomography (CT) of the chest, abdomen, and pelvis; magnetic resonance imaging (MRI) of the abdomen; and colonoscopy. No source of infection—bacterial, viral, or fungal—was found. However, CT revealed new extensive thrombosis of the right portal vein and its branches (Figure 1).
CLINICAL PRESENTATION
1. Which of the following is least consistent with the clinical presentation of acute portal vein thrombosis?
- Abdominal pain
- Fever and chills
- Hematemesis
- Leukocytosis
- Absence of symptoms
Of these signs and symptoms, hematemesis is the least likely to be associated with acute portal vein thrombosis, although it can be associated with chronic cases.
Symptoms of portal vein thrombosis
Portal vein thrombosis causes extrahepatic obstruction of the portal venous system, which provides two-thirds of the total hepatic blood flow.
Acute. Often, thrombotic occlusion of the portal vein produces no acute symptoms because of immediate, compensatory vasodilation of the hepatic arterial system.1 Additionally, in the ensuing days, the thrombus becomes an organized collagenous plug, and collateral veins develop to bypass the blocked vein and maintain portal perfusion in a process called cavernous transformation.1,2 Thus, many patients have no symptoms.
If symptoms occur, portal vein thrombosis can initially present as transient abdominal pain with fever, as seen in this patient.3 Many patients with acute portal vein thrombosis experience abdominal pain due to intra-abdominal sepsis, also referred to as pylephlebitis.2,4 High, spiking fevers and chills also occur, caused by infected thrombi associated with intra-abdominal infections such as appendicitis, diverticulitis, and pancreatitis.5,6
Chronic. In contrast, symptomatic chronic portal vein thrombosis commonly presents with sequelae of portal hypertension, most notably gastrointestinal bleeding. Hematemesis from ruptured esophageal varices is the most frequent reason for seeking medical attention, though varices also develop in the stomach, duodenum, jejunum, gallbladder, and bile ducts.2,7 Abdominal pain is less common in chronic portal vein thrombosis unless the thrombus extends into the mesenteric veins and causes bowel ischemia or infarction. Long-standing portal vein thrombosis may also lead to dilated venous collaterals that compress large bile ducts, resulting in portal cholangiopathy.1,8
Portal vein thrombosis may present as acute intestinal ischemia and bowel infarction, though this is uncommon. This is generally seen with extensive occlusive portal vein thrombosis and concomitant mesenteric venous thrombosis.1,2
Other symptoms that are common but nonspecific are nausea, vomiting, diarrhea, weight loss, and anorexia.2
Signs of portal vein thrombosis
On examination, patients with acute portal vein thrombosis have minimal physical signs unless they have other contributing conditions. For example, acute portal vein thrombosis can result in abdominal distention secondary to ileus, or guarding and ascites secondary to intestinal infarction.3,9
Some patients with chronic portal vein thrombosis also have normal physical findings, but many have signs. Splenomegaly is seen in 75% to 100% of patients.2,7 Hepatomegaly, abdominal tenderness, and low-grade fever are common as well.2,10 Ascites is usually not present without underlying cirrhosis; however, mild and transient ascites can develop immediately after the thrombotic event before the patient develops collateral circulation.2
Laboratory testing for portal vein thrombosis
Laboratory test results are typically unremarkable. Liver function tests show preserved hepatic function but may reveal mild increases in aspartate aminotransferase (AST), alanine aminotransferase (ALT), alkaline phosphatase, and bilirubin.2,10
In acute cases, elevations of acute-phase reactant levels can occur.9 Leukocytosis and blood cultures growing Bacteroides species are seen in septic cases or pylephlebitis.11,12 There may be mild anemia, particularly after a recent bleeding episode, or mild leukopenia and thrombocytopenia due to hypersplenism. Suspicion of an underlying myeloproliferative disorder is high if thrombocytosis is present.2
DIAGNOSIS
2. All of the following would be appropriate initial diagnostic studies for portal vein thrombosis except which one?
- Doppler ultrasonography
- Contrast-enhanced CT
- Contrast-enhanced MRI
- Angiography
Portal vein thrombosis is most often diagnosed with noninvasive techniques, namely Doppler ultrasonography, CT, and MRI—not angiography.
Ultrasonography can reveal an echogenic thrombus in the vessel lumen with distention of the portal vein proximal to the occlusion and extensive collateral vessels. Plain ultrasonography fails to reveal the thrombus in up to one-third of patients. However, duplex ultrasonography with color flow Doppler imaging can confirm partial or complete absence of flow in the vein with 89% sensitivity and 92% specificity.13,14
On contrast-enhanced CT, the thrombus appears as a filling defect within the portal venous segment. Complete occlusion of the vein may produce a “train track” appearance due to contrast around the vessel.10 Without contrast, the clot will appear as hyperattenuating material in the portal vein, but contrast-enhanced imaging may be necessary to differentiate the thrombus from the vessel wall.15 Gas within the portal venous system is specific for pylephlebitis.4 Evidence of cavernous transformation is seen in chronic portal vein thrombosis.
Contrast-enhanced magnetic resonance angiography can also be used to evaluate patency and flow direction. In addition, it provides detailed anatomic information about the entire portal venous system, including the intrahepatic portal vessels, which is limited in CT imaging.2,10 CT and MRI can also help to identify predisposing conditions (eg, intra-abdominal infection, hepatocellular carcinoma) and complications (eg, intestinal infarction) associated with portal vein thrombosis.
Angiography can be considered if noninvasive techniques are inconclusive but is generally not necessary, given the increased use of CT and MRI.
In our patient, abdominal CT revealed occlusive thrombosis of the right portal vein and its branches (Figure 1). The left and main portal veins were patent. There was no evidence of intra-abdominal infection or infarction.
FINDING THE CAUSE
3. Which of the following is not a common cause
of portal vein thrombosis?
- A hypercoagulable state
- Immune deficiency
- Intra-abdominal infection
- Malignancy
- Portal hypertension
Once portal vein thrombosis has been diagnosed, the cause should be identified (Table 2). The differential diagnosis is broad, including both local factors (eg, injury to the portal vein, local inflammation, infection) and general factors (eg, inherited and acquired hypercoagulable conditions). Thrombophilias are identified in 60% of patients with portal vein thrombosis and local factors in 40%.7 Moreover, the etiology is often multifactorial. However, immune deficiency is not a common cause.
Hypercoagulability
Prothrombotic disorders can be either inherited or acquired.
Inherited deficiencies in the natural anticoagulants antithrombin, protein C, and protein S are associated with a high risk of thrombosis but have a low prevalence in the general population. In the setting of liver abnormalities, familial testing may be helpful to distinguish inherited causes of portal vein thrombosis from defective liver function as a consequence of portal vein thrombosis. The factor V Leiden mutation (G1691A) and the G20210A mutation in the prothrombin gene are more prevalent (> 2%) but generally confer a lower thrombosis risk.16 The prothrombin gene mutation G20210A is the most common risk factor for portal vein thrombosis, with prevalence of 2% to 22% in adults with nonmalignant, noncirrhotic portal vein thrombosis.3
Hyperhomocysteinemia due to a methylene tetrahydrofolate reductase (MTHFR) mutation (C677T) is another inherited associated risk factor for portal vein thrombosis, but hyperhomocysteinemia can also arise as a complication of portal vein thrombosis-related liver disease.3
Acquired prothrombotic disorders, particularly myeloproliferative diseases, are found in 22% to 48% of cases of portal vein thrombosis. Many young patients with myeloproliferative disorders present with portal vein thrombosis as the first symptom, and testing for the G1849T point mutation in JAK2 can make the diagnosis.17 Splenectomy with underlying myeloproliferative disorder confers a particularly high risk for portal vein thrombosis.18
Other thrombophilic disorders including antiphospholipid antibody syndrome, paroxysmal nocturnal hemoglobinuria, and malignancy can contribute to portal vein thrombosis.3 Pregnancy and oral contraceptive use have also been associated with hypercoagulability, and cessation of oral estrogen is recommended in such cases. The risk may be further increased in patients on oral contraceptives who have a previously unrecognized hypercoagulable state.3
Inflammation and infection
Inflammation and infection are local risk factors for portal vein thrombosis. Acute portal vein thrombosis has been associated with intra-abdominal infections (eg, appendicitis, cholecystitis) and with inflammatory conditions such as inflammatory bowel disease and pancreatitis.16,19 From 3% to 5% of all portal vein thrombosis cases result from pancreatitis, either from a single acute episode or from repeat inflammation of chronic pancreatitis.10 Portal vein thrombosis in the setting of inflammatory bowel disease can occur even when the disease is in remission, particularly in ulcerative colitis.20,21
Injury to the portal venous system
Abdominal surgery, particularly splenectomy, portosystemic shunting, colectomy, and blunt abdominal trauma can cause injury to the portal venous system, resulting in portal vein thrombosis. This is usually seen only in patients with portal hypertension, an underlying prothrombotic condition such as myeloproliferative disease, or inflammatory bowel disease.10,19,22
Impaired portal vein flow
Cirrhosis and malignancy are major risk factors for portal vein thrombosis. In case series, cirrhosis was found in 24% to 32% of patients with portal vein thrombosis.2,23 However, the overall prevalence of portal vein thrombosis in cirrhotic patients varies widely, from 0.6% to 28%, depending on the degree of cirrhosis.10
The pathogenesis of portal vein thrombosis in cirrhosis is unclear but may be multifactorial. Decreased portal blood flow (with subsequent stasis) and periportal lymphangitis and fibrosis are thought to stimulate thrombus formation.3,10 Additionally, patients with advanced cirrhosis are prothrombotic because of reduced hepatic synthesis of antithrombin, protein C, protein S, and coagulation factors.
Malignancy is associated with 21% to 24% of cases of portal vein thrombosis in adults, with pancreatic cancer and hepatocellular carcinoma being the most common.2,3 Others include cholangiocarcinoma and carcinomas of the stomach, lung, prostate, uterus, and kidney. Cancer causes portal vein thrombosis through a combination of tumor invasion into the portal vein, extrinsic compression by the tumor, periportal fibrosis following surgery or radiation, and hypercoagulability secondary to malignancy.9,16,24
Idiopathic portal vein thrombosis
Portal vein thrombosis is usually caused by one or more of the underlying factors mentioned above but is idiopathic in 8% to 15% of cases.10
Back to our patient
The cause of this patient’s portal vein thrombosis is unclear. He did not have a history of cirrhosis, inflammatory bowel disease, trauma, or abdominal surgery. His febrile illness could have precipitated the formation of a thrombus, but no definitive source of infection or inflammation was discovered. His workup was negative for pancreatitis, appendicitis, cholecystitis, diverticulitis, and prostatitis. No occult malignancy was found. It is also possible that his fever was the result of the thrombosis.
A full hypercoagulability panel revealed no striking abnormalities. He did have elevated fibrinogen and factor VIII levels that were consistent with an acute-phase reaction, along with an elevated erythrocyte sedimentation rate (> 90 mm/hr) and C-reactive protein level. Aside from the portal vein thrombosis, no potential source of inflammation could be identified.
Mildly reduced levels of antithrombin III activity were attributed to enoxaparin therapy and ultimately normalized on repeated testing. The patient had very minimally elevated titers of anticardiolipin immunoglobulin G (1:10 GPL) and anti-beta-2 glycoprotein immunoglobulin M (21 SMU), which were not thought to be significant. Tests for lupus anticoagulant, prothrombin gene mutation, activated protein C resistance, and JAK2 mutation were negative.
TREATMENT
4. Treatment of symptomatic portal vein thrombosis generally includes which two of the following?
- Anticoagulation
- Intravenous gamma globulin
- Broad-spectrum antibiotics
Anticoagulant therapy
Treatment of acute, symptomatic portal vein thrombosis involves anticoagulant therapy to prevent extension of the thrombus and, ultimately, to allow for recanalization of the obstructed veins. Anticoagulant therapy is initially intravenous unfractionated heparin or subcutaneous low-molecular-weight heparin, eventually bridged to an oral agent such as warfarin.3,9 Currently, there are inadequate data on the use of oral or parenteral factor Xa inhibitors or direct thrombin inhibitors in the treatment of this disease.
When started immediately, anticoagulation therapy is associated with complete recanalization in 38.3% and partial recanalization in 14% of patients presenting with complete thrombosis. Without anticoagulation, spontaneous recanalization is unusual.25
Although the optimal duration of anticoagulant therapy is unclear, a minimum of 3 months is generally recommended.9,26 If a hypercoagulable state is present or if the portal vein thrombosis is unprovoked (eg, by surgery, trauma, or an intra-abdominal infection), long-term treatment should be considered.26
Experience with thrombolytic therapy or mechanical recanalization has been limited, but the use of catheter-based techniques for pharmacomechanical thrombolysis has been reported.27–29 Transjugular intrahepatic portosystemic shunting is also an alternative to anticoagulation, but its role in treating portal vein thrombosis is complicated by technical difficulties of the procedure, postoperative complications, and recurrent occlusion of the shunt.25
Currently, there are no data comparing the risk-benefit ratio of early anticoagulation and that of invasive procedures. These more aggressive treatments are generally considered only when there is extensive thrombosis or ascites (which are both predictive factors of poor response to anticoagulation alone) and in patients for whom anticoagulation has failed.3 Surgical thrombectomy is rarely indicated, typically only in instances in which laparotomy is being performed for suspected bowel infarction.3
Antibiotics
In addition to anticoagulation, broad-spectrum antibiotics covering gram-negative and anaerobic bacteria are indicated for those cases of portal vein thrombosis associated with underlying infection.9
For chronic cases, the goals of management are to prevent and treat gastroesophageal variceal bleeding and to prevent recurrent thrombosis.9 Nonselective beta-blockers (eg, propranolol) and endoscopic band ligation have shown evidence of reducing the incidence of recurrent bleeding and prolonging survival in retrospective studies.9,30,31 Long-term anticoagulation is generally indicated to prevent further thrombosis and to increase the likelihood of recanalization only for patients with a permanent prothrombotic condition.9 In patients with clinically significant portal hypertension, the benefit of continued anticoagulation therapy must be weighed against the risk of esophageal and gastric variceal bleeding.
There is controversy regarding how to manage portal vein thrombosis that is incidentally identified and asymptomatic (eg, if it is discovered on an imaging study for another indication). Current guidelines recommend against anticoagulation in patients with incidentally discovered and asymptomatic splanchnic vein thrombosis, including portal vein thrombosis.26
Intravenous gamma globulin is not part of the treatment.
CASE CONTINUED
The patient’s presenting symptoms of fever, chills, and abdominal pain completely resolved after a course of antibiotic therapy. The erythrocyte sedimentation rate subsequently normalized and factor VIII activity improved. We believed that an underlying infectious or inflammatory process had contributed to the development of portal vein thrombosis, though the specific cause could not be identified. The patient was treated with enoxaparin 1 mg/kg twice a day and transitioned to warfarin.
Magnetic resonance venography done 3 months after diagnosis showed persistent right portal vein thrombosis that was largely unchanged. Anticoagulation was continued for 1 year with no change in his portal vein thrombosis on sequential imaging and was subsequently discontinued. To date, no malignancy or infectious process has been found, and the patient continues to do well 2 years later.
- Ponziani FR, Zocco MA, Campanale C, et al. Portal vein thrombosis: insight into physiopathology, diagnosis, and treatment. World J Gastroenterol 2010; 16:143–155.
- Cohen J, Edelman RR, Chopra S. Portal vein thrombosis: a review. Am J Med 1992; 92:173–182.
- Primignani M. Portal vein thrombosis, revisited. Dig Liver Dis 2010; 42:163–170.
- Condat B, Valla D. Nonmalignant portal vein thrombosis in adults. Nat Clin Pract Gastroenterol Hepatol 2006; 3:505–515.
- Condat B, Pessione F, Helene Denninger M, Hillaire S, Valla D. Recent portal or mesenteric venous thrombosis: increased recognition and frequent recanalization on anticoagulant therapy. Hepatology 2000; 32:466–470.
- Sheen CL, Lamparelli H, Milne A, Green I, Ramage JK. Clinical features, diagnosis and outcome of acute portal vein thrombosis. QJM 2000; 93:531–534.
- Sogaard KK, Astrup LB, Vilstrup H, Gronbaek H. Portal vein thrombosis; risk factors, clinical presentation and treatment. BMC Gastroenterol 2007; 7:34.
- Llop E, de Juan C, Seijo S, et al. Portal cholangiopathy: radiological classification and natural history. Gut 2011; 60:853–860.
- DeLeve LD, Valla DC, Garcia-Tsao G; American Association for the Study of Liver Diseases. Vascular disorders of the liver. Hepatology 2009; 49:1729–1764.
- Sobhonslidsuk A, Reddy KR. Portal vein thrombosis: a concise review. Am J Gastroenterol 2002; 97:535–541.
- Ni YH, Wang NC, Peng MY, Chou YY, Chang FY. Bacteroides fragilis bacteremia associated with portal vein and superior mesentery vein thrombosis secondary to antithrombin III and protein C deficiency: a case report. J Microbiol Immunol Infect 2002; 35:255–258.
- Trum J, Valla D, Cohen G, et al. Bacteroides bacteraemia of undetermined origin: strong association with portal vein thrombosis and cryptogenic pylephlebitis. Eur J Gastroenterol Hepatol 1993; 5:655–659.
- Ueno N, Sasaki A, Tomiyama T, Tano S, Kimura K. Color Doppler ultrasonography in the diagnosis of cavernous transformation of the portal vein. J Clin Ultrasound 1997; 25:227–233.
- Tessler FN, Gehring BJ, Gomes AS, et al. Diagnosis of portal vein thrombosis: value of color Doppler imaging. AJR Am J Roentgenol 1991; 157:293–296.
- Hidajat N, Stobbe H, Griesshaber V, Felix R, Schroder RJ. Imaging and radiological interventions of portal vein thrombosis. Acta Radiol 2005; 46:336–343.
- Valla DC, Condat B. Portal vein thrombosis in adults: pathophysiology, pathogenesis and management. J Hepatol 2000; 32:865–871.
- Kralovics R, Passamonti F, Buser AS, et al. A gain-of-function mutation of JAK2 in myeloproliferative disorders. N Engl J Med 2005; 352:1779–1790.
- Krauth MT, Lechner K, Neugebauer EA, Pabinger I. The postoperative splenic/portal vein thrombosis after splenectomy and its prevention—an unresolved issue. Haematologica 2008; 93:1227–1232.
- Sinagra E, Aragona E, Romano C, et al. The role of portal vein thrombosis in the clinical course of inflammatory bowel diseases: report on three cases and review of the literature. Gastroenterol Res Pract 2012; 2012:916428.
- Maconi G, Bolzacchini E, Dell’Era A, Russo U, Ardizzone S, de Franchis R. Portal vein thrombosis in inflammatory bowel diseases: a single-center case series. J Crohns Colitis 2012; 6:362–367.
- Jackson LM, O’Gorman PJ, O’Connell J, Cronin CC, Cotter KP, Shanahan F. Thrombosis in inflammatory bowel disease: clinical setting, procoagulant profile and factor V Leiden. QJM 1997; 90:183–188.
- Eguchi A, Hashizume M, Kitano S, Tanoue K, Wada H, Sugimachi K. High rate of portal thrombosis after splenectomy in patients with esophageal varices and idiopathic portal hypertension. Arch Surg 1991; 126:752–755.
- Ogren M, Bergqvist D, Björck M, Acosta S, Eriksson H, Sternby NH. Portal vein thrombosis: prevalence, patient characteristics and lifetime risk: a population study based on 23,796 consecutive autopsies. World J Gastroenterol 2006; 12:2115–2119.
- Falanga A, Marchetti M, Vignoli A. Coagulation and cancer: biological and clinical aspects. J Thromb Haemost 2013; 11:223–233.
- Congly SE, Lee SS. Portal vein thrombosis: should anticoagulation be used? Curr Gastroenterol Rep 2013; 15:306.
- Kearon C, Akl EA, Comerota AJ, et al; American College of Chest Physicians. Antithrombotic therapy for VTE disease: Antithrombotic therapy and prevention of thrombosis, 9th ed: American College of Chest Physicians Evidence-Based Clinical Practice Guidelines. Chest 2012; 141(suppl 2):e419S–e494S.
- Uflacker R. Applications of percutaneous mechanical thrombectomy in transjugular intrahepatic portosystemic shunt and portal vein thrombosis. Tech Vasc Interv Radiol 2003; 6:59–69.
- Takahashi N, Kuroki K, Yanaga K. Percutaneous transhepatic mechanical thrombectomy for acute mesenteric venous thrombosis. J Endovasc Ther 2005; 12:508–511.
- Lopera JE, Correa G, Brazzini A, et al. Percutaneous transhepatic treatment of symptomatic mesenteric venous thrombosis. J Vasc Surg 2002; 36:1058–1061.
- Orr DW, Harrison PM, Devlin J, et al. Chronic mesenteric venous thrombosis: evaluation and determinants of survival during long-term follow-up. Clin Gastroenterol Hepatol 2007; 5:80–86.
- Condat B, Pessione F, Hillaire S, et al. Current outcome of portal vein thrombosis in adults: risk and benefit of anticoagulant therapy. Gastroenterology 2001; 120:490–497.
- Ponziani FR, Zocco MA, Campanale C, et al. Portal vein thrombosis: insight into physiopathology, diagnosis, and treatment. World J Gastroenterol 2010; 16:143–155.
- Cohen J, Edelman RR, Chopra S. Portal vein thrombosis: a review. Am J Med 1992; 92:173–182.
- Primignani M. Portal vein thrombosis, revisited. Dig Liver Dis 2010; 42:163–170.
- Condat B, Valla D. Nonmalignant portal vein thrombosis in adults. Nat Clin Pract Gastroenterol Hepatol 2006; 3:505–515.
- Condat B, Pessione F, Helene Denninger M, Hillaire S, Valla D. Recent portal or mesenteric venous thrombosis: increased recognition and frequent recanalization on anticoagulant therapy. Hepatology 2000; 32:466–470.
- Sheen CL, Lamparelli H, Milne A, Green I, Ramage JK. Clinical features, diagnosis and outcome of acute portal vein thrombosis. QJM 2000; 93:531–534.
- Sogaard KK, Astrup LB, Vilstrup H, Gronbaek H. Portal vein thrombosis; risk factors, clinical presentation and treatment. BMC Gastroenterol 2007; 7:34.
- Llop E, de Juan C, Seijo S, et al. Portal cholangiopathy: radiological classification and natural history. Gut 2011; 60:853–860.
- DeLeve LD, Valla DC, Garcia-Tsao G; American Association for the Study of Liver Diseases. Vascular disorders of the liver. Hepatology 2009; 49:1729–1764.
- Sobhonslidsuk A, Reddy KR. Portal vein thrombosis: a concise review. Am J Gastroenterol 2002; 97:535–541.
- Ni YH, Wang NC, Peng MY, Chou YY, Chang FY. Bacteroides fragilis bacteremia associated with portal vein and superior mesentery vein thrombosis secondary to antithrombin III and protein C deficiency: a case report. J Microbiol Immunol Infect 2002; 35:255–258.
- Trum J, Valla D, Cohen G, et al. Bacteroides bacteraemia of undetermined origin: strong association with portal vein thrombosis and cryptogenic pylephlebitis. Eur J Gastroenterol Hepatol 1993; 5:655–659.
- Ueno N, Sasaki A, Tomiyama T, Tano S, Kimura K. Color Doppler ultrasonography in the diagnosis of cavernous transformation of the portal vein. J Clin Ultrasound 1997; 25:227–233.
- Tessler FN, Gehring BJ, Gomes AS, et al. Diagnosis of portal vein thrombosis: value of color Doppler imaging. AJR Am J Roentgenol 1991; 157:293–296.
- Hidajat N, Stobbe H, Griesshaber V, Felix R, Schroder RJ. Imaging and radiological interventions of portal vein thrombosis. Acta Radiol 2005; 46:336–343.
- Valla DC, Condat B. Portal vein thrombosis in adults: pathophysiology, pathogenesis and management. J Hepatol 2000; 32:865–871.
- Kralovics R, Passamonti F, Buser AS, et al. A gain-of-function mutation of JAK2 in myeloproliferative disorders. N Engl J Med 2005; 352:1779–1790.
- Krauth MT, Lechner K, Neugebauer EA, Pabinger I. The postoperative splenic/portal vein thrombosis after splenectomy and its prevention—an unresolved issue. Haematologica 2008; 93:1227–1232.
- Sinagra E, Aragona E, Romano C, et al. The role of portal vein thrombosis in the clinical course of inflammatory bowel diseases: report on three cases and review of the literature. Gastroenterol Res Pract 2012; 2012:916428.
- Maconi G, Bolzacchini E, Dell’Era A, Russo U, Ardizzone S, de Franchis R. Portal vein thrombosis in inflammatory bowel diseases: a single-center case series. J Crohns Colitis 2012; 6:362–367.
- Jackson LM, O’Gorman PJ, O’Connell J, Cronin CC, Cotter KP, Shanahan F. Thrombosis in inflammatory bowel disease: clinical setting, procoagulant profile and factor V Leiden. QJM 1997; 90:183–188.
- Eguchi A, Hashizume M, Kitano S, Tanoue K, Wada H, Sugimachi K. High rate of portal thrombosis after splenectomy in patients with esophageal varices and idiopathic portal hypertension. Arch Surg 1991; 126:752–755.
- Ogren M, Bergqvist D, Björck M, Acosta S, Eriksson H, Sternby NH. Portal vein thrombosis: prevalence, patient characteristics and lifetime risk: a population study based on 23,796 consecutive autopsies. World J Gastroenterol 2006; 12:2115–2119.
- Falanga A, Marchetti M, Vignoli A. Coagulation and cancer: biological and clinical aspects. J Thromb Haemost 2013; 11:223–233.
- Congly SE, Lee SS. Portal vein thrombosis: should anticoagulation be used? Curr Gastroenterol Rep 2013; 15:306.
- Kearon C, Akl EA, Comerota AJ, et al; American College of Chest Physicians. Antithrombotic therapy for VTE disease: Antithrombotic therapy and prevention of thrombosis, 9th ed: American College of Chest Physicians Evidence-Based Clinical Practice Guidelines. Chest 2012; 141(suppl 2):e419S–e494S.
- Uflacker R. Applications of percutaneous mechanical thrombectomy in transjugular intrahepatic portosystemic shunt and portal vein thrombosis. Tech Vasc Interv Radiol 2003; 6:59–69.
- Takahashi N, Kuroki K, Yanaga K. Percutaneous transhepatic mechanical thrombectomy for acute mesenteric venous thrombosis. J Endovasc Ther 2005; 12:508–511.
- Lopera JE, Correa G, Brazzini A, et al. Percutaneous transhepatic treatment of symptomatic mesenteric venous thrombosis. J Vasc Surg 2002; 36:1058–1061.
- Orr DW, Harrison PM, Devlin J, et al. Chronic mesenteric venous thrombosis: evaluation and determinants of survival during long-term follow-up. Clin Gastroenterol Hepatol 2007; 5:80–86.
- Condat B, Pessione F, Hillaire S, et al. Current outcome of portal vein thrombosis in adults: risk and benefit of anticoagulant therapy. Gastroenterology 2001; 120:490–497.
When does pericarditis merit a workup for autoimmune or inflammatory disease?
Pericarditis is in most cases a one-time disease simply treated with anti-inflammatory drugs. It requires no extensive workup for systemic inflammatory or autoimmune disease. Further evaluation is required for patients who have recurrent pericarditis resistant to conventional therapy or pericarditis with manifestations of systemic disease.
ACUTE PERICARDITIS
Pericardial disease has different presentations: acute, recurrent, constrictive, effusive-constrictive, and pericardial effusion with or without tamponade. Acute pericarditis is the most common of these and can affect people of all ages. The typical acute manifestations are chest pain (usually pleuritic), a pericardial friction rub, and widespread ST-segment elevation on the electrocardiogram.1,2 The chest pain tends to be sharp and long-lasting; it radiates to the trapezius ridge and increases during respiration or body movements.
Acute pericarditis usually responds to an anti-inflammatory drug such as colchicine 0.6 mg/day for 3 months, a nonsteroidal anti-inflammatory drug such as ibuprofen 600 mg three times a day for 10 days, and in advanced resistant cases, an oral corticosteroid.3,4
Most often, pericarditis is either idiopathic or occurs after a respiratory viral illness. Much less common causes include bacterial infection, postpericardiotomy syndrome, myocardial infarction, primary or metastatic tumors, trauma, radiation, and uremia. However, pericarditis can also be part of the presentation of systemic inflammatory and autoimmune diseases such as rheumatoid arthritis and systemic lupus erythematosus; hereditary periodic fever syndromes such as familial Mediterranean fever; and systemic-onset juvenile idiopathic arthritis.1,5
In acute pericarditis, a complex workup is usually not justified, since the results will have limited usefulness in the clinical management of the patient. It is most often diagnosed by the presenting symptoms, auscultation, electrocardiography, echocardiography, and chest radiography, and by additional basic tests that include a complete blood cell count, complete metabolic profile, erythrocyte sedimentation rate, and C-reactive protein level. However, if pericarditis does not respond to anti-inflammatory treatment and if an autoimmune or infectious disease is suspected, further evaluation may include antinuclear antibody testing and testing for human immunodeficiency virus and tuberculosis. If the diagnosis of acute pericarditis remains uncertain, cardiac magnetic resonance imaging (MRI) may be useful.
RECURRENT PERICARDITIS
Although acute pericarditis most often has a benign course and responds well to anti-inflammatory drugs, 20% to 30% of patients who have a first attack of acute pericarditis have a recurrence, and up to 50% of patients who have one recurrence will have another.3,4
Disease activity can be followed with serial testing of inflammatory markers—eg, erythrocyte sedimentation rate and C-reactive protein level. Echocardiography, cardiac computed tomography, and cardiac MRI can characterize active inflammation, edema, pericardial thickness, and pericardial effusion.6–8
Recurrent pericarditis is often resistant to standard therapy and requires corticosteroids in high doses, which paradoxically can increase the risk of recurrence. Therefore, further workup for underlying autoimmune disease, systemic inflammatory disease, or infection is necessary. More potent immunosuppressive therapy may be required, not only in pericarditis associated with systemic autoimmune or inflammatory conditions, but even in idiopathic recurrent pericarditis, either to control symptoms or to mitigate the effects of corticosteroids.
SYSTEMIC INFLAMMATION
The true prevalence of pericardial disease in most systemic inflammatory and autoimmune diseases is difficult to determine from current data. But advances in serologic testing and imaging techniques have shown cardiac involvement in a number of inflammatory diseases.9
In one study, a serologic autoimmune workup in patients with acute pericarditis found that 2% had collagen vascular disease.9 Pericardial involvement is likely in systemic lupus erythematosus,10 and a postmortem study of patients with systemic sclerosis found that 72% had pericarditis.11 Mixed connective tissue disease has been associated with pericarditis in 29% of cases and 56% in autopsy studies.12,13 Pericarditis may be the initial manifestation of vasculitis—eg, Takayasu arteritis or granulomatosis with polyangiitis (formerly known as Wegener granulomatosis).
Other diseases with pericardial involvement include Still disease, Sjögren syndrome, sarcoidosis, and inflammatory bowel disease. Symptomatic pericarditis occurs in about 25% of patients with Sjögren syndrome and asymptomatic pericardial involvement in more than half. Autopsy studies reported pericardial involvement in up to 80% of patients with systemic lupus erythematosus. Cardiac tamponade occurs in fewer than 2%, and constrictive pericarditis is extremely rare.5,9–11
RECOMMENDATIONS
Patients with a first episode of pericarditis should be treated with an anti-inflammatory medication, with no comprehensive testing for autoimmune disease. An evaluation for autoimmune and infectious disease should be carried out in patients with fever (temperature > 38°C; 100.4°F), recurrent pericarditis, recurrent large pericardial effusion or tamponade, or night sweats despite conventional medical therapy. Signs of systemic disease such as renal failure, elevated liver enzymes, or skin rash merit further evaluation.
Prospective studies using appropriate serologic testing and imaging are needed to determine the correlation between myopericardial involvement and inflammatory diseases because of increased morbidity and mortality in several of these diseases.
- Troughton RW, Asher CR, Klein AL. Pericarditis. Lancet 2004; 363:717–727.
- Alraies MC, Klein AL. Should we still use electrocardiography to diagnose pericardial disease? Cleve Clin J Med 2013; 80:97–100.
- Imazio M, Brucato A, Cemin R, et al; ICAP Investigators. A randomized trial of colchicine for acute pericarditis. N Engl J Med 2013; 369:1522–1528.
- Imazio M, Cecchi E, Demichelis B, et al. Indicators of poor prognosis of acute pericarditis. Circulation 2007; 115:2739–2744.
- Zayas R, Anguita M, Torres F, et al. Incidence of specific etiology and role of methods for specific etiologic diagnosis of primary acute pericarditis. Am J Cardiol 1995; 75:378–382.
- Verhaert D, Gabriel RS, Johnston D, Lytle BW, Desai MY, Klein AL. The role of multimodality imaging in the management of pericardial disease. Circ Cardiovasc Imaging 2010; 3:333–343.
- Klein AL, Abbara S, Agler DA, et al. American Society of Echocardiography clinical recommendations for multimodality cardiovascular imaging of patients with pericardial disease: endorsed by the Society for Cardiovascular Magnetic Resonance and Society of Cardiovascular Computed Tomography. J Am Soc Echocardiogr 2013; 26:965–1012.e15.
- Yingchoncharoen T, Alraies MC, Kwon DH, Rodriguez ER, Tan CD, Klein AL. Emerging role of multimodality imaging in management of inflammatory pericardial diseases. Expert Rev Cardiovasc Ther 2013; 11:1211–1225.
- Knockaert DC. Cardiac involvement in systemic inflammatory diseases. Eur Heart J 2007; 28:1797–1804.
- Doria A, Iaccarino L, Sarzi-Puttini P, Atzeni F, Turriel M, Petri M. Cardiac involvement in systemic lupus erythematosus. Lupus 2005; 14:683–686.
- Byers RJ, Marshall DA, Freemont AJ. Pericardial involvement in systemic sclerosis. Ann Rheum Dis 1997; 56:393–394.
- Kasukawa R. Mixed connective tissue disease. Intern Med 1999; 38:386–393.
- Bezerra MC, Saraiva F Jr, Carvalho JF, Caleiro MT, Goncalves CR, Borba EF. Cardiac tamponade due to massive pericardial effusion in mixed connective tissue disease: reversal with steroid therapy. Lupus 2004; 13:618–620.
Pericarditis is in most cases a one-time disease simply treated with anti-inflammatory drugs. It requires no extensive workup for systemic inflammatory or autoimmune disease. Further evaluation is required for patients who have recurrent pericarditis resistant to conventional therapy or pericarditis with manifestations of systemic disease.
ACUTE PERICARDITIS
Pericardial disease has different presentations: acute, recurrent, constrictive, effusive-constrictive, and pericardial effusion with or without tamponade. Acute pericarditis is the most common of these and can affect people of all ages. The typical acute manifestations are chest pain (usually pleuritic), a pericardial friction rub, and widespread ST-segment elevation on the electrocardiogram.1,2 The chest pain tends to be sharp and long-lasting; it radiates to the trapezius ridge and increases during respiration or body movements.
Acute pericarditis usually responds to an anti-inflammatory drug such as colchicine 0.6 mg/day for 3 months, a nonsteroidal anti-inflammatory drug such as ibuprofen 600 mg three times a day for 10 days, and in advanced resistant cases, an oral corticosteroid.3,4
Most often, pericarditis is either idiopathic or occurs after a respiratory viral illness. Much less common causes include bacterial infection, postpericardiotomy syndrome, myocardial infarction, primary or metastatic tumors, trauma, radiation, and uremia. However, pericarditis can also be part of the presentation of systemic inflammatory and autoimmune diseases such as rheumatoid arthritis and systemic lupus erythematosus; hereditary periodic fever syndromes such as familial Mediterranean fever; and systemic-onset juvenile idiopathic arthritis.1,5
In acute pericarditis, a complex workup is usually not justified, since the results will have limited usefulness in the clinical management of the patient. It is most often diagnosed by the presenting symptoms, auscultation, electrocardiography, echocardiography, and chest radiography, and by additional basic tests that include a complete blood cell count, complete metabolic profile, erythrocyte sedimentation rate, and C-reactive protein level. However, if pericarditis does not respond to anti-inflammatory treatment and if an autoimmune or infectious disease is suspected, further evaluation may include antinuclear antibody testing and testing for human immunodeficiency virus and tuberculosis. If the diagnosis of acute pericarditis remains uncertain, cardiac magnetic resonance imaging (MRI) may be useful.
RECURRENT PERICARDITIS
Although acute pericarditis most often has a benign course and responds well to anti-inflammatory drugs, 20% to 30% of patients who have a first attack of acute pericarditis have a recurrence, and up to 50% of patients who have one recurrence will have another.3,4
Disease activity can be followed with serial testing of inflammatory markers—eg, erythrocyte sedimentation rate and C-reactive protein level. Echocardiography, cardiac computed tomography, and cardiac MRI can characterize active inflammation, edema, pericardial thickness, and pericardial effusion.6–8
Recurrent pericarditis is often resistant to standard therapy and requires corticosteroids in high doses, which paradoxically can increase the risk of recurrence. Therefore, further workup for underlying autoimmune disease, systemic inflammatory disease, or infection is necessary. More potent immunosuppressive therapy may be required, not only in pericarditis associated with systemic autoimmune or inflammatory conditions, but even in idiopathic recurrent pericarditis, either to control symptoms or to mitigate the effects of corticosteroids.
SYSTEMIC INFLAMMATION
The true prevalence of pericardial disease in most systemic inflammatory and autoimmune diseases is difficult to determine from current data. But advances in serologic testing and imaging techniques have shown cardiac involvement in a number of inflammatory diseases.9
In one study, a serologic autoimmune workup in patients with acute pericarditis found that 2% had collagen vascular disease.9 Pericardial involvement is likely in systemic lupus erythematosus,10 and a postmortem study of patients with systemic sclerosis found that 72% had pericarditis.11 Mixed connective tissue disease has been associated with pericarditis in 29% of cases and 56% in autopsy studies.12,13 Pericarditis may be the initial manifestation of vasculitis—eg, Takayasu arteritis or granulomatosis with polyangiitis (formerly known as Wegener granulomatosis).
Other diseases with pericardial involvement include Still disease, Sjögren syndrome, sarcoidosis, and inflammatory bowel disease. Symptomatic pericarditis occurs in about 25% of patients with Sjögren syndrome and asymptomatic pericardial involvement in more than half. Autopsy studies reported pericardial involvement in up to 80% of patients with systemic lupus erythematosus. Cardiac tamponade occurs in fewer than 2%, and constrictive pericarditis is extremely rare.5,9–11
RECOMMENDATIONS
Patients with a first episode of pericarditis should be treated with an anti-inflammatory medication, with no comprehensive testing for autoimmune disease. An evaluation for autoimmune and infectious disease should be carried out in patients with fever (temperature > 38°C; 100.4°F), recurrent pericarditis, recurrent large pericardial effusion or tamponade, or night sweats despite conventional medical therapy. Signs of systemic disease such as renal failure, elevated liver enzymes, or skin rash merit further evaluation.
Prospective studies using appropriate serologic testing and imaging are needed to determine the correlation between myopericardial involvement and inflammatory diseases because of increased morbidity and mortality in several of these diseases.
Pericarditis is in most cases a one-time disease simply treated with anti-inflammatory drugs. It requires no extensive workup for systemic inflammatory or autoimmune disease. Further evaluation is required for patients who have recurrent pericarditis resistant to conventional therapy or pericarditis with manifestations of systemic disease.
ACUTE PERICARDITIS
Pericardial disease has different presentations: acute, recurrent, constrictive, effusive-constrictive, and pericardial effusion with or without tamponade. Acute pericarditis is the most common of these and can affect people of all ages. The typical acute manifestations are chest pain (usually pleuritic), a pericardial friction rub, and widespread ST-segment elevation on the electrocardiogram.1,2 The chest pain tends to be sharp and long-lasting; it radiates to the trapezius ridge and increases during respiration or body movements.
Acute pericarditis usually responds to an anti-inflammatory drug such as colchicine 0.6 mg/day for 3 months, a nonsteroidal anti-inflammatory drug such as ibuprofen 600 mg three times a day for 10 days, and in advanced resistant cases, an oral corticosteroid.3,4
Most often, pericarditis is either idiopathic or occurs after a respiratory viral illness. Much less common causes include bacterial infection, postpericardiotomy syndrome, myocardial infarction, primary or metastatic tumors, trauma, radiation, and uremia. However, pericarditis can also be part of the presentation of systemic inflammatory and autoimmune diseases such as rheumatoid arthritis and systemic lupus erythematosus; hereditary periodic fever syndromes such as familial Mediterranean fever; and systemic-onset juvenile idiopathic arthritis.1,5
In acute pericarditis, a complex workup is usually not justified, since the results will have limited usefulness in the clinical management of the patient. It is most often diagnosed by the presenting symptoms, auscultation, electrocardiography, echocardiography, and chest radiography, and by additional basic tests that include a complete blood cell count, complete metabolic profile, erythrocyte sedimentation rate, and C-reactive protein level. However, if pericarditis does not respond to anti-inflammatory treatment and if an autoimmune or infectious disease is suspected, further evaluation may include antinuclear antibody testing and testing for human immunodeficiency virus and tuberculosis. If the diagnosis of acute pericarditis remains uncertain, cardiac magnetic resonance imaging (MRI) may be useful.
RECURRENT PERICARDITIS
Although acute pericarditis most often has a benign course and responds well to anti-inflammatory drugs, 20% to 30% of patients who have a first attack of acute pericarditis have a recurrence, and up to 50% of patients who have one recurrence will have another.3,4
Disease activity can be followed with serial testing of inflammatory markers—eg, erythrocyte sedimentation rate and C-reactive protein level. Echocardiography, cardiac computed tomography, and cardiac MRI can characterize active inflammation, edema, pericardial thickness, and pericardial effusion.6–8
Recurrent pericarditis is often resistant to standard therapy and requires corticosteroids in high doses, which paradoxically can increase the risk of recurrence. Therefore, further workup for underlying autoimmune disease, systemic inflammatory disease, or infection is necessary. More potent immunosuppressive therapy may be required, not only in pericarditis associated with systemic autoimmune or inflammatory conditions, but even in idiopathic recurrent pericarditis, either to control symptoms or to mitigate the effects of corticosteroids.
SYSTEMIC INFLAMMATION
The true prevalence of pericardial disease in most systemic inflammatory and autoimmune diseases is difficult to determine from current data. But advances in serologic testing and imaging techniques have shown cardiac involvement in a number of inflammatory diseases.9
In one study, a serologic autoimmune workup in patients with acute pericarditis found that 2% had collagen vascular disease.9 Pericardial involvement is likely in systemic lupus erythematosus,10 and a postmortem study of patients with systemic sclerosis found that 72% had pericarditis.11 Mixed connective tissue disease has been associated with pericarditis in 29% of cases and 56% in autopsy studies.12,13 Pericarditis may be the initial manifestation of vasculitis—eg, Takayasu arteritis or granulomatosis with polyangiitis (formerly known as Wegener granulomatosis).
Other diseases with pericardial involvement include Still disease, Sjögren syndrome, sarcoidosis, and inflammatory bowel disease. Symptomatic pericarditis occurs in about 25% of patients with Sjögren syndrome and asymptomatic pericardial involvement in more than half. Autopsy studies reported pericardial involvement in up to 80% of patients with systemic lupus erythematosus. Cardiac tamponade occurs in fewer than 2%, and constrictive pericarditis is extremely rare.5,9–11
RECOMMENDATIONS
Patients with a first episode of pericarditis should be treated with an anti-inflammatory medication, with no comprehensive testing for autoimmune disease. An evaluation for autoimmune and infectious disease should be carried out in patients with fever (temperature > 38°C; 100.4°F), recurrent pericarditis, recurrent large pericardial effusion or tamponade, or night sweats despite conventional medical therapy. Signs of systemic disease such as renal failure, elevated liver enzymes, or skin rash merit further evaluation.
Prospective studies using appropriate serologic testing and imaging are needed to determine the correlation between myopericardial involvement and inflammatory diseases because of increased morbidity and mortality in several of these diseases.
- Troughton RW, Asher CR, Klein AL. Pericarditis. Lancet 2004; 363:717–727.
- Alraies MC, Klein AL. Should we still use electrocardiography to diagnose pericardial disease? Cleve Clin J Med 2013; 80:97–100.
- Imazio M, Brucato A, Cemin R, et al; ICAP Investigators. A randomized trial of colchicine for acute pericarditis. N Engl J Med 2013; 369:1522–1528.
- Imazio M, Cecchi E, Demichelis B, et al. Indicators of poor prognosis of acute pericarditis. Circulation 2007; 115:2739–2744.
- Zayas R, Anguita M, Torres F, et al. Incidence of specific etiology and role of methods for specific etiologic diagnosis of primary acute pericarditis. Am J Cardiol 1995; 75:378–382.
- Verhaert D, Gabriel RS, Johnston D, Lytle BW, Desai MY, Klein AL. The role of multimodality imaging in the management of pericardial disease. Circ Cardiovasc Imaging 2010; 3:333–343.
- Klein AL, Abbara S, Agler DA, et al. American Society of Echocardiography clinical recommendations for multimodality cardiovascular imaging of patients with pericardial disease: endorsed by the Society for Cardiovascular Magnetic Resonance and Society of Cardiovascular Computed Tomography. J Am Soc Echocardiogr 2013; 26:965–1012.e15.
- Yingchoncharoen T, Alraies MC, Kwon DH, Rodriguez ER, Tan CD, Klein AL. Emerging role of multimodality imaging in management of inflammatory pericardial diseases. Expert Rev Cardiovasc Ther 2013; 11:1211–1225.
- Knockaert DC. Cardiac involvement in systemic inflammatory diseases. Eur Heart J 2007; 28:1797–1804.
- Doria A, Iaccarino L, Sarzi-Puttini P, Atzeni F, Turriel M, Petri M. Cardiac involvement in systemic lupus erythematosus. Lupus 2005; 14:683–686.
- Byers RJ, Marshall DA, Freemont AJ. Pericardial involvement in systemic sclerosis. Ann Rheum Dis 1997; 56:393–394.
- Kasukawa R. Mixed connective tissue disease. Intern Med 1999; 38:386–393.
- Bezerra MC, Saraiva F Jr, Carvalho JF, Caleiro MT, Goncalves CR, Borba EF. Cardiac tamponade due to massive pericardial effusion in mixed connective tissue disease: reversal with steroid therapy. Lupus 2004; 13:618–620.
- Troughton RW, Asher CR, Klein AL. Pericarditis. Lancet 2004; 363:717–727.
- Alraies MC, Klein AL. Should we still use electrocardiography to diagnose pericardial disease? Cleve Clin J Med 2013; 80:97–100.
- Imazio M, Brucato A, Cemin R, et al; ICAP Investigators. A randomized trial of colchicine for acute pericarditis. N Engl J Med 2013; 369:1522–1528.
- Imazio M, Cecchi E, Demichelis B, et al. Indicators of poor prognosis of acute pericarditis. Circulation 2007; 115:2739–2744.
- Zayas R, Anguita M, Torres F, et al. Incidence of specific etiology and role of methods for specific etiologic diagnosis of primary acute pericarditis. Am J Cardiol 1995; 75:378–382.
- Verhaert D, Gabriel RS, Johnston D, Lytle BW, Desai MY, Klein AL. The role of multimodality imaging in the management of pericardial disease. Circ Cardiovasc Imaging 2010; 3:333–343.
- Klein AL, Abbara S, Agler DA, et al. American Society of Echocardiography clinical recommendations for multimodality cardiovascular imaging of patients with pericardial disease: endorsed by the Society for Cardiovascular Magnetic Resonance and Society of Cardiovascular Computed Tomography. J Am Soc Echocardiogr 2013; 26:965–1012.e15.
- Yingchoncharoen T, Alraies MC, Kwon DH, Rodriguez ER, Tan CD, Klein AL. Emerging role of multimodality imaging in management of inflammatory pericardial diseases. Expert Rev Cardiovasc Ther 2013; 11:1211–1225.
- Knockaert DC. Cardiac involvement in systemic inflammatory diseases. Eur Heart J 2007; 28:1797–1804.
- Doria A, Iaccarino L, Sarzi-Puttini P, Atzeni F, Turriel M, Petri M. Cardiac involvement in systemic lupus erythematosus. Lupus 2005; 14:683–686.
- Byers RJ, Marshall DA, Freemont AJ. Pericardial involvement in systemic sclerosis. Ann Rheum Dis 1997; 56:393–394.
- Kasukawa R. Mixed connective tissue disease. Intern Med 1999; 38:386–393.
- Bezerra MC, Saraiva F Jr, Carvalho JF, Caleiro MT, Goncalves CR, Borba EF. Cardiac tamponade due to massive pericardial effusion in mixed connective tissue disease: reversal with steroid therapy. Lupus 2004; 13:618–620.
Penicillin allergy: A practical guide for clinicians
Most patients who report that they are allergic to penicillin can ultimately receive penicillin or a penicillin-type antibiotic again after an appropriate evaluation and, possibly, treatment. This course of action decreases the need for broad-spectrum antibiotics,1–4 reduces health care costs, and prevents the development of multidrug-resistant pathogens.5
About 10% of the general population say that they are allergic to penicillin.1,6,7 Although the prevalence of life-threatening anaphylactic reactions to penicillin has been estimated to be between 0.02% and 0.04%,6 the most common reaction is a cutaneous eruption. Since anaphylactic reactions are mediated by immunoglobulin E (IgE), evaluation of patients with a history of penicillin allergy by penicillin skin testing is recommended to rule out IgE-mediated reactions.
This review outlines a practical approach to evaluating a suspected IgE-mediated reaction to penicillin, with key points in the history and diagnostic testing. We also review subsequent management and cross-reactivity with other beta-lactam-containing antibiotics.
EVALUATING ALLERGIC PATIENTS
Evaluation of patients with a history of penicillin allergy can be improved with an understanding of the classification of drug reactions, risk factors for allergy, and the pathophysiology of penicillin allergy.
Classification of drug reactions
Adverse drug reactions include all unintended pharmacologic effects of a drug and can be classified as predictable (type A) or unpredictable (type B). Predictable reactions are dose-dependent, are related to the known pharmacologic actions of the medication, and occur in otherwise healthy individuals. Unpredictable reactions are further classified into drug intolerance, drug idiosyncrasy, drug allergy, and pseudoallergic reactions.8,9
Penicillin allergy can manifest as any hypersensitivity reaction of the Gell and Coombs classification (Table 1).9 Type I (immediate) and type IV (delayed) reactions are the most common types of reactions that occur with antibiotics and should be classified based on the onset of symptoms as immediate (within 1 hour) or delayed (days or weeks).8
Risk factors for IgE-mediated reaction
Risk factors for a hypersensitivity reaction include frequent or repetitive courses of penicillin10 and high-dose parenteral (rather than oral) administration.
Age and atopy are not risk factors for penicillin allergy.7 However, atopy increases the risk of a more severe anaphylactic reaction to penicillin, and anaphylactic reactions are most commonly reported between the ages of 20 and 49.6
Pathophysiology of penicillin allergy
All penicillins share a common core ring structure (beta-lactam and thiazolidine rings) but differ in their side chains (R group) (Figure 1).
Under physiologic conditions, the core ring structure is metabolized into major (penicilloyl) and minor (penicillin itself, penicilloate and penilloate) antigenic determinants that may trigger an immediate IgE-dependent response.9 In the United States, commercial forms of antigenic determinates for skin testing exist in the form of penicillin G (minor determinant) and penicilloyl-polylysine, better known as Prepen (major determinant).
Immediate-type reactions to similar antibiotics such as aminopenicillins and cephalosporins may be caused by IgE antibodies against the R-group side chain rather than the core penicillin major and minor determinants.11
Questions to ask patients who have a history of penicillin allergy
Patients should be questioned closely about previous and current reactions to penicillin and should undergo skin-prick and intradermal testing, followed by graded-dose challenge or drug tolerance desensitization (Figure 2).
Questions to ask patients who have a history of penicillin allergy (Table 2)9,12 include the following:
Do you remember the details of the reaction? These include the route of administration, the time between the dose of penicillin and the appearance of symptoms, and how the reaction was managed.
Immediate reactions (ie, IgE-mediated, or Gell and Coombs type I) usually occur within the first hour after the first dose of the antibiotic, although they occasionally take up to 2 hours to occur, especially if the medication is taken orally and is taken with food. Symptoms consistent with IgE-mediated reactions include urticaria (most common), pruritus, angioedema, laryngeal edema, wheezing, shortness of breath, presyncope or syncope, hypotension, and cardiorespiratory collapse.
In contrast, symptoms of a non–IgE-mediated reaction are delayed in onset, occurring after days of treatment. They include nonpruritic maculopapular eruptions, hemolytic anemia, serum sickness, Stevens-Johnson syndrome, drug rash with eosinophilia and systemic symptoms, acute interstitial nephritis, and toxic epidermal necrolysis.9
If the patient has had severe non–IgE-mediated reactions to penicillin (eg, Stevens-Johnson syndrome, toxic epidermal necrolysis, acute interstitial nephritis, hemolytic anemia, or serum sickness) in the past, skin testing, graded-dose challenge, and desensitization are contraindicated.
How many years ago did the reaction occur? Most patients lose their sensitivity to penicillin over time.7,13–15 Nearly 50% of patients with IgE-mediated penicillin allergy lose their sensitivity within 5 years of the reaction,15 increasing to 80% or more by 10 years.13
How was the reaction managed? What was the outcome? Use of and positive response to epinephrine and histamine 1 receptor antagonists (antihistamines) with resolution or significant improvement of symptoms within a few hours may indicate an IgE-mediated reaction.
What was the indication for penicillin? Many cutaneous reactions are a result of an underlying viral or bacterial infection. For example, up to 90% of patients with Epstein-Barr virus infection develop a maculopapular rash when given penicillin.16
Have you tolerated other forms of penicillin since the reaction? Sometimes the patient has already tolerated other beta-lactams such as aminopenicillins, cephalosporins, and semisynthetic penicillins (piperacillin-tazobactam). Patients who tolerate other beta-lactams without adverse reactions are not allergic to beta-lactams.
Diagnostic tests
Skin testing. The only validated test for diagnosing IgE-mediated reactions caused by penicillin is the immediate hypersensitivity skin test,9 which should be performed by a board-certified allergist. The test consists of skin-prick and intradermal testing with the major determinant (penicilloyl-polylysine), the minor determinant (penicillin G), a negative control (normal saline), and a positive control (histamine). Minor-determinant mix is not commercially available in the United States.
Results of skin-prick testing are read 15 minutes after application. A positive response is a wheal at least 3 mm larger in diameter (with equivalent erythema) than the negative control done simultaneously. Intradermal testing is only done after a negative skin-prick test. If the allergic reaction was severe (ie, anaphylaxis), skin testing should be done at least 4 to 6 weeks after the reaction.
A history of severe non–IgE-mediated reaction to penicillin is a contraindication to skin-prick testing for penicillin allergy. The positive predictive value of penicillin skin testing is 50%, and the negative predictive value is 97%.3,7,9,13
Commercial in vitro testing (serum-specific IgE assays) for IgE-mediated hypersensitivity to penicillin is inferior to skin testing in terms of the negative predictive value and is not a suitable substitute for penicillin skin testing.
MANAGING PENICILLIN ALLERGY
If skin testing is positive, use another antibiotic, or refer for desensitization
If penicillin skin testing is positive (Figure 2), use another antibiotic that is equally efficacious. Patients who absolutely need a beta-lactam may undergo drug desensitization, performed by a board-certified allergist.
During desensitization, patients receive progressively higher doses of the drug every 15 to 20 minutes subcutaneously or intravenously, or every 20 to 30 minutes orally, until a full therapeutic dose is tolerated. Most protocols begin with a dose ranging from 1/10,000 to 1/1,000 of the final dose, depending on the severity of the allergic reaction.9,17
Using modern protocols, the success rate for tolerance induction is extremely high (75% to 100% in patients with cystic fibrosis, a group with a high rate of drug allergy18–20).
Drug desensitization is contraindicated in patients with non–IgE-mediated reactions.
If skin testing is negative, refer for graded-dose challenge
If skin testing is negative (Figure 2), graded-dose challenge is recommended. This procedure must be done by a board-certified allergist. If the original reaction was life-threatening, graded-dose challenge may entail giving 1/100 of the therapeutic dose. Then, if no reaction occurs during a brief observation period (usually 30 minutes), a full dose is given. However, many patients can start with 1/10 or even a full dose of the drug, especially if the original reaction was limited to the skin and the penicillin skin test is negative.
Graded-dose challenge is contraindicated if the original reaction was a severe non–IgE-mediated reaction.
UNDERSTANDING CROSS-REACTIVITY OF PENICILLIN
Penicillin is the only antibiotic for which skin testing is reliable and validated. If a drug that cross-reacts with penicillin is needed, it is important to know the rate of cross-reactivity (Table 3). The rate of cross-reactivity between penicillin and aminopenicillins (amoxicillin and ampicillin) is less than 1.3% in the United States.10,21 However, the cross-reactivity rate among aminopenicillins and cephalosporins is between 10% to 40%. For that reason, patients with prior reactions to aminopenicillins should avoid cephalosporins that share identical R-chain side groups with aminopenicillins.9,22
The rate of cross-reactivity between penicillin and cephalosporins was reported as 10% 40 years ago.23,24 But this was with early, first-generation cephalosporins that may have been contaminated with penicillin. The cross-reactivity rate with cephalosporins today is 3%.25 In general, first- and second-generation cephalosporins cause more allergic reactions than third- and fourth-generation cephalosporins.26
Patients with a history of penicillin allergy who require a cephalosporin should still undergo penicillin skin testing. Skin testing with cephalosporins has not been validated. However, skin testing with nonirritating concentrations of cephalosporins9 may be done to elucidate IgE reactions.
In a study by Romano et al,27 110 patients who had positive results on penicillin skin testing completed graded-dose challenge with the carbapenem antibiotic imipenem. The rate of cross-reactivity between penicillin and imipenem was less than 1%.
Monobactam antibiotics do not cross-react with other beta-lactams, except ceftazidime with aztreonam. This is probably because of similarities in their chemical structure.
- Park M, Markus P, Matesic D, Li JT. Safety and effectiveness of a preoperative allergy clinic in decreasing vancomycin use in patients with a history of penicillin allergy. Ann Allergy Asthma Immunol 2006; 97:681–687.
- Arroliga ME, Radojicic C, Gordon SM, et al. A prospective observational study of the effect of penicillin skin testing on antibiotic use in the intensive care unit. Infect Control Hosp Epidemiol 2003; 24:347–350.
- del Real GA, Rose ME, Ramirez-Atamoros MT, et al. Penicillin skin testing in patients with a history of beta-lactam allergy. Ann Allergy Asthma Immunol 2007; 98:355–359.
- Nadarajah K, Green GR, Naglak M. Clinical outcomes of penicillin skin testing. Ann Allergy Asthma Immunol 2005; 95:541–545.
- Harris AD, Sauberman L, Kabbash L, Greineder DK, Samore MH. Penicillin skin testing: a way to optimize antibiotic utilization. Am J Med 1999; 107:166–168.
- Idsoe O, Guthe T, Willcox RR, de Weck AL. Nature and extent of penicillin side-reactions, with particular reference to fatalities from anaphylactic shock. Bull World Health Organ 1968; 38:159–188.
- Gadde J, Spence M, Wheeler B, Adkinson NF Jr. Clinical experience with penicillin skin testing in a large inner-city STD clinic. JAMA 1993; 270:2456–2463.
- Johansson SG, Bieber T, Dahl R, et al. Revised nomenclature for allergy for global use: report of the Nomenclature Review Committee of the World Allergy Organization, October 2003. J Allergy Clin Immunol 2004; 113:832–836.
- Joint Task Force on Practice Parameters; American Academy of Allergy, Asthma and Immunology; American College of Allergy, Asthma and Immunology; Joint Council of Allergy, Asthma and Immunology. Drug allergy: an updated practice parameter. Ann Allergy Asthma Immunol 2010; 105:259–273.
- Park MA, Matesic D, Markus PJ, Li JT. Female sex as a risk factor for penicillin allergy. Ann Allergy Asthma Immunol 2007; 99:54–58.
- Moreno E, Macias E, Davila I, Laffond E, Ruiz A, Lorente F. Hypersensitivity reactions to cephalosporins. Expert Opin Drug Saf 2008; 7:295–304.
- Salkind AR, Cuddy PG, Foxworth JW. The rational clinical examination. Is this patient allergic to penicillin? An evidence-based analysis of the likelihood of penicillin allergy. JAMA 2001; 285:2498–2505.
- Sullivan TJ, Wedner HJ, Shatz GS, Yecies LD, Parker CW. Skin testing to detect penicillin allergy. J Allergy Clin Immunol 1981; 68:171–180.
- Macy E, Schatz M, Lin C, Poon KY. The falling rate of positive penicillin skin tests from 1995 to 2007. Perm J 2009; 13:12–18.
- Blanca M, Torres MJ, Garcia JJ, et al. Natural evolution of skin test sensitivity in patients allergic to beta-lactam antibiotics. J Allergy Clin Immunol 1999; 103:918–924.
- Patel BM. Skin rash with infectious mononucleosis and ampicillin. Pediatrics 1967; 40:910–911.
- Liu A, Fanning L, Chong H, et al. Desensitization regimens for drug allergy: state of the art in the 21st century. Clin Exp Allergy 2011; 41:1679–1688.
- Burrows JA, Toon M, Bell SC. Antibiotic desensitization in adults with cystic fibrosis. Respirology 2003; 8:359–364.
- Turvey SE, Cronin B, Arnold AD, Dioun AF. Antibiotic desensitization for the allergic patient: 5 years of experience and practice. Ann Allergy Asthma Immunol 2004; 92:426–432.
- Legere HJ 3rd, Palis RI, Rodriguez Bouza T, Uluer AZ, Castells MC. A safe protocol for rapid desensitization in patients with cystic fibrosis and antibiotic hypersensitivity. J Cyst Fibros 2009; 8:418–424.
- Lin E, Saxon A, Riedl M. Penicillin allergy: value of including amoxicillin as a determinant in penicillin skin testing. Int Arch Allergy Immunol 2010; 152:313–318.
- Dickson SD, Salazar KC. Diagnosis and management of immediate hypersensitivity reactions to cephalosporins. Clin Rev Allergy Immunol 2013; 45:131–142.
- Dash CH. Penicillin allergy and the cephalosporins. J Antimicrob Chemother 1975; 1(suppl 3):107–118.
- Petz LD. Immunologic cross-reactivity between penicillins and cephalosporins: a review. J Infect Dis 1978; 137(suppl):S74–S79.
- American Academy of Allergy Asthma & Immunology. Cephalosporin administration to patients with a history of penicillin allergy. www.aaaai.org/Aaaai/media/MediaLibrary/PDF%20Documents/Practice%20and%20Parameters/Cephalosporin-administration-2009.pdf. Accessed April 2, 2015.
- Fonacier L, Hirschberg R, Gerson S. Adverse drug reactions to cephalosporins in hospitalized patients with a history of penicillin allergy. Allergy Asthma Proc 2005; 26:135–141.
- Romano A, Viola M, Gueant-Rodriguez RM, Gaeta F, Pettinato R, Gueant JL. Imipenem in patients with immediate hypersensitivity to penicillins. N Engl J Med 2006; 354:2835–2837.
Most patients who report that they are allergic to penicillin can ultimately receive penicillin or a penicillin-type antibiotic again after an appropriate evaluation and, possibly, treatment. This course of action decreases the need for broad-spectrum antibiotics,1–4 reduces health care costs, and prevents the development of multidrug-resistant pathogens.5
About 10% of the general population say that they are allergic to penicillin.1,6,7 Although the prevalence of life-threatening anaphylactic reactions to penicillin has been estimated to be between 0.02% and 0.04%,6 the most common reaction is a cutaneous eruption. Since anaphylactic reactions are mediated by immunoglobulin E (IgE), evaluation of patients with a history of penicillin allergy by penicillin skin testing is recommended to rule out IgE-mediated reactions.
This review outlines a practical approach to evaluating a suspected IgE-mediated reaction to penicillin, with key points in the history and diagnostic testing. We also review subsequent management and cross-reactivity with other beta-lactam-containing antibiotics.
EVALUATING ALLERGIC PATIENTS
Evaluation of patients with a history of penicillin allergy can be improved with an understanding of the classification of drug reactions, risk factors for allergy, and the pathophysiology of penicillin allergy.
Classification of drug reactions
Adverse drug reactions include all unintended pharmacologic effects of a drug and can be classified as predictable (type A) or unpredictable (type B). Predictable reactions are dose-dependent, are related to the known pharmacologic actions of the medication, and occur in otherwise healthy individuals. Unpredictable reactions are further classified into drug intolerance, drug idiosyncrasy, drug allergy, and pseudoallergic reactions.8,9
Penicillin allergy can manifest as any hypersensitivity reaction of the Gell and Coombs classification (Table 1).9 Type I (immediate) and type IV (delayed) reactions are the most common types of reactions that occur with antibiotics and should be classified based on the onset of symptoms as immediate (within 1 hour) or delayed (days or weeks).8
Risk factors for IgE-mediated reaction
Risk factors for a hypersensitivity reaction include frequent or repetitive courses of penicillin10 and high-dose parenteral (rather than oral) administration.
Age and atopy are not risk factors for penicillin allergy.7 However, atopy increases the risk of a more severe anaphylactic reaction to penicillin, and anaphylactic reactions are most commonly reported between the ages of 20 and 49.6
Pathophysiology of penicillin allergy
All penicillins share a common core ring structure (beta-lactam and thiazolidine rings) but differ in their side chains (R group) (Figure 1).
Under physiologic conditions, the core ring structure is metabolized into major (penicilloyl) and minor (penicillin itself, penicilloate and penilloate) antigenic determinants that may trigger an immediate IgE-dependent response.9 In the United States, commercial forms of antigenic determinates for skin testing exist in the form of penicillin G (minor determinant) and penicilloyl-polylysine, better known as Prepen (major determinant).
Immediate-type reactions to similar antibiotics such as aminopenicillins and cephalosporins may be caused by IgE antibodies against the R-group side chain rather than the core penicillin major and minor determinants.11
Questions to ask patients who have a history of penicillin allergy
Patients should be questioned closely about previous and current reactions to penicillin and should undergo skin-prick and intradermal testing, followed by graded-dose challenge or drug tolerance desensitization (Figure 2).
Questions to ask patients who have a history of penicillin allergy (Table 2)9,12 include the following:
Do you remember the details of the reaction? These include the route of administration, the time between the dose of penicillin and the appearance of symptoms, and how the reaction was managed.
Immediate reactions (ie, IgE-mediated, or Gell and Coombs type I) usually occur within the first hour after the first dose of the antibiotic, although they occasionally take up to 2 hours to occur, especially if the medication is taken orally and is taken with food. Symptoms consistent with IgE-mediated reactions include urticaria (most common), pruritus, angioedema, laryngeal edema, wheezing, shortness of breath, presyncope or syncope, hypotension, and cardiorespiratory collapse.
In contrast, symptoms of a non–IgE-mediated reaction are delayed in onset, occurring after days of treatment. They include nonpruritic maculopapular eruptions, hemolytic anemia, serum sickness, Stevens-Johnson syndrome, drug rash with eosinophilia and systemic symptoms, acute interstitial nephritis, and toxic epidermal necrolysis.9
If the patient has had severe non–IgE-mediated reactions to penicillin (eg, Stevens-Johnson syndrome, toxic epidermal necrolysis, acute interstitial nephritis, hemolytic anemia, or serum sickness) in the past, skin testing, graded-dose challenge, and desensitization are contraindicated.
How many years ago did the reaction occur? Most patients lose their sensitivity to penicillin over time.7,13–15 Nearly 50% of patients with IgE-mediated penicillin allergy lose their sensitivity within 5 years of the reaction,15 increasing to 80% or more by 10 years.13
How was the reaction managed? What was the outcome? Use of and positive response to epinephrine and histamine 1 receptor antagonists (antihistamines) with resolution or significant improvement of symptoms within a few hours may indicate an IgE-mediated reaction.
What was the indication for penicillin? Many cutaneous reactions are a result of an underlying viral or bacterial infection. For example, up to 90% of patients with Epstein-Barr virus infection develop a maculopapular rash when given penicillin.16
Have you tolerated other forms of penicillin since the reaction? Sometimes the patient has already tolerated other beta-lactams such as aminopenicillins, cephalosporins, and semisynthetic penicillins (piperacillin-tazobactam). Patients who tolerate other beta-lactams without adverse reactions are not allergic to beta-lactams.
Diagnostic tests
Skin testing. The only validated test for diagnosing IgE-mediated reactions caused by penicillin is the immediate hypersensitivity skin test,9 which should be performed by a board-certified allergist. The test consists of skin-prick and intradermal testing with the major determinant (penicilloyl-polylysine), the minor determinant (penicillin G), a negative control (normal saline), and a positive control (histamine). Minor-determinant mix is not commercially available in the United States.
Results of skin-prick testing are read 15 minutes after application. A positive response is a wheal at least 3 mm larger in diameter (with equivalent erythema) than the negative control done simultaneously. Intradermal testing is only done after a negative skin-prick test. If the allergic reaction was severe (ie, anaphylaxis), skin testing should be done at least 4 to 6 weeks after the reaction.
A history of severe non–IgE-mediated reaction to penicillin is a contraindication to skin-prick testing for penicillin allergy. The positive predictive value of penicillin skin testing is 50%, and the negative predictive value is 97%.3,7,9,13
Commercial in vitro testing (serum-specific IgE assays) for IgE-mediated hypersensitivity to penicillin is inferior to skin testing in terms of the negative predictive value and is not a suitable substitute for penicillin skin testing.
MANAGING PENICILLIN ALLERGY
If skin testing is positive, use another antibiotic, or refer for desensitization
If penicillin skin testing is positive (Figure 2), use another antibiotic that is equally efficacious. Patients who absolutely need a beta-lactam may undergo drug desensitization, performed by a board-certified allergist.
During desensitization, patients receive progressively higher doses of the drug every 15 to 20 minutes subcutaneously or intravenously, or every 20 to 30 minutes orally, until a full therapeutic dose is tolerated. Most protocols begin with a dose ranging from 1/10,000 to 1/1,000 of the final dose, depending on the severity of the allergic reaction.9,17
Using modern protocols, the success rate for tolerance induction is extremely high (75% to 100% in patients with cystic fibrosis, a group with a high rate of drug allergy18–20).
Drug desensitization is contraindicated in patients with non–IgE-mediated reactions.
If skin testing is negative, refer for graded-dose challenge
If skin testing is negative (Figure 2), graded-dose challenge is recommended. This procedure must be done by a board-certified allergist. If the original reaction was life-threatening, graded-dose challenge may entail giving 1/100 of the therapeutic dose. Then, if no reaction occurs during a brief observation period (usually 30 minutes), a full dose is given. However, many patients can start with 1/10 or even a full dose of the drug, especially if the original reaction was limited to the skin and the penicillin skin test is negative.
Graded-dose challenge is contraindicated if the original reaction was a severe non–IgE-mediated reaction.
UNDERSTANDING CROSS-REACTIVITY OF PENICILLIN
Penicillin is the only antibiotic for which skin testing is reliable and validated. If a drug that cross-reacts with penicillin is needed, it is important to know the rate of cross-reactivity (Table 3). The rate of cross-reactivity between penicillin and aminopenicillins (amoxicillin and ampicillin) is less than 1.3% in the United States.10,21 However, the cross-reactivity rate among aminopenicillins and cephalosporins is between 10% to 40%. For that reason, patients with prior reactions to aminopenicillins should avoid cephalosporins that share identical R-chain side groups with aminopenicillins.9,22
The rate of cross-reactivity between penicillin and cephalosporins was reported as 10% 40 years ago.23,24 But this was with early, first-generation cephalosporins that may have been contaminated with penicillin. The cross-reactivity rate with cephalosporins today is 3%.25 In general, first- and second-generation cephalosporins cause more allergic reactions than third- and fourth-generation cephalosporins.26
Patients with a history of penicillin allergy who require a cephalosporin should still undergo penicillin skin testing. Skin testing with cephalosporins has not been validated. However, skin testing with nonirritating concentrations of cephalosporins9 may be done to elucidate IgE reactions.
In a study by Romano et al,27 110 patients who had positive results on penicillin skin testing completed graded-dose challenge with the carbapenem antibiotic imipenem. The rate of cross-reactivity between penicillin and imipenem was less than 1%.
Monobactam antibiotics do not cross-react with other beta-lactams, except ceftazidime with aztreonam. This is probably because of similarities in their chemical structure.
Most patients who report that they are allergic to penicillin can ultimately receive penicillin or a penicillin-type antibiotic again after an appropriate evaluation and, possibly, treatment. This course of action decreases the need for broad-spectrum antibiotics,1–4 reduces health care costs, and prevents the development of multidrug-resistant pathogens.5
About 10% of the general population say that they are allergic to penicillin.1,6,7 Although the prevalence of life-threatening anaphylactic reactions to penicillin has been estimated to be between 0.02% and 0.04%,6 the most common reaction is a cutaneous eruption. Since anaphylactic reactions are mediated by immunoglobulin E (IgE), evaluation of patients with a history of penicillin allergy by penicillin skin testing is recommended to rule out IgE-mediated reactions.
This review outlines a practical approach to evaluating a suspected IgE-mediated reaction to penicillin, with key points in the history and diagnostic testing. We also review subsequent management and cross-reactivity with other beta-lactam-containing antibiotics.
EVALUATING ALLERGIC PATIENTS
Evaluation of patients with a history of penicillin allergy can be improved with an understanding of the classification of drug reactions, risk factors for allergy, and the pathophysiology of penicillin allergy.
Classification of drug reactions
Adverse drug reactions include all unintended pharmacologic effects of a drug and can be classified as predictable (type A) or unpredictable (type B). Predictable reactions are dose-dependent, are related to the known pharmacologic actions of the medication, and occur in otherwise healthy individuals. Unpredictable reactions are further classified into drug intolerance, drug idiosyncrasy, drug allergy, and pseudoallergic reactions.8,9
Penicillin allergy can manifest as any hypersensitivity reaction of the Gell and Coombs classification (Table 1).9 Type I (immediate) and type IV (delayed) reactions are the most common types of reactions that occur with antibiotics and should be classified based on the onset of symptoms as immediate (within 1 hour) or delayed (days or weeks).8
Risk factors for IgE-mediated reaction
Risk factors for a hypersensitivity reaction include frequent or repetitive courses of penicillin10 and high-dose parenteral (rather than oral) administration.
Age and atopy are not risk factors for penicillin allergy.7 However, atopy increases the risk of a more severe anaphylactic reaction to penicillin, and anaphylactic reactions are most commonly reported between the ages of 20 and 49.6
Pathophysiology of penicillin allergy
All penicillins share a common core ring structure (beta-lactam and thiazolidine rings) but differ in their side chains (R group) (Figure 1).
Under physiologic conditions, the core ring structure is metabolized into major (penicilloyl) and minor (penicillin itself, penicilloate and penilloate) antigenic determinants that may trigger an immediate IgE-dependent response.9 In the United States, commercial forms of antigenic determinates for skin testing exist in the form of penicillin G (minor determinant) and penicilloyl-polylysine, better known as Prepen (major determinant).
Immediate-type reactions to similar antibiotics such as aminopenicillins and cephalosporins may be caused by IgE antibodies against the R-group side chain rather than the core penicillin major and minor determinants.11
Questions to ask patients who have a history of penicillin allergy
Patients should be questioned closely about previous and current reactions to penicillin and should undergo skin-prick and intradermal testing, followed by graded-dose challenge or drug tolerance desensitization (Figure 2).
Questions to ask patients who have a history of penicillin allergy (Table 2)9,12 include the following:
Do you remember the details of the reaction? These include the route of administration, the time between the dose of penicillin and the appearance of symptoms, and how the reaction was managed.
Immediate reactions (ie, IgE-mediated, or Gell and Coombs type I) usually occur within the first hour after the first dose of the antibiotic, although they occasionally take up to 2 hours to occur, especially if the medication is taken orally and is taken with food. Symptoms consistent with IgE-mediated reactions include urticaria (most common), pruritus, angioedema, laryngeal edema, wheezing, shortness of breath, presyncope or syncope, hypotension, and cardiorespiratory collapse.
In contrast, symptoms of a non–IgE-mediated reaction are delayed in onset, occurring after days of treatment. They include nonpruritic maculopapular eruptions, hemolytic anemia, serum sickness, Stevens-Johnson syndrome, drug rash with eosinophilia and systemic symptoms, acute interstitial nephritis, and toxic epidermal necrolysis.9
If the patient has had severe non–IgE-mediated reactions to penicillin (eg, Stevens-Johnson syndrome, toxic epidermal necrolysis, acute interstitial nephritis, hemolytic anemia, or serum sickness) in the past, skin testing, graded-dose challenge, and desensitization are contraindicated.
How many years ago did the reaction occur? Most patients lose their sensitivity to penicillin over time.7,13–15 Nearly 50% of patients with IgE-mediated penicillin allergy lose their sensitivity within 5 years of the reaction,15 increasing to 80% or more by 10 years.13
How was the reaction managed? What was the outcome? Use of and positive response to epinephrine and histamine 1 receptor antagonists (antihistamines) with resolution or significant improvement of symptoms within a few hours may indicate an IgE-mediated reaction.
What was the indication for penicillin? Many cutaneous reactions are a result of an underlying viral or bacterial infection. For example, up to 90% of patients with Epstein-Barr virus infection develop a maculopapular rash when given penicillin.16
Have you tolerated other forms of penicillin since the reaction? Sometimes the patient has already tolerated other beta-lactams such as aminopenicillins, cephalosporins, and semisynthetic penicillins (piperacillin-tazobactam). Patients who tolerate other beta-lactams without adverse reactions are not allergic to beta-lactams.
Diagnostic tests
Skin testing. The only validated test for diagnosing IgE-mediated reactions caused by penicillin is the immediate hypersensitivity skin test,9 which should be performed by a board-certified allergist. The test consists of skin-prick and intradermal testing with the major determinant (penicilloyl-polylysine), the minor determinant (penicillin G), a negative control (normal saline), and a positive control (histamine). Minor-determinant mix is not commercially available in the United States.
Results of skin-prick testing are read 15 minutes after application. A positive response is a wheal at least 3 mm larger in diameter (with equivalent erythema) than the negative control done simultaneously. Intradermal testing is only done after a negative skin-prick test. If the allergic reaction was severe (ie, anaphylaxis), skin testing should be done at least 4 to 6 weeks after the reaction.
A history of severe non–IgE-mediated reaction to penicillin is a contraindication to skin-prick testing for penicillin allergy. The positive predictive value of penicillin skin testing is 50%, and the negative predictive value is 97%.3,7,9,13
Commercial in vitro testing (serum-specific IgE assays) for IgE-mediated hypersensitivity to penicillin is inferior to skin testing in terms of the negative predictive value and is not a suitable substitute for penicillin skin testing.
MANAGING PENICILLIN ALLERGY
If skin testing is positive, use another antibiotic, or refer for desensitization
If penicillin skin testing is positive (Figure 2), use another antibiotic that is equally efficacious. Patients who absolutely need a beta-lactam may undergo drug desensitization, performed by a board-certified allergist.
During desensitization, patients receive progressively higher doses of the drug every 15 to 20 minutes subcutaneously or intravenously, or every 20 to 30 minutes orally, until a full therapeutic dose is tolerated. Most protocols begin with a dose ranging from 1/10,000 to 1/1,000 of the final dose, depending on the severity of the allergic reaction.9,17
Using modern protocols, the success rate for tolerance induction is extremely high (75% to 100% in patients with cystic fibrosis, a group with a high rate of drug allergy18–20).
Drug desensitization is contraindicated in patients with non–IgE-mediated reactions.
If skin testing is negative, refer for graded-dose challenge
If skin testing is negative (Figure 2), graded-dose challenge is recommended. This procedure must be done by a board-certified allergist. If the original reaction was life-threatening, graded-dose challenge may entail giving 1/100 of the therapeutic dose. Then, if no reaction occurs during a brief observation period (usually 30 minutes), a full dose is given. However, many patients can start with 1/10 or even a full dose of the drug, especially if the original reaction was limited to the skin and the penicillin skin test is negative.
Graded-dose challenge is contraindicated if the original reaction was a severe non–IgE-mediated reaction.
UNDERSTANDING CROSS-REACTIVITY OF PENICILLIN
Penicillin is the only antibiotic for which skin testing is reliable and validated. If a drug that cross-reacts with penicillin is needed, it is important to know the rate of cross-reactivity (Table 3). The rate of cross-reactivity between penicillin and aminopenicillins (amoxicillin and ampicillin) is less than 1.3% in the United States.10,21 However, the cross-reactivity rate among aminopenicillins and cephalosporins is between 10% to 40%. For that reason, patients with prior reactions to aminopenicillins should avoid cephalosporins that share identical R-chain side groups with aminopenicillins.9,22
The rate of cross-reactivity between penicillin and cephalosporins was reported as 10% 40 years ago.23,24 But this was with early, first-generation cephalosporins that may have been contaminated with penicillin. The cross-reactivity rate with cephalosporins today is 3%.25 In general, first- and second-generation cephalosporins cause more allergic reactions than third- and fourth-generation cephalosporins.26
Patients with a history of penicillin allergy who require a cephalosporin should still undergo penicillin skin testing. Skin testing with cephalosporins has not been validated. However, skin testing with nonirritating concentrations of cephalosporins9 may be done to elucidate IgE reactions.
In a study by Romano et al,27 110 patients who had positive results on penicillin skin testing completed graded-dose challenge with the carbapenem antibiotic imipenem. The rate of cross-reactivity between penicillin and imipenem was less than 1%.
Monobactam antibiotics do not cross-react with other beta-lactams, except ceftazidime with aztreonam. This is probably because of similarities in their chemical structure.
- Park M, Markus P, Matesic D, Li JT. Safety and effectiveness of a preoperative allergy clinic in decreasing vancomycin use in patients with a history of penicillin allergy. Ann Allergy Asthma Immunol 2006; 97:681–687.
- Arroliga ME, Radojicic C, Gordon SM, et al. A prospective observational study of the effect of penicillin skin testing on antibiotic use in the intensive care unit. Infect Control Hosp Epidemiol 2003; 24:347–350.
- del Real GA, Rose ME, Ramirez-Atamoros MT, et al. Penicillin skin testing in patients with a history of beta-lactam allergy. Ann Allergy Asthma Immunol 2007; 98:355–359.
- Nadarajah K, Green GR, Naglak M. Clinical outcomes of penicillin skin testing. Ann Allergy Asthma Immunol 2005; 95:541–545.
- Harris AD, Sauberman L, Kabbash L, Greineder DK, Samore MH. Penicillin skin testing: a way to optimize antibiotic utilization. Am J Med 1999; 107:166–168.
- Idsoe O, Guthe T, Willcox RR, de Weck AL. Nature and extent of penicillin side-reactions, with particular reference to fatalities from anaphylactic shock. Bull World Health Organ 1968; 38:159–188.
- Gadde J, Spence M, Wheeler B, Adkinson NF Jr. Clinical experience with penicillin skin testing in a large inner-city STD clinic. JAMA 1993; 270:2456–2463.
- Johansson SG, Bieber T, Dahl R, et al. Revised nomenclature for allergy for global use: report of the Nomenclature Review Committee of the World Allergy Organization, October 2003. J Allergy Clin Immunol 2004; 113:832–836.
- Joint Task Force on Practice Parameters; American Academy of Allergy, Asthma and Immunology; American College of Allergy, Asthma and Immunology; Joint Council of Allergy, Asthma and Immunology. Drug allergy: an updated practice parameter. Ann Allergy Asthma Immunol 2010; 105:259–273.
- Park MA, Matesic D, Markus PJ, Li JT. Female sex as a risk factor for penicillin allergy. Ann Allergy Asthma Immunol 2007; 99:54–58.
- Moreno E, Macias E, Davila I, Laffond E, Ruiz A, Lorente F. Hypersensitivity reactions to cephalosporins. Expert Opin Drug Saf 2008; 7:295–304.
- Salkind AR, Cuddy PG, Foxworth JW. The rational clinical examination. Is this patient allergic to penicillin? An evidence-based analysis of the likelihood of penicillin allergy. JAMA 2001; 285:2498–2505.
- Sullivan TJ, Wedner HJ, Shatz GS, Yecies LD, Parker CW. Skin testing to detect penicillin allergy. J Allergy Clin Immunol 1981; 68:171–180.
- Macy E, Schatz M, Lin C, Poon KY. The falling rate of positive penicillin skin tests from 1995 to 2007. Perm J 2009; 13:12–18.
- Blanca M, Torres MJ, Garcia JJ, et al. Natural evolution of skin test sensitivity in patients allergic to beta-lactam antibiotics. J Allergy Clin Immunol 1999; 103:918–924.
- Patel BM. Skin rash with infectious mononucleosis and ampicillin. Pediatrics 1967; 40:910–911.
- Liu A, Fanning L, Chong H, et al. Desensitization regimens for drug allergy: state of the art in the 21st century. Clin Exp Allergy 2011; 41:1679–1688.
- Burrows JA, Toon M, Bell SC. Antibiotic desensitization in adults with cystic fibrosis. Respirology 2003; 8:359–364.
- Turvey SE, Cronin B, Arnold AD, Dioun AF. Antibiotic desensitization for the allergic patient: 5 years of experience and practice. Ann Allergy Asthma Immunol 2004; 92:426–432.
- Legere HJ 3rd, Palis RI, Rodriguez Bouza T, Uluer AZ, Castells MC. A safe protocol for rapid desensitization in patients with cystic fibrosis and antibiotic hypersensitivity. J Cyst Fibros 2009; 8:418–424.
- Lin E, Saxon A, Riedl M. Penicillin allergy: value of including amoxicillin as a determinant in penicillin skin testing. Int Arch Allergy Immunol 2010; 152:313–318.
- Dickson SD, Salazar KC. Diagnosis and management of immediate hypersensitivity reactions to cephalosporins. Clin Rev Allergy Immunol 2013; 45:131–142.
- Dash CH. Penicillin allergy and the cephalosporins. J Antimicrob Chemother 1975; 1(suppl 3):107–118.
- Petz LD. Immunologic cross-reactivity between penicillins and cephalosporins: a review. J Infect Dis 1978; 137(suppl):S74–S79.
- American Academy of Allergy Asthma & Immunology. Cephalosporin administration to patients with a history of penicillin allergy. www.aaaai.org/Aaaai/media/MediaLibrary/PDF%20Documents/Practice%20and%20Parameters/Cephalosporin-administration-2009.pdf. Accessed April 2, 2015.
- Fonacier L, Hirschberg R, Gerson S. Adverse drug reactions to cephalosporins in hospitalized patients with a history of penicillin allergy. Allergy Asthma Proc 2005; 26:135–141.
- Romano A, Viola M, Gueant-Rodriguez RM, Gaeta F, Pettinato R, Gueant JL. Imipenem in patients with immediate hypersensitivity to penicillins. N Engl J Med 2006; 354:2835–2837.
- Park M, Markus P, Matesic D, Li JT. Safety and effectiveness of a preoperative allergy clinic in decreasing vancomycin use in patients with a history of penicillin allergy. Ann Allergy Asthma Immunol 2006; 97:681–687.
- Arroliga ME, Radojicic C, Gordon SM, et al. A prospective observational study of the effect of penicillin skin testing on antibiotic use in the intensive care unit. Infect Control Hosp Epidemiol 2003; 24:347–350.
- del Real GA, Rose ME, Ramirez-Atamoros MT, et al. Penicillin skin testing in patients with a history of beta-lactam allergy. Ann Allergy Asthma Immunol 2007; 98:355–359.
- Nadarajah K, Green GR, Naglak M. Clinical outcomes of penicillin skin testing. Ann Allergy Asthma Immunol 2005; 95:541–545.
- Harris AD, Sauberman L, Kabbash L, Greineder DK, Samore MH. Penicillin skin testing: a way to optimize antibiotic utilization. Am J Med 1999; 107:166–168.
- Idsoe O, Guthe T, Willcox RR, de Weck AL. Nature and extent of penicillin side-reactions, with particular reference to fatalities from anaphylactic shock. Bull World Health Organ 1968; 38:159–188.
- Gadde J, Spence M, Wheeler B, Adkinson NF Jr. Clinical experience with penicillin skin testing in a large inner-city STD clinic. JAMA 1993; 270:2456–2463.
- Johansson SG, Bieber T, Dahl R, et al. Revised nomenclature for allergy for global use: report of the Nomenclature Review Committee of the World Allergy Organization, October 2003. J Allergy Clin Immunol 2004; 113:832–836.
- Joint Task Force on Practice Parameters; American Academy of Allergy, Asthma and Immunology; American College of Allergy, Asthma and Immunology; Joint Council of Allergy, Asthma and Immunology. Drug allergy: an updated practice parameter. Ann Allergy Asthma Immunol 2010; 105:259–273.
- Park MA, Matesic D, Markus PJ, Li JT. Female sex as a risk factor for penicillin allergy. Ann Allergy Asthma Immunol 2007; 99:54–58.
- Moreno E, Macias E, Davila I, Laffond E, Ruiz A, Lorente F. Hypersensitivity reactions to cephalosporins. Expert Opin Drug Saf 2008; 7:295–304.
- Salkind AR, Cuddy PG, Foxworth JW. The rational clinical examination. Is this patient allergic to penicillin? An evidence-based analysis of the likelihood of penicillin allergy. JAMA 2001; 285:2498–2505.
- Sullivan TJ, Wedner HJ, Shatz GS, Yecies LD, Parker CW. Skin testing to detect penicillin allergy. J Allergy Clin Immunol 1981; 68:171–180.
- Macy E, Schatz M, Lin C, Poon KY. The falling rate of positive penicillin skin tests from 1995 to 2007. Perm J 2009; 13:12–18.
- Blanca M, Torres MJ, Garcia JJ, et al. Natural evolution of skin test sensitivity in patients allergic to beta-lactam antibiotics. J Allergy Clin Immunol 1999; 103:918–924.
- Patel BM. Skin rash with infectious mononucleosis and ampicillin. Pediatrics 1967; 40:910–911.
- Liu A, Fanning L, Chong H, et al. Desensitization regimens for drug allergy: state of the art in the 21st century. Clin Exp Allergy 2011; 41:1679–1688.
- Burrows JA, Toon M, Bell SC. Antibiotic desensitization in adults with cystic fibrosis. Respirology 2003; 8:359–364.
- Turvey SE, Cronin B, Arnold AD, Dioun AF. Antibiotic desensitization for the allergic patient: 5 years of experience and practice. Ann Allergy Asthma Immunol 2004; 92:426–432.
- Legere HJ 3rd, Palis RI, Rodriguez Bouza T, Uluer AZ, Castells MC. A safe protocol for rapid desensitization in patients with cystic fibrosis and antibiotic hypersensitivity. J Cyst Fibros 2009; 8:418–424.
- Lin E, Saxon A, Riedl M. Penicillin allergy: value of including amoxicillin as a determinant in penicillin skin testing. Int Arch Allergy Immunol 2010; 152:313–318.
- Dickson SD, Salazar KC. Diagnosis and management of immediate hypersensitivity reactions to cephalosporins. Clin Rev Allergy Immunol 2013; 45:131–142.
- Dash CH. Penicillin allergy and the cephalosporins. J Antimicrob Chemother 1975; 1(suppl 3):107–118.
- Petz LD. Immunologic cross-reactivity between penicillins and cephalosporins: a review. J Infect Dis 1978; 137(suppl):S74–S79.
- American Academy of Allergy Asthma & Immunology. Cephalosporin administration to patients with a history of penicillin allergy. www.aaaai.org/Aaaai/media/MediaLibrary/PDF%20Documents/Practice%20and%20Parameters/Cephalosporin-administration-2009.pdf. Accessed April 2, 2015.
- Fonacier L, Hirschberg R, Gerson S. Adverse drug reactions to cephalosporins in hospitalized patients with a history of penicillin allergy. Allergy Asthma Proc 2005; 26:135–141.
- Romano A, Viola M, Gueant-Rodriguez RM, Gaeta F, Pettinato R, Gueant JL. Imipenem in patients with immediate hypersensitivity to penicillins. N Engl J Med 2006; 354:2835–2837.
KEY POINTS
- The prevalence of reported penicillin allergy is 10% in the general population. However, more than 90% of these patients are found not to be allergic to penicillin after skin testing.
- In patients found to have penicillin allergy, the frequency of positive results on skin testing decreases by 10% per year of avoidance. Therefore, 80% to 100% of patients are expected to test negative for penicillin allergy by 10 years after their reaction.
- Skin testing for penicillin allergy is only useful for type 1 IgE-mediated reactions. However, in properly selected patients, the negative predictive value of penicillin skin testing is nearly 97%.
- The rate of cross-reactivity between penicillin and cephalosporins is approximately 3%.
Morphea-like plaques, induration of the extremities, and eosinophilia
A 52-year-old woman who had been diagnosed with morphea 1 year before was referred to our department for evaluation of a 6-week history of myalgia and limited flexion and extension of the wrists and elbows. She had a history of hypertension, which was controlled with enalapril 10 mg daily. She had no fever, weight loss, dyspnea, dysphagia, Raynaud phenomenon, or other symptoms, and she had not traveled recently.
On physical examination, she had multiple dark-brown plaques on the legs (Figure 1), induration of both forearms, puckering of the skin (peau d’orange) of both arms (Figure 1), and marked limitation of motion of the wrists (Figure 2). Sclerodactyly was absent, and the rest of the examination was unremarkable. Laboratory tests showed an elevated serum C-reactive protein (1.97 mg/dL, reference range 0–0.5) and a white blood cell count of 9.59 × 109/L, with 25.2% eosinophils (reference range 0%–7%). Antinuclear antibodies were negative.
Because the patient refused full-thickness biopsy of the skin (ie, down to superficial muscle), magnetic resonance imaging (MRI) of the right arm was ordered. T1-weighted fat-saturated imaging with contrast showed marked contrast enhancement along all muscle fascia. This finding, along with the patient’s presentation and the results of laboratory testing, indicated a diagnosis of eosinophilic fasciitis.
Prednisone was prescribed in an initial dose of 1 mg/kg/day and was subsequently tapered, and this brought a partial response. Methotrexate (15 mg weekly) was added 1 month later. Follow-up visits at 3, 6, and 12 months were scheduled. At 12 months, levels of markers of inflammation were normal, the eosinophilia had completely resolved (< 0.125 × 109/L, < 2%), joint mobility had improved (Figure 2), and the morphea-like lesions had lightened somewhat. MRI repeated 6 months after the start of methotrexate showed complete resolution of the fascia infiltration.
EOSINOPHILIC FASCIITIS
Eosinophilic fasciitis, or Shulman syndrome, is a rare localized fibrosing disorder of the fascia characterized in its early phase by symmetrical limb or trunk erythema and swelling, and later by progressive thickening and induration of the dermis and subcutaneous fascia.1
The cause is unknown, but triggers have been suggested such as vigorous exercise or trauma, Borrelia infection, and drugs (statins, phenytoin, l-tryptophan). It has also been associated with hematologic disease. In our patient, none of these was present.2,3
There are no international diagnostic criteria for eosinophilic fasciitis. Rather, the diagnosis is suspected in a patient with eosinophilia and characteristic cutaneous features, such as pitting edema, induration, hyperpigmentation, and a peau d’orange aspect caused by inflammation and fibrosis of the fascia.1,2 These features have been reported in up to 90% of patients at the time of diagnosis.1–3 Myalgia, muscle weakness, and articular involvement including inflammatory arthralgia and joint contracture, as in our patient, are also common.1,2,4,5 Raynaud phenomenon in these patients is typically absent. Furthermore, pulmonary, cardiac, and renal involvement has also been reported, but this is very infrequent, and if it is present, other diseases such as Churg-Strauss vasculitis and hypereosinophilic syndrome should be excluded.1,2,4
The presence of thickened fascia and inflammatory infiltrates composed of lymphocytes or eosinophils (or both) on histologic examination of the skin-to-muscle biopsy remains the gold standard for diagnosis.2,4 However, when biopsy is nondiagnostic or is not possible, imaging with MRI or ultrasonography has been helpful.6,7 Muscle MRI is considered the best morphologic procedure for diagnosis, as it can specifically indicate the optimal location for biopsy in atypical cases (eg, fasciitis without skin changes and when eosinophilia is absent); it is also used to monitor the response to therapy.7,8 Although nonspecific, muscle MRI typically shows a markedly increased signal intensity within the fascia. And gadolinium administration shows marked fascia enhancement in the acute phase of the disease.1,4,6–8
MORPHEA PREDICTS POOR OUTCOME
Morphea has been reported to be present in 19% to 41% of patients with eosinophilic fasciitis due to dermis infiltration.2,4,5 Although rare, morphea plaques may also be present before the onset of eosinophilic fasciitis.3,4 Morphea has been described as predictive of poor outcome and residual fibrosis, and it has been associated with eosinophilic fasciitis resistant to usual therapies.3,5
Treatment of eosinophilic fasciitis is largely empirical because evidence from controlled trials is lacking. Corticosteroids remain the standard therapy. However, in patients with morphea-like lesions, treatment with an immunosuppressive drug such as methotrexate has been reported to be useful.1,4
- Lebeaux D, Sène D. Eosinophilic fasciitis (Shulman disease). Best Pract Res Clin Rheumatol 2012; 26:449–458.
- Lakhanpal S, Ginsburg WW, Michet CJ, Doyle JA, Moore SB. Eosinophilic fasciitis: clinical spectrum and therapeutic response in 52 cases. Semin Arthritis Rheum 1988; 17:221–231.
- Moulin C, Cavailhes A, Balme B, Skowron F. Morphoea-like plaques revealing an eosinophilic (Shulman) fasciitis. Clin Exp Dermatol 2009; 34:e851–e853.
- Lebeaux D, Francès C, Barete S, et al. Eosinophilic fasciitis (Shulman disease): new insights into the therapeutic management from a series of 34 patients. Rheumatology (Oxford) 2012; 51:557–561.
- Endo Y, Tamura A, Matsushima Y, et al. Eosinophilic fasciitis: report of two cases and a systematic review of the literature dealing with clinical variables that predict outcome. Clin Rheumatol 2007; 26:1445–1451.
- Dybowski F, Neuen-Jacob E, Braun J. Eosinophilic fasciitis and myositis: use of imaging modalities for diagnosis and monitoring. Ann Rheum Dis 2008; 67:572–574.
- Moulton SJ, Kransdorf MJ, Ginsburg WW, Abril A, Persellin S. Eosinophilic fasciitis: spectrum of MRI findings. AJR Am J Roentgenol 2005; 184:975–978.
- Ronneberger M, Janka R, Schett G, Manger B. Can MRI substitute for biopsy in eosinophilic fasciitis? Ann Rheum Dis 2009; 68:1651–1652.
A 52-year-old woman who had been diagnosed with morphea 1 year before was referred to our department for evaluation of a 6-week history of myalgia and limited flexion and extension of the wrists and elbows. She had a history of hypertension, which was controlled with enalapril 10 mg daily. She had no fever, weight loss, dyspnea, dysphagia, Raynaud phenomenon, or other symptoms, and she had not traveled recently.
On physical examination, she had multiple dark-brown plaques on the legs (Figure 1), induration of both forearms, puckering of the skin (peau d’orange) of both arms (Figure 1), and marked limitation of motion of the wrists (Figure 2). Sclerodactyly was absent, and the rest of the examination was unremarkable. Laboratory tests showed an elevated serum C-reactive protein (1.97 mg/dL, reference range 0–0.5) and a white blood cell count of 9.59 × 109/L, with 25.2% eosinophils (reference range 0%–7%). Antinuclear antibodies were negative.
Because the patient refused full-thickness biopsy of the skin (ie, down to superficial muscle), magnetic resonance imaging (MRI) of the right arm was ordered. T1-weighted fat-saturated imaging with contrast showed marked contrast enhancement along all muscle fascia. This finding, along with the patient’s presentation and the results of laboratory testing, indicated a diagnosis of eosinophilic fasciitis.
Prednisone was prescribed in an initial dose of 1 mg/kg/day and was subsequently tapered, and this brought a partial response. Methotrexate (15 mg weekly) was added 1 month later. Follow-up visits at 3, 6, and 12 months were scheduled. At 12 months, levels of markers of inflammation were normal, the eosinophilia had completely resolved (< 0.125 × 109/L, < 2%), joint mobility had improved (Figure 2), and the morphea-like lesions had lightened somewhat. MRI repeated 6 months after the start of methotrexate showed complete resolution of the fascia infiltration.
EOSINOPHILIC FASCIITIS
Eosinophilic fasciitis, or Shulman syndrome, is a rare localized fibrosing disorder of the fascia characterized in its early phase by symmetrical limb or trunk erythema and swelling, and later by progressive thickening and induration of the dermis and subcutaneous fascia.1
The cause is unknown, but triggers have been suggested such as vigorous exercise or trauma, Borrelia infection, and drugs (statins, phenytoin, l-tryptophan). It has also been associated with hematologic disease. In our patient, none of these was present.2,3
There are no international diagnostic criteria for eosinophilic fasciitis. Rather, the diagnosis is suspected in a patient with eosinophilia and characteristic cutaneous features, such as pitting edema, induration, hyperpigmentation, and a peau d’orange aspect caused by inflammation and fibrosis of the fascia.1,2 These features have been reported in up to 90% of patients at the time of diagnosis.1–3 Myalgia, muscle weakness, and articular involvement including inflammatory arthralgia and joint contracture, as in our patient, are also common.1,2,4,5 Raynaud phenomenon in these patients is typically absent. Furthermore, pulmonary, cardiac, and renal involvement has also been reported, but this is very infrequent, and if it is present, other diseases such as Churg-Strauss vasculitis and hypereosinophilic syndrome should be excluded.1,2,4
The presence of thickened fascia and inflammatory infiltrates composed of lymphocytes or eosinophils (or both) on histologic examination of the skin-to-muscle biopsy remains the gold standard for diagnosis.2,4 However, when biopsy is nondiagnostic or is not possible, imaging with MRI or ultrasonography has been helpful.6,7 Muscle MRI is considered the best morphologic procedure for diagnosis, as it can specifically indicate the optimal location for biopsy in atypical cases (eg, fasciitis without skin changes and when eosinophilia is absent); it is also used to monitor the response to therapy.7,8 Although nonspecific, muscle MRI typically shows a markedly increased signal intensity within the fascia. And gadolinium administration shows marked fascia enhancement in the acute phase of the disease.1,4,6–8
MORPHEA PREDICTS POOR OUTCOME
Morphea has been reported to be present in 19% to 41% of patients with eosinophilic fasciitis due to dermis infiltration.2,4,5 Although rare, morphea plaques may also be present before the onset of eosinophilic fasciitis.3,4 Morphea has been described as predictive of poor outcome and residual fibrosis, and it has been associated with eosinophilic fasciitis resistant to usual therapies.3,5
Treatment of eosinophilic fasciitis is largely empirical because evidence from controlled trials is lacking. Corticosteroids remain the standard therapy. However, in patients with morphea-like lesions, treatment with an immunosuppressive drug such as methotrexate has been reported to be useful.1,4
A 52-year-old woman who had been diagnosed with morphea 1 year before was referred to our department for evaluation of a 6-week history of myalgia and limited flexion and extension of the wrists and elbows. She had a history of hypertension, which was controlled with enalapril 10 mg daily. She had no fever, weight loss, dyspnea, dysphagia, Raynaud phenomenon, or other symptoms, and she had not traveled recently.
On physical examination, she had multiple dark-brown plaques on the legs (Figure 1), induration of both forearms, puckering of the skin (peau d’orange) of both arms (Figure 1), and marked limitation of motion of the wrists (Figure 2). Sclerodactyly was absent, and the rest of the examination was unremarkable. Laboratory tests showed an elevated serum C-reactive protein (1.97 mg/dL, reference range 0–0.5) and a white blood cell count of 9.59 × 109/L, with 25.2% eosinophils (reference range 0%–7%). Antinuclear antibodies were negative.
Because the patient refused full-thickness biopsy of the skin (ie, down to superficial muscle), magnetic resonance imaging (MRI) of the right arm was ordered. T1-weighted fat-saturated imaging with contrast showed marked contrast enhancement along all muscle fascia. This finding, along with the patient’s presentation and the results of laboratory testing, indicated a diagnosis of eosinophilic fasciitis.
Prednisone was prescribed in an initial dose of 1 mg/kg/day and was subsequently tapered, and this brought a partial response. Methotrexate (15 mg weekly) was added 1 month later. Follow-up visits at 3, 6, and 12 months were scheduled. At 12 months, levels of markers of inflammation were normal, the eosinophilia had completely resolved (< 0.125 × 109/L, < 2%), joint mobility had improved (Figure 2), and the morphea-like lesions had lightened somewhat. MRI repeated 6 months after the start of methotrexate showed complete resolution of the fascia infiltration.
EOSINOPHILIC FASCIITIS
Eosinophilic fasciitis, or Shulman syndrome, is a rare localized fibrosing disorder of the fascia characterized in its early phase by symmetrical limb or trunk erythema and swelling, and later by progressive thickening and induration of the dermis and subcutaneous fascia.1
The cause is unknown, but triggers have been suggested such as vigorous exercise or trauma, Borrelia infection, and drugs (statins, phenytoin, l-tryptophan). It has also been associated with hematologic disease. In our patient, none of these was present.2,3
There are no international diagnostic criteria for eosinophilic fasciitis. Rather, the diagnosis is suspected in a patient with eosinophilia and characteristic cutaneous features, such as pitting edema, induration, hyperpigmentation, and a peau d’orange aspect caused by inflammation and fibrosis of the fascia.1,2 These features have been reported in up to 90% of patients at the time of diagnosis.1–3 Myalgia, muscle weakness, and articular involvement including inflammatory arthralgia and joint contracture, as in our patient, are also common.1,2,4,5 Raynaud phenomenon in these patients is typically absent. Furthermore, pulmonary, cardiac, and renal involvement has also been reported, but this is very infrequent, and if it is present, other diseases such as Churg-Strauss vasculitis and hypereosinophilic syndrome should be excluded.1,2,4
The presence of thickened fascia and inflammatory infiltrates composed of lymphocytes or eosinophils (or both) on histologic examination of the skin-to-muscle biopsy remains the gold standard for diagnosis.2,4 However, when biopsy is nondiagnostic or is not possible, imaging with MRI or ultrasonography has been helpful.6,7 Muscle MRI is considered the best morphologic procedure for diagnosis, as it can specifically indicate the optimal location for biopsy in atypical cases (eg, fasciitis without skin changes and when eosinophilia is absent); it is also used to monitor the response to therapy.7,8 Although nonspecific, muscle MRI typically shows a markedly increased signal intensity within the fascia. And gadolinium administration shows marked fascia enhancement in the acute phase of the disease.1,4,6–8
MORPHEA PREDICTS POOR OUTCOME
Morphea has been reported to be present in 19% to 41% of patients with eosinophilic fasciitis due to dermis infiltration.2,4,5 Although rare, morphea plaques may also be present before the onset of eosinophilic fasciitis.3,4 Morphea has been described as predictive of poor outcome and residual fibrosis, and it has been associated with eosinophilic fasciitis resistant to usual therapies.3,5
Treatment of eosinophilic fasciitis is largely empirical because evidence from controlled trials is lacking. Corticosteroids remain the standard therapy. However, in patients with morphea-like lesions, treatment with an immunosuppressive drug such as methotrexate has been reported to be useful.1,4
- Lebeaux D, Sène D. Eosinophilic fasciitis (Shulman disease). Best Pract Res Clin Rheumatol 2012; 26:449–458.
- Lakhanpal S, Ginsburg WW, Michet CJ, Doyle JA, Moore SB. Eosinophilic fasciitis: clinical spectrum and therapeutic response in 52 cases. Semin Arthritis Rheum 1988; 17:221–231.
- Moulin C, Cavailhes A, Balme B, Skowron F. Morphoea-like plaques revealing an eosinophilic (Shulman) fasciitis. Clin Exp Dermatol 2009; 34:e851–e853.
- Lebeaux D, Francès C, Barete S, et al. Eosinophilic fasciitis (Shulman disease): new insights into the therapeutic management from a series of 34 patients. Rheumatology (Oxford) 2012; 51:557–561.
- Endo Y, Tamura A, Matsushima Y, et al. Eosinophilic fasciitis: report of two cases and a systematic review of the literature dealing with clinical variables that predict outcome. Clin Rheumatol 2007; 26:1445–1451.
- Dybowski F, Neuen-Jacob E, Braun J. Eosinophilic fasciitis and myositis: use of imaging modalities for diagnosis and monitoring. Ann Rheum Dis 2008; 67:572–574.
- Moulton SJ, Kransdorf MJ, Ginsburg WW, Abril A, Persellin S. Eosinophilic fasciitis: spectrum of MRI findings. AJR Am J Roentgenol 2005; 184:975–978.
- Ronneberger M, Janka R, Schett G, Manger B. Can MRI substitute for biopsy in eosinophilic fasciitis? Ann Rheum Dis 2009; 68:1651–1652.
- Lebeaux D, Sène D. Eosinophilic fasciitis (Shulman disease). Best Pract Res Clin Rheumatol 2012; 26:449–458.
- Lakhanpal S, Ginsburg WW, Michet CJ, Doyle JA, Moore SB. Eosinophilic fasciitis: clinical spectrum and therapeutic response in 52 cases. Semin Arthritis Rheum 1988; 17:221–231.
- Moulin C, Cavailhes A, Balme B, Skowron F. Morphoea-like plaques revealing an eosinophilic (Shulman) fasciitis. Clin Exp Dermatol 2009; 34:e851–e853.
- Lebeaux D, Francès C, Barete S, et al. Eosinophilic fasciitis (Shulman disease): new insights into the therapeutic management from a series of 34 patients. Rheumatology (Oxford) 2012; 51:557–561.
- Endo Y, Tamura A, Matsushima Y, et al. Eosinophilic fasciitis: report of two cases and a systematic review of the literature dealing with clinical variables that predict outcome. Clin Rheumatol 2007; 26:1445–1451.
- Dybowski F, Neuen-Jacob E, Braun J. Eosinophilic fasciitis and myositis: use of imaging modalities for diagnosis and monitoring. Ann Rheum Dis 2008; 67:572–574.
- Moulton SJ, Kransdorf MJ, Ginsburg WW, Abril A, Persellin S. Eosinophilic fasciitis: spectrum of MRI findings. AJR Am J Roentgenol 2005; 184:975–978.
- Ronneberger M, Janka R, Schett G, Manger B. Can MRI substitute for biopsy in eosinophilic fasciitis? Ann Rheum Dis 2009; 68:1651–1652.